You are on page 1of 171

Student Guide 1

PREFACE



NTS, a project of its own kind is aimed at setting standards of excellence in
educational testing. NTS is a rapidly growing and credible entity of national
repute. Its context and potential coincides with the expressed need of the
Government for the establishment of a National Accreditation Council and
Testing Service.

Establishment of this set-up has been necessitated by the prevailing lack of
credibility and absence of standardized testing facilities in our national
Institutes. Institutions waste massive amount of work hours in preparing
and conducting diverse kind of entrance tests with least credible and
authentic consequences.

NTS has made a bid to restore some order to the growing chaos in the field
of testing. It has developed the capability to offer the testing services to all
national institutes providing education. Its content is aimed at identifying
the ability-oriented strengths among prospective candidates for IT education.
NTS is a multifaceted and comprehensive facility that can provide services at
an enormous scale in all dimensions of testing. Based on a vision aimed at
providing quality services in educational testing and assessment mechanisms
for various stakeholders, NTS is committed to promoting the overall quality
of education as a value-added adjunct.

It also extends Services for the employed & unemployed professionals to get
satisfactory jobs in their relevant fields through an assessment of their
knowledge base, skills, and professional expertise & abilities. NTS is a
unique and trend-setting venture that has shown great promise.
Student Guide 2

1 NTS INTRODUCTION

NTS was formed in 2001 with the basic objective of establishing a credible testing
system to evaluate the knowledge level of students against a consistent and
standard yardstick. It was felt that although our universities and other educational
institutions are producing a large number of qualified professional and non-
professional manpower, there is no single standard for quality evaluation of this
educated human resource. Since there is no single syllabus right from basic to the
advanced levels of education, the normal performance evaluation procedures fail to
measure the quality as well as the extent of discrepancy between the existing and
the desirable quality for individual candidates.

Similarly, a big variation in syllabi and the apparent status of the educational
institutions have created a huge gap between the haves and have-nots of the
society. The students from big name institutions always have an added advantage
where mediocre students are preferred over excellent students from low status
institutes because of the brand name of the institutions they carry with them. NTS
addresses these important issues by providing testing procedures that evaluate the
knowledge and skills of individuals on a unified scale using standardized measures
that are independent of class differentiation.

In a short period of three years, NTS has made a significant contribution by
developing and implementing training and testing programs for individual students,
educational institutions and the potential employers of the country. The hallmarks of
the NTS system are its consistency, transparency, reliability and efficiency. Being a
non-profit organization, it takes pride in offering a cost-efficient system that could
meet the needs of individuals and organizations to their utmost satisfaction.

NTS continues to strive for excellence through research and development in its
testing system and related services. The goal of the organization is to keep
contributing towards improvement in education by building standards for educational
and professional testing. Its ultimate aim is to be recognized nationally and
internationally as a credible and reliable name in the field of testing.

1.1 NTS BACKGROUND

There i s an ever-i ncreasi ng demand of qual i ty professi onal s around the gl obe.
Our uni versi ti es and other educati onal i nsti tuti ons are produci ng l arge
quanti ti es of professi onal s , yet there appears to be no si ngl e standard f or the
perf ormance eval uati on of these professi onal s and thei r qual i ty assurance.
Si nce there isn t a si ngl e standard syl l abus, the performance eval uati on
procedures f ai l to measure the qual i ty as wel l as the extent of di screpancy
between the exi sti ng and the desi rabl e qual i ty of i ndi vi dual candi date. This
si tuati on i s rai si ng a l ack of confi dence among the candi dates and the
entrepreneurs.
To al l evi ate thi s, there i s an i mmedi ate need f or i ncorporati ng an autonomous
and a soverei gn body, whi ch shoul d cater to the needs o f the entrepreneur and
harness the i ndi vi dual tal ent. Thi s body shoul d be made responsi bl e for
bri dgi ng the gap between the educati on i ndustry and the practi cal /professi onal
i ndustry by provi di ng an eval uati on and testi ng standard . Thi s Non
Government Organi zati on (NGO), under the name of NTS Nati onal Testi ng
Servi ce faci l i tates the eval uati on of graduate or undergraduate candi dates for
admi ssi on to any uni versi ty. NTS wi l l not onl y l ead the educati onal i ndustry to
Student Guide 3
a heal thy competi ti on but al so i mprove the standards of the educati onal
i nsti tuti ons i n al l fi el ds.

1.2. NTS Objectives
Wi thi n the overal l premi se of contri buti ng to the i mprovement of the qual i ty of
educati on i n Paki stan, the i mportant obj ecti ves of NTS i ncl ude:

To i mprove upon the shortcomi ngs i n our prevai l i ng performance eval uati on
system of candi dates through establ i shi ng a credi bl e, techni cal l y
appropri ate and accurate system of performance eval uati on for students of
di f f erent educati onal i nsti tuti ons at vari ous l evel s.

To provi de a sta ndardi zed and uni form cri teri on for sel ecti ng candi dates
seeki ng admi ssi on to vari ous uni versi ti es i n the country and abroad and
al so to provi de a basi s on whi ch students, i ndi vi dual s and i nsti tuti ons can
assess perf ormance f or preparati on of vari ous types of tests conducted by
di f f erent i nternati onal testi ng agenci es.

Enabl i ng assessment and ranki ng not onl y i n a parti cul ar subject on a
general competency l evel but al so hel p i n i denti fi cati on of areas of
strengths and weaknesses i n i ts sub-contents on a d i saggregated basi s.


To provi de a rel i abl e and effi ci ent system for eval uati ng the knowl edge
base and ski l l s of candi dates i ncl udi ng professi onal s - empl oyed and
unempl oyed.

Establ i shi ng a credi bl e and rel i abl e measure of assessi ng the students
knowl edge i n commonl y taught programs for rel ati ve ranki ng.


To bri dge the gap between academi c preparati on and the practi cal market
needs i n a hi ghl y dynami c and competi ti ve envi ronment.

1.3 NTS COMMITMENT

National Testing Service (NTS) is a non profit, autonomous organization established
with the basic objective of developing a credible, technically appropriate and
accurate system to evaluate the knowledge level of students of different educational
institutions at various levels. It thus assists in carrying out an estimated ranking of
students for admissions in various educational institutions and also provides potential
employers with a credible yardstick for evaluation and induction of quality human
resource. NTS operates on national level and is committed to:

1.3.1 Promoting Overall Quality of Education
Based on a vi si on ai med at provi di ng qual i ty Servi ces i n educati onal testi ng
and assessme nt mechani sms for vari ous stakehol ders, NTS i s commi tted to
promoti ng the overal l qual i ty of educati on as a val ue-added adj unct.

1.3.2 Performance Evaluation
NTS undertakes programs that are ai med at establ i shi ng a credi bl e, techni cal l y
appropri ate and accurate system of performance eval uati on of
students/candi dates of vari ous educati onal i nsti tuti ons at di fferent l evel s.

1.3.3 Assessing Knowledge, Skills & Professional Abilities
Student Guide 4
It al so extends servi ces f or the empl oyed and unempl oyed prof essi onal s to get
sati sfactory j obs i n thei r rel evant area through an assessment of thei r
knowl edge base, ski l l s, and prof essi onal abi l i ti es.

1.3.4 A Rapidly Growing & Credible Entity
NTS i s a rapi dl y growi ng and credi bl e enti ty of nati onal repute. Its context and
potenti al coi nci des wi t h the expressed need of the Government for the
establ i shment of a Nati onal Accredi tati on Counci l and Testi ng Servi ce.

1.3.5 Conducting Various Types of Tests
Whi l e the begi nni ngs were made by conducti ng General Tests through an IT-
focused thrust, NTS has expand ed i ts servi ces to other di sci pl i nes of
educati onal testi ng. It now provi des servi ces for conducti ng Subject Tests by
provi di ng faci l i ti es both for i ndi vi dual candi dates and i nsti tuti ons. These tests
are prepared and assessed by experts and prof essi onal s f rom rel evant fi el ds.

1.3.6 Linkages & Networks
NTS now has growi ng l i nkages and associ ati ons wi th educati onal and
prof essi onal organi zati ons & Insti tuti ons, i ncl udi ng those engaged i n tal ent
search and career & educati onal promoti on acti vi ti es, at bot h nati onal and
i nternati onal l evel s.

1.4 NTS- BENEFITS

NTS ai ms at bei ng benef i ci al f or both students and i nsti tutes.

1.4.1 Students
NTS Tests enabl e students to draw benef i ts i n a number of ways:

It provi des a si ngl e standard f or measuri ng knowl edge base and al so ski l l
level eval uati on of candi dates.
Candi dates can have a more transparent, credi bl e and standardi zed
assessment of thei r knowl edge.
Besi des bei ng economi cal , NTS decl ares the test resul ts i n just a few days
ti me.
NTS scores are a rel i abl e source to measure candi dates knowl edge i n
commonl y taught academi c courses.
Testi ng through NTS automati cal l y l eads to standardi zati on at a nati onal
l evel , whi ch el i mi nates chances of l arge dropouts duri ng the course of
educati on.

1.4.2 Institutions
Besi des students, NTS i s also useful for the educati onal i nsti tutes and si mi l ar
enti ti es:

Insti tutes, wi shi ng to get admi ssi on tests prepared for thei r candi dates,
and al so wanti ng to get them scored i n a more transparent and i n a l ess
ti me consumi ng manner can rel y on NTS.
Thi s testi ng servi ce i s hi ghl y rel i abl e, effi ci ent and standardi zed; so any of
the i nsti tutes can share thei r responsi bi l i ty wi th NTS.
The preparati on and hol di ng of tests consume a l ot of ti me and ef f ort. The
i nsti tutes can l essen thi s burden of responsi bi l i ty of conducti ng tests by
shari ng i t wi th NTS. Preparati on of regi strati on forms, rol l number sl i ps,
questi on papers, answer sheets, sati sfactory arrangements for i nvi gi l ati on,
paper marki ng, resul t decl arati on etc. are arranged by NTS.
The Insti tutes can total l y rel y on NTS for effi ci ency of Servi ces i ncl udi ng
transparency of resul ts.
Student Guide 5

The Insti tutes are provi ded wi th a credi bl e system of perf ormance ranki ng.

1.5 NTS SERVICES

NTS provi des i t s servi ces i n vast areas:

1.5.1 National Aptitude/General Testing
NTS conducts General Tests so as to eval uate the cri ti cal thi nki ng ski l l s of
anal yti cal reasoni ng and al so perf ormance assessment of candi dates wi th
regard to quanti tati ve & verbal abi l i ty. General Tests al so cal l ed Nati onal
Apti tude Tests, hel p establ i shi ng a rel i abl e and credi bl e measure of j udgi ng
t he knowl edge base of st udent s i n commonl y t aught programs.

1.5.2 Services - Measuring Knowledge Base and Skill Levels
The NTS Subj ect Tests are prepared and desi gned wi th a vi ew to assessi ng
the qual i f i cati ons, knowledge base, competency and ski l l l evel of the
candi dates i n a speci f i c area of study.

1.5.3 Services for Test Preparation
NTS provi des compl ete gui del i nes wi th regard to Test Preparati on. A gui de
f or users i s avai l abl e on our website as wel l as i n the market i n pri nted form.
Model Questi ons and Sampl e Tests are al so avai l abl e on the websi te f or
hel pi ng the candi dates to practi ce and be wel l prepared f or the test.
Furthermore Ref erence Books are al so recommended by NTS to be an
addi ti onal source of hel p f or the test takers for computer Sci ence Subject
Test s.

1.5.4 Administrative Services
NTS provi des an admi ni strati ve servi ce f or not onl y t he candi dat es but al so
the i nsti tuti ons. Al l admi ni strati ve arrangements regardi ng regi strati on, test
center sel ecti on, test exe cuti on, resul t preparati on and then i ts reporti ng etc,
are t aken as a responsi bi l i t y by NTS.

1.5.5 Admission Test Services for Institutions
NTS conducts Admi ssi on Tests f or vari ous i nsti tutes and i nsti tuti ons, al l
servi ces l i ke the arrangements of test regi stra ti on, paper generati on, test
executi on, paper marki ng, resul t decl arati on etc. are pro vi ded by NTS f or
t hese enti ti es.

1.5.6 Services for Online Testing
NTS al so conducts Onl i ne Tests f or the candi dates ef f i ci ent and credi bl e
eval uat i on of t hei r knowl edge and ski l l s. Arrangements l i ke the avai l abi l i ty
of l abs, content devel opment f or the assessment, onl i ne paper generati on and
test executi on and onl i ne resul t f ormati on i mmedi atel y af ter the test, are
made by NTS.

1.5.7 Establishing Performance Ranking
NTS al so assi sts i n carryi ng out an esti mated ranki ng of students as wel l as
prof essi onal s. Thi s ranki ng i s not onl y i n a parti cul ar subj ect on a general
competency l evel but al so hel ps i n the i denti f i cati on of areas of strengths and
weaknesses i n i ts sub-cont ent s on a di saggregat ed basi s.




Student Guide 6
1.5.8 Services for Score Reporting
For General Tests, NTS decl ares i ts resul ts el ectroni cal l y wi thi n seven days of
the test bei ng conducted, the resul ts of each candi date are sent to thei r
homes through a f ast and rel i abl e couri er servi ce.

The resul t of Onl i ne Subj ect Tests i s di spl ayed onl i ne i mmedi atel y af ter the
candi date compl etes hi s/her test. The score of candi dates i s al so reported i n
t he f orm of a cert i f i cat e at t hei r respect i ve addresses.

1.5.9 Services to Facilitate Employment
NTS extends i ts servi ces f or empl oyed and unempl oyed candi dates to get
sati sf actory j obs i n thei r rel evant areas through a very esti mated and caref ul
j udgment of t hei r knowl edge base and ski l l s.

It f aci l i tates access to j ob opportuni ti es f or vari ous professi onal s and
candi dates both wi thi n the country and abroad on the basi s of thei r scores i n
NTS Subj ect Tests.

1.5.10 Statistical Analysis and Reports
NTS by vi rtue of i t s f l exi bl e and user f ri endl y sol uti on can better hel p the
i nst i t ut i ons and organi zati ons to have stati sti cal anal ysi s and report f or
admi ni strati ve tasks and i t wi l l ul ti matel y l ead to true deci si on maki ng and
manageri al tasks.

1.5.11 Survey
NTS has very robust and accurate system f or processi ng f orms and surveys.
In addi ti on to the educati onal and prof essi onal testi ng servi ces NTS has the
survey f aci l i ty f or di f f erent organi zati ons, government data col l ecti on and
other methods to get f eedback f rom peopl e.

1.6 NTS PARTNERS

1.6.1 COMSATS Institute of Information Technology

COMSATS, through a noti f i cati on i n 1979, setup the COMSATS Insti tute of
Inf ormati on Technol ogy (CIIT), whi ch became f uncti onal i n Apri l 1998. CI I T
i s a worl d-cl ass teachi ng i nsti tute, creati ng knowl edge base and advance d
t echnol ogi es t hat are i nt ended t o shape t he new directi on of the country and
educate i ts f uture l eaders f or the 21st century.

The ai m of the Insti tute i s to trai n the country s human resource i n the f i el d
of Inf ormati on Technol ogy so as to equi p them wi th emergi ng IT trends. The
CIIT i s f ul l y equi pped to cope wi th the needs of a growi ng and f ast-paced IT
i ndustry , f l exi bl e enough to adapt to the chal l enges of the f uture. The CIIT
has a mi ssi on to del i ver new i deas and products through research,
devel opment and educati on i n strategi c partnershi p wi th IT i ndustry and
organi zat i ons.

CIIT has a total of f i ve campuses across the country, two i n Isl amabad, one
each i n Wah, Abbot t abad and Lahore.

Online Test

NTS i s currentl y conducti ng onl i ne exam (2 0 %) for t he students at CIIT
Abbottabad and i t wi l l l i kel y to be adopted i n al l the campuses i n near f uture.
Students whi l e taki ng thi s ki nd of test can have a better vi si on and f rom
Student Guide 7
academi c and teachi ng poi nt of vi ew NTS can prepare di f f erent stati sti cal
anal ysi s and report s to i mprove the qual i ty of educati on, performance and
knowledge.

Admi ssi on Tests for CIIT
Nati onal Testi ng Servi ce (NTS) has been conducti ng Admi ssi on Tests f or CIIT
across al l i ts campuses si nce Jul y 2002. The General Tests are conducted f or
admi ssi ons at bot h Graduat e and Post -Graduate l e vel s cateri ng f or candi dates
f rom di f f erent educati onal backgrounds.

1.6.2 Virtual University

Vi rtual Uni versi ty (VU) i s one of the pri ori ty proj ects of the Mi ni stry of
Sci ence and Technol ogy (MoST), whi ch parti ci pates as a key contri butor i n
the soci o -economi c devel opment of the country. VU i s based on an excel l ent
tel ecommuni cati on i nf rastructure and provi des educati onal f aci l i ti es on a
nat i onwi de basi s.

NTS Conducts Admi ssi on Tests for VU
NTS conducted admi ssi on tests (NAT -IS) at vari ous Pri vate Vi rtual Centers
(PVCs) of VU i n August 02. To f aci l i tate the conduct of these Tests, NTS
provi ded Servi ces on a countrywi de basi s si mul taneousl y i n 135 centers
spread over 51 maj or ci ti es of Paki stan.

On behal f of the sati sf actory and successf ul servi ces provi ded by NTS, Vi rtual
Uni versi ty reposed i ts compl ete trust i n the authenti ci ty and credi bi l i ty of
NTS i ni ti al testi ng f or the uni versi ty whi ch they reconfi rmed by thei r pl edge
to conti nue avai l i ng the servi ces of NTS f or any f urther testi ng. We certai nl y
vi ew thi s wi th great appreci ati on and encouragement.

1.6.3 Higher Education Commission (HEC)

The Hi gher Educati on Commi ssi on has been setup to f aci l i tate the
devel opment of the uni versi ti es of Paki stan to become worl d-cl ass centers of
educati on, research and deve l opment. A cri ti cal addi ti onal chal l enge i s to
vast l y i ncrease access t o hi gher educat i on i n Paki st an so t hat our tal ented
peopl e are not deni ed avenues f or personal and prof essi onal growth. The
HEC i ntends to pl ay i ts part i n spearheadi ng the bui l di ng of a knowl edge -
based economy i n Paki stan. Peopl e are the real assets of Paki stan however it
i s onl y through educati on and research that our country can prof i t f rom thi s
great asset. A wel l -educated graduate i s the bui l di ng bl ock of a knowl edge -
based economy, and i t i s f or thi s reason that the HEC i s f ocusi ng on the i ssue
of qual i ty of hi gher educat i on.

Conducti ng Schol arshi p Tests f or HEC

In February 2003 NTS successful l y conducted Subj ect tests for vari ous areas
of study across Paki stan on behal f of the Hi gher Educati on Commi ssi on and
si nce then al l the schol arshi p tests (Subj ect/General ) of f ered by HEC are
carri ed out by NTS.

1.6.4 Pakistan Telecommunication Corporation Limited
(PTCL)

Paki stan Tel ecommuni cati on Corporati on (PTC) set sai l s f or i ts voyage of
gl ory i n December 1990, taki ng over operati ons and f uncti ons f rom Paki stan
Tel ephone and Tel egraph Department under Paki stan Tel ecommuni cati on
Corporati on Act 1991. Thi s coi nci ded wi th the Government' s competi ti ve
Student Guide 8
pol i cy, encouragi ng pri vate sector parti ci pati on and resul ti ng i n award of
l i censes for cel l ul ar, card -operated payphones, pagi ng and, l atel y, data
communi cati on servi ces.

Si nce then, PTCL has been worki ng vi gorousl y to meet the dual chal l enge of
tel ecom devel opment and soci o -economi c upl i f t of the country. Thi s i s
characteri zed by a cl ear appreci ati on of ongoi ng tel ecom scenari o wherei n
convergence of technol ogi es conti nuousl y changes the shape of the sector. A
measure of thi s understandi ng i s ref l ect ed by progressi ve measures such as
establ i shment of the company' s mobi l e and Internet subsi di ari es i n 1998.

The tel ecom sector of Paki stan has al so entered i nto a new era. The monopol y
of PTCL i s now over and the sector i s movi ng towards l i beral i zati on. Whi l e
l i beral i zati on and competi ti on are synonymous, the future scenari o al so poses
some chal l enges to the market domi nance of PTCL. The Company i s ready to
f ace thi s chal l enge and mai ntai n i ts domi nati ng posi ti on, whi l e guardi ng i ts
revenue streams i n the f ace of f orthcomi ng competi ti on.

Recruitment Test for PTCL
Bei ng an el i te cl i ent, PTCL uses the servi ces of Nati onal Testi ng Servi ce
[NTS] f or hi ring peopl e f or the organi zati on. Around thousands of candi dates
appl y f or j obs i n PTCL t hrough t he t est i ng servi ces of NTS.

1.6.5 Bolan Medical College

Bol an Medi cal Col l ege as seat of medi cal educati on i n the provi nce of
Bal ochi stan has a remarkabl e hi story.

Admission Test for BOLAN Medical College
NTS has broadened i t s servi ces and has al so conduct ed admi ssi on t est f or
Bol an Medi cal Col l ege Quetta, the conduct and out come of whi ch was t o t he
enti re sati sf acti on of al l candi dates. The students coul d tal l y thei r score wi th
the key di spl ayed af terwards.

1.6.6 Engineering University Khuzdar

NTS does not l i mi t i ts servi ces to a speci f i c educati onal group but has al so
provi ded i ts servi ces f or the conduct of Admi ssi on Tests at Engi neeri ng
Uni versi ty Khuzdar.

1.6.7 Federal Urdu University of Arts, Science and Technology,
Islamabad

The government has establ i shed a Federal Urdu Uni versi ty of Arts, Sci ences
and Technol ogy Isl amabad through an ordi nance wi th Urdu to remai n the
mai n l anguage of i nstructi on and teachi ng. The uni versi ty wi l l have i ts
pri nci pal seat at Isl amabad and wi l l have the Federal Government Urdu
Sci ence Col l ege, Karachi , and Federal Government Urd u Arts Col l ege, Karachi ,
as consti tuent i nsti tuti ons. Under the rul e, the uni versi ty wi l l have academi c,
f i nanci al and admi ni strati ve autonomy, i ncl udi ng the power to empl oy
of f i cers, teachers and other empl oyees whi l e the uni versi ty wi l l be control l ed
by a senat e, a syndi cat e and an academi c counci l .

Recruitment Test for Urdu University
Student Guide 9
The Nati onal Testi ng Servi ce [NTS] provi ded i ts servi ces to the Uni versi ty f o r
t he hi ri ng purposes of t hei r facul ty members i n ei ght subj ects thus maki ng
the sel ecti on tra nsparent .

1.6.8 Board of Intermediate and Secondary Education,
Abbottabad

Recruitment Test
Abbotta bad Educati onal Board si gned up the servi ces of NTS f or hi ri ng
empl oyees. A Computer Sci ence subj ect test was conducted f or testi ng the
academi c ski l l s of the candi dates. The assessment was l ater used to
determi ne the meri t f or f i nal sel ecti on.

1.6.9 Others
NTS now has growi ng l i nkages and asso ci ati ons wi th many educati onal and
prof essi onal organi zati ons and i nsti tuti ons, i ncl udi ng those engaged i n tal ent
search, career and educati onal promoti onal acti vi ti es, at bot h nati onal and
i nternati onal l evel s.

As a rapi dl y growi ng and credi bl e enti ty of nati onal repute, the context and
potenti al of NTS coi nci de wi th the expressed need of the Government f or the
establ i shment of a Nati onal Accredi tati on Counci l and Testi ng Servi ce.

1.7 NTS Tests

NTS general l y conducts two ki nds of tests:

General Tests
Subj ect Tests

These are conducted bot h as comput er- ai ded, paper based and onl i ne
tests, as requi red.

General Tests

NTS conducts di fferent types of General Tests f or the graduate and
undergraduate students;

The General Test compri ses Quanti tati ve, Anal yti cal Reasoni ng and Verbal
Secti ons f or assessment.

Subject Tests

Subj ect Tests are desi gned to eval uate the educati onal ski l l s i n a speci f i c
area of study. So f ar NTS has conducted the Subj ect Test i n di f f erent
subj ects:

1. Physics
2. Electronics/Engineering
3. Biology
4. Chemistry
5. Pharmacy
6. Computer Science
7. Mathematics
Student Guide 10
8. Telecommunication
9. Biochemistry
10. Management Science
11. Health Sciences
12. Environmental Science
13. Earth Science
14. Statistics
15. Veterinary Science
16. Economics
17. Agricultural Science
18. Islamic Studies
19. Pakistan Studies
20. English


1.8 NTS TEST CENTERS

NTS Head Of f i ce i s i n Isl amabad, i t conducts tests across Paki stan, f or any
Insti tute or i nsti tuti on requesti ng our servi ces, provi di ng suf f i ci ent ti me i s
provi ded f or preparati on.

1.9 SCHEDULE AND INFORMATION

There are several sources f or obtai ni ng i nf ormati on and schedul es on the NTS
Test s.

i Adverti sements through Pri nted Medi a
ii Adverti sements through El ectroni c Medi a
i i i NTS Al l i ed Insti tuti ons
iv NTS and i ts Al l i ed Insti tuti ons' Websi te

1.10 CONTACT NTS

For Tests and Test Center queries related to Score Reporting, and also for other
reasons like putting complaints or providing Feedback, you may:


Vi si t NTS at i ts websi te i .e. http: //www. nts. org. pk

Wri te to NTS at
Secretary
Nati onal Testi ng Servi ce (NTS)
Offi ce # 10 11,
Fi rst Fl oor Pl aza 2000
Pl ot # 43, Markaz I-8, Isl amabad.

Emai l Di r ect or NTS at di rectornts@ci i t. net. pk
di rectornts@nts. org. pk

Emai l NTS at ntscoord@ci i t. net. pk
support@nts. org. pk
Student Guide 11
















TECHNICAL REVIEW
Student Guide 12

2. Technical Procedures and Operations

2.1 Content Collection

As NTS is an autonomous body following a set of SOPs, trying to maintain the National and
International state of the art technology and standards. Keeping this vision in mind, NTS team
would like to share the most important and central activity that is how the contents are being
gradually increased, updated, the template upon which questions are being collected and after
carefully collecting the questions how quality is controlled. Template creation and paper
generation for individual tests is flexible and fair enough to meet the requirements of different
clients and produces results is different scenarios reliably. NTS test engine and paper
processing mechanisms are matured and error prone and we are still trying to introduce new
features.
NTS is highly conscious about the sensitive issue of contents. To date, NTS has conducted
The Subject Test in several subjects and has formulated Subject Committees for
each of those subjects. The committee members consist of subject specialists from
various institutes and organizations, providing valuable input in building up an
authentic and reliable database.

Subject Committees
These committees are made for each subject i.e. Engineering, Biology, Physics etc.,
comprising of a Chairperson being a PhD doctor and at least four members who have
a Masters Degree. Making curricula, policies, identifying other subject experts for
the Review and Question Making Committee are all responsibilities of this working
group. Other committees formulated by the Subject Committee are:

i. Review Committees
ii. Question Making Committees

NTS operations and methodology for content collection and improved quality is very secure
and protected. We put the contents in encrypted/coded format into the central database along
with other precautions to make the system safe and fool proof. NTS has huge central database
in different areas of study. The method of question collection is of two types.

Paper Base Content Collection
Online Content Collection

2.1.1 Paper Based Content Collection

For paper based content collection a template has been designed with some fields and
attributes, and information required are associated with each question. Some are narrated

Education Level of the Question
Difficulty Level of the Question (Easy, Moderate, Difficult)
Reference Information
As the question can be obtained from the book, own creativity (self-made), website etc., in
any case the additional information is required.
As an example in case a question is picked from the book the following information will be
linked with the question

Title & edition of the referenced book
Publisher of the referenced book
Topic & subtopic the questions belong to
Page No. of the Book.
Student Guide 13


Field

Subject:
Section:
Branch/Topic

Sub-Topic
Type: o Conceptual o Practical o logical
Statement:










Difficulty Level: o Easy o Moderate o Hard
Max/Min Time
Correct Answer:
Possible Choice:
(Min 4, Max 5)


A







B





C






D



E


Author: o
Website: o
Book: o
Details:
Student Guide 14
The Sample Template for Content Collection
After collecting the paper based question it is carefully entered into the central repository by
skilled Data Entry Operators. Each Question passes through different stages of Quality Control
and undergoes the detailed review to ensure the Quality and finally it becomes the part of NTS
Question Bank.

2.1.2

Online Content Collection
As it is the age if IT and internet so NTS has also the online mechanism for building question
bank we have the user friendly web based front end to collect the questions in different
disciplines. The requirement is almost the same i.e. to ensure the quality of questions.

The Snapshot of the Front End for Online Content Collection is
Give Below
Student Guide 15

2.2 Review Process

2.2.1 Review by Reviewer

As soon as question is added by the data entry operator, it becomes available to the reviewer
for review. Reviewer takes care of the typing mistakes, spelling, and formatting (font face,
font size) of the question. Questions that are marked as reviewed by reviewer become non-
editable for this operator.

2.2.2 Technical Review

Student Guide 16
After review from reviewer, it is then viewed technically by a person/committee nominated by
the peer committee relevant to that subject for the verification of the educational level,
difficulty level, quality, correct answer, time, etc., regarding each question

2.2.3 Grammatical Review

The grammatical reviewer takes care of all the questions for grammatical and language point
of view.

Note: in both cases either paper based or online content collection, all the question passes
through the same review process.

2.3 Paper Generation Process
NTS information system is capable of generating fully-formatted paper by selecting questions
randomly and/or according to the template requirement.
The Following can be specified as an input while generating the Paper

Education level of the paper
Subjects/Topics/Subtopics to be included in the paper.
No. of easy, moderate, difficult questions to be included from each subject
Selection of the questions can be controlled by including or excluding topics/subtopics.
Selection of a question can also be limited on the basis of No. of times question used

Note: By default, the system automatically selects 30% easy, 40% moderate and 30%
difficult questions to generate a paper. However this can be changed.

After the template is defined, as many sample papers can be generated as required having the
following in it;

Each sample paper can be a distinct one.
Each sample paper can be re-randomized to change the sequence of the question.

If the No. of questions selected from a topic or subtopic is not according to requirement, then
same template can be redefined on topic or subtopic level by limiting the No. of questions
from each topic or subtopic. The discussed process of paper generation is for both Paper based
and online test.
For online test the questions are picked randomly from the template constructed from the
question bank and the test engine manipulates the sequence of questions for every candidate

Test Engine

For online test NTS has developed a very robust, user friendly, stable and secured Test Engine
for online test. It has very distinguished features like on spot registration, shuffling the
questions randomly for each candidate, result display immediately after the completion of test,
tracking of user session, flexible enough to cope with the requirement of a particular exam,
continuous time monitoring, pass box for placing questions temporarily, pictures handling in
the question statement and question choice, can accommodate multiple true answers, manage
scenario based questions, and toggle freely between Question Box and Pass Box.








Student Guide 17
Glimpse of the web based online test application is given below User Registration
Process



Instructions for the Test

Student Guide 18
Question Box

Pass Box


Student Guide 19
Paper Checking and Result Preparation

NTS has very reliable, error free, efficient and transparent system for paper checking and
result preparation. It is fast enough and flexible to cope with the requirements of any client.
As a reference we are quoting the Bolan Medical College, Quetta Entry Test we meet the
requirement of delivering the result after few hours on the same day the test was conducted.
The result was handed over along with merit lists in their required format and the notable
feature was display of answer key on the spot form where any candidate could calculate
his/her result because the carbon copy of answer sheet was given to the candidates after they
finished their test. Despite all these facts there was not a single error in the result which
shows the competency, reliability, transparency of NTS Paper Checking and Result Preparation
System. We are continuously improving our System day by day by introducing new features
and customization. A team of skilled professionals are dedicatedly working on the task.


































Student Guide 20
Sample of an Answer Sheet is shown.


Student Guide 21







PATTERNS AND
PRACTICE
Student Guide 22

2 Quantitative Ability

The Quanti tati ve secti on measures your basi c mathemati cal ski l l s,
understandi ng of el ementary mathemati cal concepts, and the abi l i ty to reason
quanti tati vel y and solve probl ems i n a quanti tati ve setti ng. There i s a
bal ance of questi ons requi ri ng basi c knowl edge of ari thmeti c, al gebra,
geometry, and data anal ysi s. These are essenti al content areas usual l y
studi ed at the hi gh school l evel .

The questi ons i n t he quanti ta ti ve secti on can al so be f rom

Di screte Quanti tati ve Questi on
Quanti tati ve Compari son Questi on
Data Interpretati on Questi on etc.

The di stri buti on i n thi s gui de i s onl y to f aci l i tate the candi dates. Thi s
di stri buti on i s not a part of test templ ate, so, a test may contai n al l the
quest ions of one f ormat or may have a random number of questi ons of
di f f erent f ormats.

Thi s chapter i s di vi ded i nto 4 maj or secti ons. The fi rst di scusses the syl l abus
/contents i n each secti on of the test respecti vel y and the re mai ni ng three
secti ons address the questi on f ormat, gui de l i nes to attempt the questi ons i n
each f ormat and some exampl e quest i ons.

2.1 General Mathematics Review

2.1.1 Arithmetic

The fol l owi ng are some key poi nts, whi ch are phrased here to ref resh your
knowl edge of basi c ari thmeti c pri nci pl es.

Basic arithmetic

For any number a, exactl y one of the f ol l owi ng i s true:
o a i s negati ve
o a i s zero
o a i s posi ti ve

The onl y number that i s equal to i ts opposi te i s 0 (e.g.
0 a aonl yi f a = =
)


If 0 i s mul ti pl i ed to any other number, i t wi l l make i t zero (
0 0 a =
).

Product or quoti ent of two numbers of the same si gn are al ways posi ti ve
and of a di fferent si gn are al ways negati ve. E.g. if a posi ti ve number i s
mul ti pl i ed to a negati ve number the resul t wi l l be negati ve and i f a
negati ve number i s di vi ded by anot her negati ve number the resul t wi l l be
posi ti ve.

See the f ol l owi ng tabl es f or al l combi nati ons.


Student Guide 23
+ or + = +
+ or =
or + =
or = +
The sum of two posi ti ve numbers i s al ways posi ti ve.

The sum of two negati ve numbers i s al ways negati ve.

Subtracti ng a number from anot her i s the same as addi ng i ts opposi te

( ) a b a b = +

The reci procal of a number a i s
a
1


The product of a number and i ts reci procal i s al ways one

1
1
=
a
a


Di vi di ng by a number i s the same as mul ti pl yi ng by i ts reci procal

b
a b a
1
=


Every i nteger has a f i ni te set of factors (di vi sors) and an i nf i ni te set of
multipliers.

If a and b are two i ntegers,t he fol l owi ng four terms are synonyms
o a i s a divisor of b
o a i s a factor of b
o b i s a divisible b y a
o b i s a multiple of a

They al l mean that when a i s di vi ded by b t here i s no remai nder.

Posi ti ve i ntegers, other than 1, have at l e ast two posi ti ve f actors.

Posi ti ve i ntegers, other than 1, whi ch have exactl y two f actors, are known
as prime numbers.

Every i nteger greater than 1 that i s not a pri me can be wri tten as a product
of pri mes.
To f i nd the pri me f actori zati on of an i nteger, f i nd any two f actors of that
number, i f both are pri mes, you are done; i f not, conti nue f actori zati on
unti l each f actor i s a pri me.
E. g. to f i nd the pri me f actori zati on of 48, two f actors are 8 and 6. Both of
them are not pri me numbers, so conti nue to f actor them.
Factors of 8 are 4 and 2, and of 4 are 2 and 2 (2 2 2).
Factors of 6 are 3 and 2 (3 2).
So the number 48 can be wri tten as 2 2 2 2 3 .

The L east Common Mul ti pl e (LCM) of two i ntegers a and b i s the smal l est
i nteger whi ch i s di vi si b l e by both a and b, e.g. the LCM of 6 and 9 i s 18.

Student Guide 24
The Greatest Common Di vi sor (GCD) of two i ntegers a and b i s the l argest
i nteger whi ch di vi des both a and b, e. g. the GCD of 6 and 9 i s 3.

The product of GCD and LCM of two i ntegers i s equal to the products of
numbers i tsel f. E.g. 6 9 = 54
3 18 = 54 (where 3 i s GCD and 18 i s LCM of 6 and 9).

Even numbers are al l the mul ti pl es of 2 e.g. ( , 4, 2, 0, 2, 4, )

Odd numbers are al l i ntegers not di vi si bl e by 2 ( , 5, 3, 1, 1, 3, 5, )

If two i ntegers are both even or both odd, thei r sum and di f f erence are
even.

If one i nteger i s even and t he other i s odd, thei r sum and di f f erence are
odd.

The product of two i ntegers i s even unl ess both of them are odd.

When an equati on i nvol ves more than one operati on, i t i s i mportant to
carry them out i n t he correct order. The correct order i s Parentheses,
Exponents, Mul ti pl i cati on and Di vi si on, Addi ti on and Subtracti on, or j ust
the f i rst l etters PEMDAS to remember the proper order.

Exponents and Roots

Repeated addi ti on of the same number i s i ndi cated by mul ti pl i cati on:
17 + 17 + 17 + 17 + 17 = 5 17

Repeated mul ti pl i cati on of the same number i s i ndi cated by an exponent:
17 17 17 17 17 = 17
5


In the expressi on 17
5
, 17 i s cal l ed base and 5 i s the exponent .

For any number b: b
1
= b and b
n
= b b b, wher e b i s used n ti mes
as f actor.

For any numbers b and c and posi ti ve i ntegers m and n:
o
n m n m
b b b
+
=
o
n m
n
m
b
b
b

=
o
mn n m
b b = ) (
o
m m m
bc c b ) ( =

If a i s negati ve, a
n
i s posi ti ve i f n i s even, and negati ve i f n i s odd.

There are two numbers that sati sf y the equati on
9
2
= x
:
3 = x
and
3 = x
.
The posi ti ve one, 3, i s cal l ed the (pri nci pal ) square root of 9 and i s denoted
by symbol
9
. Cl earl y, each perf ect square has a square root:
0 0 =
,
3 9 =
,
6 36 =
,
13 169 =
,
25 225 =
etc.

Student Guide 25
For any posi ti ve number a there i s a posi ti ve number b that sati sf i es the
equati on
b a =
.
For any posi ti ve i nteger,
a a a a = =
2
) (
.

For any posi ti ve numbers a and b:
o
b a ab =


and
b
a
b
a
=


o
7 4 3 16 9 16 9 25 5 = + = + + = =
+ +
as
b a b a


Al though i t i s al ways true that
a a =
2
) (
,
a a =
2
i s true onl y i f a i s
posi ti ve as
5 5 25 ) 5 (
2
= =

For any number a,
2
n
n
a a =
.


For any number a, b and c:


ac ab c b a + = + ) (

ac ab c b a = ) (


and i f
0 a



a
c
a
b
a
c b
+ =
+ ) (

a
c
a
b
a
c b
=
) (


Inequal i ti es

For any number a and b, exactl y one of the f ol l owi ng i s true:

b a >
or
b a =
or
b a <
.

For any number a and b,
b a >
means t hat
b a
i s posi ti ve.

For any number a and b,
b a <
means t hat
b a
i s negati ve.

For any number a and b,
b a =
means t hat
b a
i s zero.

The symbol

means greater than or equal to and the symbol

means l ess
than or equal to. E. g. the statement
5 x
means that x can be 5 or any
number greater than 5.
The st atement
5 2 < < x
i s an abbrevi ati on of
x < 2
and
5 < x
.

Addi ng or subtracti ng a number to an i nequal i ty preserves i t.

If
b a <
, then
c b c a + < +
and
c b c a <
.

Student Guide 26
e.g.
10 6 10 5 10 6 10 5 6 5 < + < + < and



Addi ng i nequal i ti es i n same di recti on preserves i t:
If
d c and b a < <
, then
d b c a + < +
.

Mul ti pl yi ng or di vi di ng an i nequal i ty by a posi ti ve number preserves i t. If
b a <
and
c
i s a posi ti ve number, then
c b c a <
and
c
b
c
a
<
.

Mul ti pl yi ng or di vi di ng an i nequal i ty by a negati ve number reverses i t. If
b a <
and
c
i s a negati ve number, then
c b c a >
and
c
b
c
a
>
.

If si des of an i nequal i ty are both posi ti ve and both negati ve, taki ng the
reci procal reverses the i nequal i ty.

If
0 1 x < <
and
a
i s posi ti ve, then
xa a <
.

If
0 1 x < <
and
m and n
are i ntegers wi th
m n >
, then
m n
x x x < <
.


If
0 1 x < <
, then
x x >
.


If
0 1 x < <
, then
1
x
x
>
and
1
1
x
>


Properti es of Zero

0 i s the onl y number that i s nei ther negati ve nor posi ti ve.

0 i s smal l er than every posi ti ve number and greater than every negati ve
number.

0 i s an even i nteger.

0 i s a mul ti pl e of every i nteger.

For every number
a
:
0 0 a a a n d a a + = =
.

For every number
a
:
0 0 a =
.

For every posi ti ve i nteger
n
:
0 0
n
=
.

For every number
a
(i ncl udi ng 0):
0
0
a
a and

are undef i ned symbol s.

For every number a (other than 0):
0
0 0 a
a
= =
.
Student Guide 27
0 i s the onl y number that i s equal to i ts o pposi te:
0 0 =
.

If t he product of two or more numbers i s 0, at l east one of them i s 0.

Properti es of One
For any number
a
:
1 a a =
and
1
a
a =
.
For any number
n
:
1 1
n
=
.

1 i s the di vi sor of every i nteger.

1 i s the smal l est posi ti ve i nteger.

1 i s an odd i nteger.

1 i s not a pri me.

Fractions and Decimals

When a whol e i s di vi ded i nto n equal parts, each part i s cal l ed one nth of
the whol e, wri tten

1
n
. For exampl e, i f a pi zza i s cut (di vi ded) i nto 8 equal
sl i ces, each sl i ce i s one ei ghth (
1
8
) of the pi zza; a day i s di vi ded i nto 24
equal hours, so an hour i s one twenty-fourth
1
( )
24

of a day and an i nch i s
one twel fth (
1
12
) of a f oot. If one works f or 8 hours a day, he works ei ght
twenty-fourth (
8
24
) of a day. If a hockey sti ck i s 40 i nches l ong, i t
measures f orty twel f ths
40
( )
12
of a f oot .
The numbers such as
1
8
,
1
24
,
8
24
and
40
12
, i n whi ch one i nteger i s wri tten
over the second i nteger, are call ed fractions. The center l i ne i s cal l ed the
fracti on bar. The number above the bar i s cal l ed the numerator , and the
number bel ow the bar i s cal l ed denominator .

The denomi nator of a f racti on can never be 0.
A f racti on, such as
1
24
, i n whi ch the denomi nator i s greater than
numerator, i s known as a proper fraction. Its val ue i s l ess than one.
A f racti on, such as
40
12
, i n whi ch the denomi nator i s l ess than numerator, i s
known as an improper fraction. Its val ue i s greater than one.
Student Guide 28
A fracti on, such as,
12
12
i n whi ch the denomi nator i s equal to the numerator,
i s al so known as an improper fraction. But, Its val ue i s one.
Every f racti on can be expressed i n deci mal f orm (or as a whol e number) by
di vi di ng the number by the denomi nator.
3 3 8 48 100
0.3, 0.75, 1, 3, 12.5
10 4 8 16 8
= = = = =

Unl i ke the exampl es above, when most fracti ons are converted to deci mal s,
the di vi si on does not termi nate, af ter 2 or 3 or 4 deci mal pl aces; rather i t
goes on f orever wi th some set of di gi ts repeati ng it .
2 3 5 17
0.66666..., 0.272727..., 0.416666..., 1.133333...
3 11 12 15
= = = =

Student Guide 29

To compare two deci mal s, f ol l ow these rul es:

o Whi chever number has the greater number to the l eft of the deci mal
poi nt i s greater: si nce 11 > 9, 11.0001 > 9.8965 and si nce 1 > 0,
1.234 > .8. (Recal l that i f a deci mal i s wri tten wi thout a number on
l eft of deci mal poi nt, you may assume that a 0 i s there, so, .8 =
0.8).
o If the numbers to the l eft of the deci mal poi nt are equal , proceed as
f ol l ows:

If the numbers do not have the same number of di gi ts to the right
of the deci mal poi nt, add zeroes to the end of the shorter one to
make them equal i n l ength.
Now compare the numbers i gnori ng the deci mal poi nt.
For exampl e, to compare 1.83 and 1.823, add a 0 to the end of
1.83 f ormi ng 1.830. Now compare them, thi nki ng of them as whol e
numbers wi thout deci mal poi nt: si nce 1830 > 1823, then 1.830
>1.823.

There are two ways to compare f racti ons:
o Convert them to deci mal s by di vi di ng, and use the method al ready
descri bed to compare these deci mal s. For exampl e to compare
2
5

and
1
4
, convert them to deci mal s.
2
0.4
5
=
and
1
0.25
4
=
. Now, as 0.4
> 0.25,
2
5
>
1
4
.
o Cross mul ti pl y the fracti ons. For exampl e to compare
2
5
and
1
4
,
cross mul ti pl y:
2
5

1
4

Since

2 4 1 5 >
,

t hen

2
5
>
1
4
.
Whi l e compari ng the fracti ons, i f they have same the denomi nators, the
f racti on wi th the l arger numerator i s greater. For exampl e
3 2
5 5
>
.

If the fractions have the same numerator, the fraction with the
smaller denominator is greater. For example
3 3
5 10
>
.
Two f racti ons are cal l ed equivalent fractions i f both of them have same
deci mal val ue.
For exampl e,
1 5
2 10
=
as both of these are equal to 0. 5.
Another way to check the equi val ence of two f racti ons i s to cross-mul ti pl y.
If both of the products are same, the fracti ons are equi val ent. For Exampl e,
to compare
2
5
wi th
6
15
, cross-mul ti pl y. Si nce
2 15 6 5 =
, both of the
fractions are equi val ent.
Every fracti on can be reduced to l owest terms by di vi di ng the numerator
and denomi nator by thei r greatest common di vi sor (GCD). If the GCD i s 1,
Student Guide 30
the f racti on i s al ready i n l owest terms. For exampl e to reduce
10
15
, di vi de
both numerator and denomi nator by 5 (whi ch i s GCD of 10 and 15). Thi s
wi l l reduce the f racti on to
2
3
.
To mul ti pl y two fracti ons, mul ti pl y thei r numerators and mul ti pl y thei r
denomi nators. For exampl e
3 4 3 4 12
5 7 5 7 35

= =

.
To mul ti pl y a number to a fracti on, wri te that number as a fracti on whose
denomi nator i s 1. For exampl e
3 3 7 3 7 21
7
5 5 1 5 1 5

= = =

.
When a probl em requi res you to fi nd the fracti on of a number, mul ti pl y that
fracti on wi th t he number. For exampl e, to fi nd two fi fth (
2
5
) of 200,
mul ti pl y:
2 2 200 400
200
5 5 1
= =
80
80
5
=
/
.
The reci procal of a fracti on
a
b
i s another f racti on
b
a
si nce
1
a b
b a
=

To di vi de one fracti on by the other fracti on, mul ti pl y the reci procal of
di vi sor wi th the di vi dend. For exampl e,
22 11 22
7 7
=
2
7
7 11

2
2
1
= =
.
To add or subtract t he fracti ons wi th same denomi nator, add or subtract
numerators and keep the denomi nator. For
exampl e
4 1 5 4 1 3
9 9 9 9 9 9
and + = =
.

Percents

The word percent means hundredth. We use the symbol % to express the
wor d per cent. For exampl e 15 percent means 15 hundredths and can be
wri tten wi th a % symbol , as a f racti on, or as a deci mal .
20
20% 0.20
100
= =
.
To convert a deci mal to a percent, move the deci mal poi nt two pl aces to the
ri ght, addi ng 0s i s necessary, and add the percent symbol (%).
For exampl e, 0.375 = 37.5% 0.3 = 30% 1.25 = 125% 10=1000%

To convert a f racti on to a percent, f i rst convert that f racti on to deci mal ,
than use the method stated above to convert i t to a percent.

To convert a percent to a deci mal , move the deci mal poi nt two pl aces to the
l ef t and remove the % symbol . Add 0s i f necessary.
For exampl e, 25% = 0.25 1% =0.01 100% = 1

You shoul d be f ami l i ar wi th the f ol l owi ng basi c conversi ons:
1 5
0.50 50%
2 10
= = =

1 2
0.20 20%
5 10
= = =

Student Guide 31
1
0.25 25%
4
= =

3
0.75 75%
4
= =


For any posi ti ve i nteger
a
:
% a
of 100 i s
a
.

For any posi ti ve numbers
a
and
b
:
% % a of b b of a =


The percent change i n the quanti ty i s
100%
actual change
original amount

. For exampl e:
If the pri ce of a l amp goes from Rs . 80 to Rs. 100, the actual i ncrease i s
Rs.20, and the percent i ncrease i s
20 1
100% 100% 25%
80 4
= =
.
If
a b <
, the percent i ncrease i n goi ng f rom
a
t o
b
i s al ways greater than
perce nt decrease i n goi ng from
b
t o
a
.

To i ncrease a number by % k , mul ti pl y i t by 1 % k + , and to decrease a
number by % k , mul ti pl y i t by 1 % k . For exampl e, the val ue of an
i nvestment of Rs. 20,000 af ter 25% i ncrease i s
20,000 (1 25%) 20,000 (1.25) 25,000 + = = .

If a number i s the resul t of i ncreasi ng another number by % k , to f i nd the
ori gi nal number di vi de i t by
1 % k +
, and i f a number i s the resul t of
decreasi ng another number by % k , to f i nd the ori gi nal number, di vi de i t by
1 % k .
For exampl e, The government announced an 20%i ncrease i n sal ari es. If
after the i ncrement, The sal ary of a parti cul ar empl oyee i s Rs. 18, 000,
what was the ori gi nal sal ary?
Ori gi nal sal ary (i n Rs.) =
18,000 18,000
15,000
1 1 20% 1.20
current salary
percent increase
= = =
+ +


Rati os and Proporti ons

A rati o i s a fracti on that compares two quanti ti es that are measured i n t he
same uni ts. The fi rst quanti ty i s the numerator and the second quanti ty i s
denomi nator. For exampl e, i f there are 16 boys and 4 gi rl s, we say that the
rati o of the number of boys to the number of gi rl s on the team i s 16 t o 4,
or
16
4
. Thi s i s often wri tten as 16:4. Si nce a rati o i s just a fracti on, i t can
be reduced or converted to a deci mal or a percent. The Fol l owi ng are
di fferent ways to express the same rati o:

16 4 to
,
16: 4
,
16
4
,
4
1
,
0.25
,
25%

If a set of obj ects i s di vi ded i nto two groups i n the rati on
: a b
, then the
fi rst group contai ns
a
a b +
of the total objects and si mi l arl y the second group
Student Guide 32
contai ns
b
a b +
of the total number of obj ects. Thi s rul e appl i es to extended
rati os, as wel l . If a set i s di vi ded i nto three groups i n the rati o
: : a b c
, then
the f i rst group contai ns
a
a b c + +
of the total obj ects, and so on.

A proporti on i s an equati on that states that two rati os are equi val ent. Si nce
rati os are j ust fracti ons, any equati on such as
4 10
6 15
=
i n whi ch each si de i s
a si ngl e f racti on i s proporti on. Thi s proporti on states that 4 rel ates to 6 i n
same rati o as 10 rel ates to 15.
For each proporti on of the form
a c
b d
=
,
ad bc =
. This property can be used
to sol ve proporti ons for unknowns (vari abl es). For exampl e: If 3 oranges
cost Rs.5, how many oranges can you buy for Rs.100. To sol ve thi s
probl em we have to set up a proporti on. If the number of oranges for
Rs.100 i s
x
, then:
3 3 100
3 100 5 60
5 100 5
x
x x x

= = = =


Averages

The average of a set of n numbers i s the sum of those numbers di vi ded by
n.

sumof nnumbers
average
n
=

or si mpl y

Sum
A
n
=

the techni cal name for t hese ki nd of averages i s Ari thmeti c Mean.

If you know the average of n numbers, mul ti pl y that average wi th n to get
the sum of numbers.

If al l the numbers i n a set are the same, then that number i s the average.


Assume that the average of a set of numbers i s A. If a new number x is
added to that set, the new average wi l l be;

o Greater if x is greater than the existing average
o Smaller if x is smaller than the existing average
o Unchanged if x is equal to the existing average

Ari thmeti c sequence i s an ordered set of numbers, such that, the di fference
between two consecuti ve numbers i s t he same.

If there i s an ari thmeti c sequence of n terms, then t he average cal cul ati on
can be made si mpl e by usi ng these rul es.
o The average of the terms i n that sequence wi l l be the mi ddl e term,
i f n i s odd.
o If n i s even, the average wi l l be the average of two mi ddl e terms.

2.1.2 Algebra

Student Guide 33
Pol ynomi al s


A monomi al i s any number or vari abl e or product of numbers and vari abl es.
For exampl e
3 2 3 2
3, 4, , , 3 , 2 , 5 ,1.5 , x y x xyz x xy a b
are al l monomi al s.

The number that appears i n front of a vari abl e i n a monomi al i s cal l ed the
coeffi ci ent. The coeffi ci ent of
3
5x
i s 5. If there i s no number, the
coef f i ci ent i s ei ther 1 or 1, because
x
means
1x
and
x
means
1x
.

A pol ynomi al i s a monomi al or the sum of two or more monomi al s. Each
monomi al that makes up the pol ynomi al i s cal l ed a term of that pol ynomi al .

If a pol ynomi al has onl y one term i t i s a si mpl e monomi al , i f i t has two
terms, i t i s known as bi nomi al and i f i t has three terms, i t i s cal l ed
tri nomi al .

Two terms are cal l ed l i ke terms i f they di ffer onl y i n thei r coeffi ci ents.
3
5x

and
3
2x
are l i ke terms, whereas,
3
5x
and
2
5x
are not.

If l i ke terms are i nvol ved i n a pol ynomi al , they can be combi ned, by addi ng
thei r coeffi ci ents, to make that pol ynomi al si mpl er. The pol ynomi al
2 2
3 4 5 2 7 x x x x + +
is equi val ent to the pol ynomi al
2
9 7 x x +
.
Al l l aws of ari thmeti c are al so appl i cabl e to pol ynomi al s. Most i mportant of
them i s PEMDAS.

Pol ynomi al s can be added, subtracted, mul ti pl i ed or di vi ded.

To add two pol ynomi al s, put a pl us si gn between them, erase the
parentheses, and combi ne l i ke terms.


Example:
What i s the sum of
2
5 10 7 x x +
and
2
3 4 2 x x +
?

Solution:
2 2
2 2
2
(5 10 7) (3 4 2)
5 10 7 3 4 2
8 6 5
x x x x
x x x x
x x
+ + +
= + + +
= +


To subtract two pol ynomi al s, reverse the si gns of subtrahend, and add two
pol ynomi al s as done bef ore.

Example:
Subtract
2
3 4 2 x x +
from
2
5 10 7 x x +


Student Guide 34
Solution:
2 2
2 2
2 2
2
(5 10 7) (3 4 2)
(5 10 7) ( 3 4 2)
5 10 7 3 4 2
2 14 9
x x x x
x x x x
x x x x
x x
+ +
= + + +
= + +
= +


To mul ti pl y monomi al s, fi rst mul ti pl y thei r coeffi ci ents, and then mul ti pl y
thei r vari abl es by addi ng the exponents.

Example:
What i s the product of
2
3x yz
from
2 2
2x y
?

Solution:
2 2 2
2 2 2
4 3
(3 )( 2 )
(3 2)( )( )( )
6
x yz x y
x x y y z
x y z

=
=


To mul ti pl y a monomi al by a pol ynomi al , j ust mul ti pl y each term of the
pol ynomi al by the monomi al .






Example:
What i s the product of
3x
from
2 2
3 6 2 x xy +
?

Solution:
2 2
2 2
3 2 2
(3 )(3 6 2)
(3 3 ) (3 6 ) (3 2)
9 18 6
x x xy
x x x xy x
x x y x
+
= +
= +


To mul ti pl y two bi nomi al s, mul ti pl y each term of fi rst bi nomi al by each term
of second bi nomi al , then add the resul ts.

Example:
What i s the product of
3x y +
from
2 2
3 6 x xy
?

Solution:
2 2
2 2 2 2
2 2 2 2 3
2 2 2 2 3
(3 )(3 6 )
(3 3 ) (3 ( 6 )) ( 3 ) ( ( 6 ))
(9 ) ( 18 ) (3 ) ( 6 )
9 18 3 6
x y x xy
x x x xy y x y xy
x x y x y xy
x x y x y xy
+
= + + +
= + + +
= +


The three most i mportant bi nomi al products are :
Student Guide 35
o
2 2 2 2
( )( ) x y x y x xy xy y x y + = + + = +

o
2 2 2 2
( )( ) 2 x y x y x xy xy y x xy y + + = + + + = + +

o
2 2 2 2
( )( ) 2 x y x y x xy xy y x xy y = + = +

Memori zi ng these can save a l ot of cal cul ati on ti me duri ng t he t est .

To di vi de a pol ynomi al by a monomi al , di vi de each term of the pol ynomi al
by the monomi al .

Example:
What i s the quoti ent i f
2 3
32 12 x y xy z +
i s di vi ded by
8xy
?

Solution:
2 3 2 3
2
32 12 32 12 3
4 (by reducing theterms)
8 8 8 2
x y xy z x y xy z
x y z
xy xy xy
+
= + = +

Student Guide 36
Sol vi ng Equati ons and Inequal i ti es

The basi c pri nci pl e i n sol vi ng equati ons and i nequal i ti es i s that you can
mani pul ate them i n any way as l ong as you do the same thing to both
si des. For example you may add a number to both si des, or you may
di vi de or mul ti pl y both si des wi th same number etc.
By usi ng the f ol l owi ng si x-step method, you can sol ve most of the
equati ons and i nequal i ti es. The method i s expl ai ned wi th the hel p of an
exampl e.
Example:
if
1
3( 2) 2( 1) 1
2
x x x + = + +
, what i s the val ue of
x
?

Solution:
Step What to do Example
1 Get ri d of fracti ons and
deci mal s by mul ti pl yi ng
both si des by the LCD.
Mul ti pl y each si de by 2 to get:
6( 2) 4( 1) 2 x x x + = + +

2 Get ri d of al l parentheses
by sol vi ng them.
6 12 4 4 2 x x x + = + +

3 Combi ne l i ke terms on
each si de.
7 12 4 6 x x = +

4 By addi ng and subtracti ng
get al l the vari abl es on
one si de (mostl y l eft).
Subtract
4x
from each si de to get:
3 12 6 x =

5 By addi ng or subtracti ng
get al l pl ai n numbers on
the other si de.
Add 12 to each si de to get:
3 18 x =

6 Di vi de both si des by the
coeffi ci ent of the vari abl e.
(If you are deal i ng wi th
an i nequal i ty and you
di vi de wi th a negati ve
number, remember to
reverse the i nequal i ty.)
Di vi de both si des by 3 to get:
6 x =


When you have to sol ve one vari abl e and the equati on/i nequal i ty i nvol ve
more t han one vari abl e, t reat al l other vari abl es as pl ai n numbers and
appl y the si x-step method.
Example:
if
3 a b c =
, what i s the val ue of
b
i n terms of
a
and
c
?
Solution:
Step What to do Example
1 There are no fracti ons and
deci mal s.

2 There are no parentheses.
3 There are no l i ke terms.
4 By addi ng and subtracti ng get
al l the vari abl es on one si de.
Remember there i s onl y one
vari abl e
b
, whi ch i s on one
si de onl y.
5 By addi ng or subtracti ng get
al l pl ai n numbers on the
other si de.
Remember we are
consi deri ng
a
and
c
as
pl ai n number. Add
c
to
each si de to get:
3 a c b + =

Student Guide 37
6 Di vi de both si des by the
coef f i ci ent of the vari abl e.
Di vi de both si des by 3 to
get:
3
a c
b
+
=

It i s not necessary to f ol l ow these steps i n the order speci f i ed. Some ti mes
i t makes the probl em much easi er, i f you change the order of these steps.

Example:
If
4 11 x =
, what i s the val ue of
x
-8?

Solution:
Goi ng i mmedi atel y to step 5, add 4 to each si de to get:
15 x =
. Now
subtract 8 f rom both si des to get:
8 7 x =
.

Doi ng the same thi ng on each si de of an equati on does not mean doi ng the
same thi ng to each term of the equati on. Thi s i s very i mportant if you are
doi ng di vi si ons, or deal i ng wi th exponents and roots.

Example:
If
0 a >
and
2 2 2
a b c + =
, what i s the val ue of
a
i n terms of
b
and
c
.

Solution:
2 2 2 2 2 2
a b c a c b + = =
. Now you can t take a square root of each term
t o get
a c b =
. You must take the square root of each si de:
2 2 2 2 2
a c b a c b = =


Another type of equati on i s that i n whi ch the vari abl e appears i n exponent.
These equati ons are basi cal l y sol ved by i ncepti on.
Exampl e:
If
3
2 32
x+
=
, what i s the val ue of
2
3
x+
?
Sol uti on:
3 3 5
2 32 2 2 3 5 2
x x
x x
+ +
= = + = =
.
Now as
2 x =
, you can get
2 4
2 2 4 3 3 81
x
x x
+
= + = = =


A system of equati ons i s a set of two or more equati ons havi ng two or more
vari abl es. To sol ve such equati ons, you must f i nd the val ue of each
vari abl e that wi l l make each equati on true.

To sol ve a system of equati ons, add or subtract them to get a thi rd
equati on. If there are more than two equati ons you can j ust add them.

Example:
If
10 x y + =
and
10 x y =
what i s the val ue of
y
?


Solution:
Student Guide 38
Add two equati ons:
10
2
2 12 6
x y
x y
x x
+ =
=
= =

Now repl aci ng
x
wi th 6 i n t he f i rst equat ion:
6 10 4 y y + = =


If you know the val ue of one vari abl e i n a system of two equati ons, you can
use thi s val ue to get the val ue of the other vari abl e. As i t i s done i n the
previ ous questi on.


Word problems
To sol ve word probl ems, fi rst translate the probl em from Engl i sh to
Al gebra. Whi l e transl ati ng, use vari abl es to represent unknowns. Once
transl ated, i t i s easy to sol ve them usi ng the techni ques you have l earned
i n previ ous secti ons.
Fol l owi ng Engl i sh to Al gebra di cti onary wi l l be hel pful in transl ati ng word
probl ems to al gebrai c expressi ons.

English words
Mathematical
meani ng
Symbol
Is, was, wi l l be, had, has, wi l l
have, i s equal to, i s the same as
Equal s =

Pl us, more than, sum, i ncreased
by, added to, exceeds, recei ved,
got, ol der than, farther than,
greater than
Addi ti on +

Mi nus, fewer, l ess than,
di fference, decreased by,
subtracted from, younger than,
gave, l ost
Subtracti on

Ti mes, of, product, mul ti pl i ed by Mul ti pl i cati on

Di vi ded by, quoti ent, per, f or Di vi si on
or
a
b


More than, greater than Inequal i ty >

At l east Inequal i ty

Fewer than, l ess than Inequality <

At most Inequal i ty

What, how many, etc. Unknown quanti ty x

(Some vari abl e)

Exampl es:
o The sum of 5 and some number is 13. 5 13 x + =
o Javed was two years younger than Saleem. 2 J S =
o Bilal has at most Rs.10,000. 10000 B
o The product of 2 and a number exceeds that
number by 5 (is 5 more than that number). 2 5 N N = +

In word probl ems, you must be sure about what you are answeri ng. Do not
answer the wrong questi on.

In probl ems i nvol vi ng ages, remember that years ago means you need to
subtract, and years f rom now means you need to add.

Student Guide 39
Di stance probl ems al l depend on three vari ati ons of the same formul a:
o
distance speed time =

o
distance
speed
time
=

o
distance
time
speed
=


Example:
How much l onger, i n seconds, i s requi red to dri ve 1 mi l e at 40 mi l es per
hour than at 60 mi l es per hour?

Solution:
The ti me to dri ve at 40 mi l es p er hour can be cal cul ated as
1 time
=
1
40
hours =
1
40
2
60
3
mi nutes =
3
2
60
30
seconds =
90
seconds

The ti me to dri ve at 60 mi l es per hour can be cal cul ated as
2 time
=
1
60
hours =
1
60
60
mi nutes =
1 60
seconds =
60
seconds

1 2 90 60 30 difference time time = = = seconds.

2.1.3 Geometry

Li nes and Angles

An angl e i s f ormed at the i ntersecti on of two l i ne segments, rays or l i nes.
The poi nt of i ntersecti on i s cal l ed the vertex. Angl es are measured i n
degrees.

Angl es are cl assi f i ed accordi ng to thei r degree measures.
An acute angl e measures l ess than
90

A right angl e measures
90

An obtuse angl e measures more than
90
but l ess t han
180

A strai ght angl e measures
180


If t wo or more angl es combi ne together to form a strai ght angl e, the sum
of thei r measures i s
180
.
180 a b c d + + + =



Student Guide 40
The sum of al l the measures of al l the angl es around a poi nt i s
360

360 a b c d e + + + + =


When two l i nes i ntersect, f our angl es are f ormed, two angl es i n each pai r of
opposi te angl es are cal l ed vertical angles. Verti cal angl es, f ormed by the
i ntersecti on of two l i nes, have equal measures.
a c =
and
b d =


If one of the angl es formed by the i ntersecti on of two l i nes i s a ri ght angl e,
then al l f our angl es are ri ght angl es. Such l i nes are cal l ed perpendicular
l i nes

90 a b c = = =


In the f i gure bel ow a l i ne l di vi des the angl e i n two equal parts. Thi s l i ne i s
sai d to bisect t he angl e. The other l i ne k bi sects another l i ne i nto two
equal parts. Thi s l i ne i s sai d to bi sect a l i ne.

Two l i nes are sai d to be parallel, i f they never i ntersect each other.
However, i f a thi rd l i ne, cal l ed a transversal , i ntersects a pai r of paral l el
l i nes, ei ght angl es are f ormed. And the rel ati onshi p among theses angl es i s
shown i n the fol l owi ng di agram.

- Al l f our acute angl es are equal
a c e g = = =

- Al l four obtuse angl es are equal
b d f h = = =

- The sum of any pai r of acute and
obtuse angl e i s
180
, e.g.
180 , 180 , 180 a d d e b g + = + = + =
etc.

Triangles
Student Guide 41
In any tri angl e, the sum of the measures of the three angl es i s
180
.
180 x y z + + =


In any tri angl e:
o The l ongest si de of tri angl e i s opposi te the l argest angl e.
o The shortest si de i s opposi te the smal l est angl e.
o Si des wi th the same l ength are opposi te the angl es wi th the same
measure.

Tri angl es are cl assifi ed i nto three di fferent ki nds wi th respect to the
l engt hs of si des.
o Scalene: i n whi ch al l three si des are of di f f erent l engths.
o Isosceles: i n whi ch two of the si des of tri angl e are equal i n
l ength, the thi rd i s di f f erent.
o Equilateral : i n whi ch al l three si des are equal i n l ength.

Tri angl es are al so cl assif i ed wi th respect to the angl es.
o Acute triangle: i n whi ch al l three angl es are acute.
o Obtuse triangle: i n whi ch one angl e i s obtuse and two are acute.
o Right triangle: Thi s has one ri ght and two acute ang l es.

In a ri ght tri angl e, the opposi te to the ri ght angl e i s known as hypotenuse
and i s the l ongest si de. The other two si des are cal l ed l egs.

In any ri ght tri angl e, the sum of the measures of the two acute angl es i s
90
.


By Pythagorean Theorem, the sum of squares of the l engths of l egs of a
ri ght tri angl e i s al ways equal to the square of l ength of hypotenuse.

2 2 2
a b c + =

In any tri angl e, the sum of any two si des i s al ways greater than the thi rd
one. And the di f f erence of any two si des i s al ways l ess than the thi rd one.

a b c + >
and
a b c <


The perimeter of a tri angl e i s cal cul ated by addi ng the l engths of al l the
si des of that tri angl e.
Student Guide 42


perimeter a b c = + +

The area of a tri angl e i s cal cul ated by the f ormul a:
1
2
area bh =
wher e
b
i s
the base of t he tri angl e and
h
i s the height of t he tri angl e.
o Any si de of tri angl e can be taken as t he base.
o Height i s the al ti tude (perpendi cul ar) drawn to the base from i ts
opposi te vertex.
o In a ri ght tri angl e any l eg coul d be taken as the base, the other
wi l l be the al ti tude.




Quadri l ateral and other Pol ygons

A polygon i s a cl osed geometri c fi gure, made up of l i ne segments. The
l i ne segments are cal l ed sides and the end poi nts of l i nes are cal l ed
vertices (pl ural of vertex). Li nes, i nsi de the pol ygon, drawn f rom one
vertex to the other, are cal l ed di agonal s.



The sum of the measures of t he
n
angl es i n a pol ygon wi th
n
si des i s
al ways
( 2) 180 n
.

In any quadri l ateral , the sum of the measures of the f our angl es i s
360
.

A regul ar pol ygon i s a pol ygon i n whi ch al l of the si des are of the same
l ength. In any regul ar pol ygon, the measure of each i nteri or angl e i s
( 2) 180 n
n

and the measure of each exteri or angl e i s
360
n

.
Student Guide 43
A paral l el ogram i s a speci al quadri l ateral , i n whi ch both pai rs of opposi te
si des are paral l el . The Fol l owi ng are some properti es of paral l el ogram.

o Lengths of opposi te si des are equal .
AB CD =
and
AD BC =

o Measures of opposi te angl es are equal .
a c =
and
b d =

o Consecuti ve angl es add up to
180
.
180 a b + =
,
180 b c + =
etc.
o The two di agonal s bi sect each other.
AE EC =
and
BE ED =

o A di agonal di vi des the paral l el ogram i nto two tri angl es that are
congruent.

A rectangle i s a paral l el ogram i n whi ch al l four angl es are ri ght angl es. It
has al l the properti es of a paral l el ogram. In addi ti on i t has the fol l owi ng
properti es:
o The measure of each angl e i n a rectangl e i s
90
.
o The di agonal s of a rectangl e are equal i n l ength.

A square i s a rectangl e that has t he fol l owi ng addi ti onal properti es:
o A square has al l i ts si des equal i n l ength.
o In a square, di agonals are perpendi cul ar to each other.





To cal cul ate the area, t he fol l owi ng formul as are requi red:
o For a paral l el ogram,
Area bh =
, where
b
is the base and
h
i s t he
hei ght.
o For a rectangle,
Area lw =
, where
l
is the l ength and
w
i s the
wi dt h.
o For a square,
2
Area s =
, where
s
i s t he si de of t he square.

Peri meter for any pol ygon i s t he sum of l engths, of al l i ts si des.
Student Guide 44
Circles

A circle consi sts of al l the poi nts that are the same di stance from one fi xed
poi nt cal l ed the center. That di stance i s cal l ed the radius of a circle. The
word radi us i s al so used to represent any of the l i ne segments joi ni ng the
cent er and a point on the ci rcl e. The pl ural of radi us i s radii .

Any tri angl e, such as
CED V
i n the fi gure, formed by connecti ng the end
poi nts of two radii, is an i soscel es.

A l i ne segment, such as
ED
i n the di agram above, both of whose end
poi nts are on a ci rcl e i s cal l ed a chord.

A chord that passes through the center of the ci rcl e i s cal l ed t he diameter
of t he ci rcl e. The l ength of t he di ameter i s al ways doubl e the radi us of the
ci rcl e. The di ameter i s the l ongest cord that can be drawn i n a ci rcl e.

The total l ength around a ci rcl e i s known as the circumference of the
ci rcl e.

The rati o of the ci rcumference to the di ameter i s al ways the same for any
ci rcl e. Thi s rati o i s denoted by the symbol

(pronounced as pi ).
2
C
C d C r
d
= = = where C i s t he ci rcumference, d i s t he di ameter
and r i s t he radi us of t he ci rcl e.
Val ue of

i s approxi matel y
3.14


An arc consi sts of two poi nts in a ci rcl e and al l the poi nts between them.
E.g. PQi s an arc i n the di agram.

An angl e whose vertex i s at the center of t he ci rcl e i s cal l ed the central
angle.
PCQ
i n the di agram above i s a central angle.

The degree measure of a compl ete ci rcl e i s
360
.

The degree measure of an arc i s the measure of the central angl e that
i ntercepts i t. E.g. the degree measure of

PQ i s equal to the measure of


PCQ i n the di agram above.
Student Guide 45
If
x
i s the degree measure of an arc, i ts l ength can be cal cul ated as
360
x
C
,
where C i s the ci rcumf erence.
The area of a ci rcl e can be cal cul ated as
2
r
.
The area of a sector formed by the arc and two radii can be
calculated as
A
x
360
, where A is the area of a circle.

2.2 Discrete Quantitative Questions

These are standard multiple-choice questions. Most of such questions require you to do some
computations and you have to choose exactly one of the available choices based upon those
computations. This section will teach you the basic tactics to attempt such questions.

2.2.1 Question format
Each question will consist of a question statement and the choices labeled from A to E. The
number of choices may vary from 2 to 5, but exactly one choice will be correct for each
question.

2.2.2 How to attempt?

Following are some tactics, which will lead you to the correct answer.

Whenever you know how to answer a questi on di rectl y, just do i t. The
tacti cs shoul d be used onl y when you do not know the exact sol uti on, and
you j ust want to el i mi nate the choi ces.

Remember that no probl em requi res l engthy or di ffi cul t computati ons. If
you fi nd yoursel f doi ng a l ot of compl ex ari thmeti c, thi nk agai n. You may
be goi ng i n t he wrong di recti on.

Whenever there i s a questi on wi th some unknowns (vari abl es), repl ace
them wi th t he appropri ate numeri c val ues for ease of cal cul ati on.

When you need to repl ace vari abl es wi th val ues, choose easy-t o-use
numbers, e.g. the number 100 i s appropri ate i n most percent-rel ated
probl ems and the LCD (l east common denomi nator) i s best sui ted i n
questi ons that i nvol ve f racti ons.

Appl y back-sol vi ng whenever you know what to do to answer the questi on
but you want to avoi d doi ng al gebra. To understand thi s tacti c read the
f ol l owi ng exampl e:

On Monday, a storeowner recei ved a shi pment of books. On Tuesday, she
sol d hal f of them. On Wednesday after two more were sol d, she had
exactl y 2/5 of the books l ef t. How many were i n the shi pment?

(A) 10 (B) 20 (C) 30 (D) 40 (E) 50

now by thi s tacti c:

Assume that (A) i s the correct answer, i f so; she must have 3 books on
Wednesday. But 2/5 of 10 are 4, so, (A) i s i ncorrect;

Assume that (B) i s the correct answer, i f so; she must have 8 books on
Wednesday. 2/5 of 20 are 8, so, (B) i s the correct choi ce, and as there may
Student Guide 46
be onl y one correct choi ce, there i s no need to check for remai ni ng choi ces.

Thi s tacti c i s very hel pf ul when a normal al gebrai c sol uti on for the probl em
i nvol ves compl ex or l engthy cal cul ati ons.
If you are not sure how to answer the questi on, do not l eave it
unanswered. Try to el i mi nate absurd choi ces and guess f rom the remai ni ng
ones. Most of the ti mes four of the choi ces are absurd and your answer i s
no l onger a guess.

Many thi ngs may hel p you to real i ze that a parti cul ar choi ce i s absurd.
Some of them are l i sted bel ow.

o The answer must be posi ti ve but some of the choi ces are negati ve
so el i mi nate al l t he negati ve ones.
o The answer must be even but some of the choi ces are odd so
el i mi nate al l t he odd choi ces.
o The answer must be l ess then 100, but some of the choi ces are
greater than 100 (or any other val ue) so el i mi nate al l choi ces that
are out of range.
o The answer must be a whol e number, but some of the choi ces are
f racti ons so el i mi nate al l f racti ons.
o These are some exampl es. There may be numerous si tuati ons
where you can appl y thi s tacti c and fi nd the correct answer even i f
you do not know the ri ght way to sol ve the probl em.

Example questions with solutions

The fol l owi ng are some exampl es, whi ch wi l l hel p you to master these types of
questi ons.

Example
If 25% of 220 equal s 5. 5% of X, what i s X?
(A) 10 (B) 55 (C) 100 (D) 110 (E) 1000

Solution:
Si nce 5. 5% of X equal s 25% of 220, X i s much greater than 220. So, choi ces
A, B, C, and D are i mmedi atel y el i mi nated because these are not l arger than
220. And t he correct answer i s choi ce E.

(Note: An important point here is that, even if you know how to solve a problem, if
you immediately see that four of the five choices are absurd, just pick the remaining
choice and move on.)

Example
Sci ence students choose exactl y one of three fi el ds (i .e. medi cal sci ences,
engi neeri ng sci ences and computer sci ences). If, i n a col l ege, three -fi fths of
the students choose medi cal sci ences, one-forth of the remai ni ng students take
computer sci ences, what percent of the students take engi neeri ng sci ences?

(A) 10 (B) 15 (C) 20 (D) 25 (E) 30

Solution:
The l east common denomi nator of 3/5 and 1/4 is 20, so assume that there are
20 students i n that col l ege. Then the number of students choosi ng medi cal
sci ences i s 12 (3/4 of 20). Of the remai ni ng 8 students, 2 (1/4 of 8) choose
computer sci ences. The remai ni ng 6 choose engi neeri ng sci ences. As 6 i s
30% of 20, the answer i s E.
Student Guide 47

Example
If a school caf eteri a needs C cans of soup each week f or each student and
there are S students, f or how many weeks wi l l X cans of soup l ast?

(A) CX/S (B) XS/C (C) S/CX (D) X/CS (E) CSX

Solution:
Repl ace C, S and X wi th three easy to use numbers. Let C=2, S=5 and X=20.
Now each student wi l l need 2 cans per week and there are 5 students, so 10
cans are needed per week and 20 cans wi l l l ast f or 2 weeks. Now put these
val ues i n choi ces to f i nd the correct one.
The choi ces A, B, C, D and E become 8, 50, 1/8, 2 and 200 respecti vel y. So
the choi ce D represents the correct answer.

2.3 Quantitative Comparison Questions

Some of the questi ons i n the Quanti tati ve secti on of the test may be
quanti tati ve compari son questi ons. The Fol l owi ng text wi l l expl ai n you the
format and techni ques u need to attempt the questi ons of thi s f ormat.

2.3.1Question format

Such questi ons consi st of two quanti ti es, one i n col umn A and the other i n
col umn B. You have to compare the two quanti ti es. The i nformati on
concerni ng one or both quanti ti es i s presented be fore them. Onl y the fol l owi ng
f our choi ces wi l l be gi ven:

A. The quanti ty i n col umn A i s greater
B. The quanti ty i n col umn B i s greater
C. The two quanti ti es i n both col umns are equal
D. The rel ati onshi p cannot be determi ned f rom the i nf ormati on gi ven

And as it is clear from the choices, only one will be correct at one time. Your job is
to choose one of them after careful comparison. The following text explains some
simple tactics to attempt such questions.

2.3.2How to attempt

Whenever you encounter a quanti tati ve compari son questi on, the fol l owi ng
gui del i nes wi l l hel p you to f i nd the correct answer qui ckl y.

If the questi on i nvol ves some vari abl es, repl ace them wi th appropri ate
numbers. Here are some gui del i nes i n choosi ng an appropri ate number:
o The very best numbers to use are 1, 0 and 1.
o Of ten f racti ons between 0 and 1 are usef ul (e.g. 1/2, 3/4 etc.).
o Occasi onal l y, l arge numbers such as 10 or 100 can be used.
o If there i s more than one vari abl e, i t i s permi ssi bl e to repl ace each
wi th the same number.
o Do not i mpose any un-speci f i ed condi ti ons on numbers. Choose
them randoml y.

El i mi nate the choi ces and choose f rom the remai ni ng ones. For exampl e If
you found the quanti ti es ever equal , the correct choi ce coul d never be A or
B, so, el i mi nate A and B.
A quanti tati ve compari son questi on can be treated as an equati on or
i nequal i ty. Ei ther:
Column A < Column B, or
Column A = Column B, or
Student Guide 48
Column A > Column B
So, you can perf orm si mi l ar operati on on both col umns to si mpl i f y the
probl em j ust as i n equati ons (or i nequal i ti es).

Example:
m > 0 and m 1




In thi s exampl e di vi de both the quanti ti es by m2. Thi s wi l l change
col umn A to 1 and col umn B to m. Now the compari son i s very si mpl e,
as we know t hat m i s greater than 0 and cannot be 1. So the
rel ati onshi p i s not determi nabl e usi ng the current i nformati on. m can
be both greater than 1 or between 0 and l ess than 1.
2.3.3 Example questions with Answers and
Explanations
Example 1:
A student earned a 75 on each of her f i rst three
math tests and an 80 on her fourth and fi fth
t est s.
A B
Average after 4 tests Average after 5 tests
A. The quanti ty i n col umn A i s greater
B. The quanti ty i n col umn B i s greater
C. The two quanti ti es i n both col umns are equal
D. The rel ati onshi p cannot be determi ned f rom the i nformati on gi ven
Remember you want to know whi ch average i s hi gher, not what the averages
are. After 4 tests, the average i s cl earl y l ess than 80, so an 80 on the fi fth
test had to rai se the average. So the answer i s choi ce (B).


Example 2:
A B
The time i t takes to
dri ve 40 mi l es at 35
mph
The ti me i t takes to
dri ve 35 mi l es at 40
mph
A. The quanti ty i n col umn A i s greater
B. The quanti ty i n col umn B i s greater
C. The two quanti ti es i n both col umns are equal
D. The rel ati onshi p cannot be determi ned f rom the i nf ormati on gi ven

Once agai n there i s no need for cal cul ati on, as the speed i n col umn B i s hi gher
than that i n col umn A. It i s obvi ous that i t wi l l take l ess ti me to travel shorter
di stance at a greater speed. So the val ue i n col umn A i s l arger. The answer is
opti on (A).

Example 3:
A B
m
2
m
3

Student Guide 49
20
2

5
5


A. The quanti ty i n col umn A i s greater
B. The quanti ty i n col umn B i s greater
C. The two quanti ti es i n both col umns are equal
D. The rel ati onshi p cannot be determi ned from the i nf ormati on gi ven
Square each col umn:
2
20 20
5
2 4

= =



and
2
5 25
5
5
5

= =


. So both col umns are
equal and the answer i s choi ce (C).
Example 4:
A B
13y

15y

To sol ve thi s questi on, subtract
13y
f rom both col umns to get
13 13 0 y y =
for
col umn A and
15 13 2 y y y =
for col umn B. As there are no restri cti ons,
2y
can
be greater than, l ess than or equal to 0. So the correct choi ce i s (D).

2.4 Data Interpretation Questions

These questi ons are based on the i nf ormati on that i s presented i n the f orm of
a graph, chart or tabl e. Most of the data i s presented graphi cal l y. The most
common types of graphs are l i ne graphs, bar graphs and ci rcl e graphs. The
objecti ve of such questi ons i s to test your abi l i ty to understand and anal yze
stati sti cal data.

2.4.1 Question Format
Data i nterpretati on questi ons al ways appear i n sets, you are presented wi th
some data i n any f ormat (chart, graph or tabl e), and you wi l l then be asked
wi th some questi ons about that data.

The f ol l owi ng exampl e expl ai ns the f ormat of such questi ons.

Example:

Question 1:
What i s the average sal e, i n mi l l i on Rs., f or the peri od 1994-2000?
(A) 5.5 (B) 6.0 (C) 7.0 (D) 8.0
(E) 8.5

Question 2:

For which year, the percentage increase in sales from the previous year is the greatest.
(A) 1995 (B) 1996 (C) 1999 (D) 2000
(E) 2001

2.4.2 How to attempt
Student Guide 50
Do not try to answer such questi ons i mmedi atel y, fi rst of al l read the
presented data careful l y. You must be very cl ear about the data and i ts
meani ngs even bef ore readi ng the f i rst questi on.

Do not confuse numbers wi th percents. Thi s confusi on i s most l i kel y to
occur when data i s presented i n pi e graphs. For exampl e i n the
f ol l owi ng graph

0
2
4
6
8
10
12
1994 1995 1996 1997 1998 1999 2000 2001
Years
S
a
l
e
s

i
n

m
i
l
l
i
o
n

R
s
.



Now i t woul d be a great mi stake here to thi nk that sal es of TVs & VCRs i s
15% more than the sal es of Computers i n 2001 by XYZ Corporati on. To
know thi s you have to cal cul ate i t as
15
100 60%
25
=
.
Try to avoi d un-necessary cal cul ati ons. Most of the questi ons coul d
easi l y be sol ved by observati on and esti mati on. Use esti mati on to
el i mi nate the choi ces, i f you are not abl e to fi nd the correct answer
wi thout cal culati on. For exampl e to sol ve Questi on 1 presented i n the
exampl e at the start of thi s secti on, i f you are not sure of the correct
answer, you can then try to cut down the number of possi bl e choi ces by
observati on. You are bei ng asked to tel l the percentage i ncrease.
Where as, i n year 2000, the sal e i s decreasi ng i nstead of i ncreasi ng, so,
you can i mmedi atel y el i mi nate choi ce (D) i n that questi on.
Student Guide 51
Your answers must be based upon the i nformati on presented i n the
gi ven charts and graphs. If your knowl edge contradi cts any of the data
presented, i gnore what you know and sti ck to the presented data. The
presented data shoul d be the onl y base f or your cal cul ati ons and
esti mati ons.
Al ways use the proper uni ts, there may be some questi ons that ask you
to compare di fferent data i tems possi bl y from di fferent data sets. Be
caref ul about the uni ts used to represent the data.
Because graphs and charts present data i n a f orm that enabl es you to
readi l y see the rel ati onshi ps among val ues and to make qui ck
compari sons, you shoul d al wa ys try to vi sual i ze you answer i n the same
f ormat as the ori gi nal data was presented.
Be sure that your answer i s reasonabl e. For exampl e, the profi t coul d
never i ncrease the actual sal es, or the expenses coul d never be negati ve
etc. Whi l e answeri ng the questi on, fi rst of al l el i mi nate such un-
reasonabl e choi ces, and then choose f rom the remai ni ng ones.

2.5 Practice exercise
1 What i s the average of posi ti ve i ntegers f rom 1 to 100 i ncl usi ve?

(A) 49
(B) 49.5
(C) 50
(D)50.5
(E) 51

2 If
6 x y + =
,
7 y z + =
, and
9 x z + =
, what i s the average of
x
,
y
and
z
?
(A)
11
3
(B)
11
2
(C)
22
3

(D)
11
(E)
22


3 In the di agram bel ow, l i nes l and m are not paral l el .



If A represents the average measure of al l the ei ght angl es, what i s the val ue
of A?
(A)
45 A =

(B)
45 90 A < <

(C)
90 A =

(D)
90 180 A < <

(E)
180 A =


4 Asl am has 4 ti mes as many books as Sal man and 5 ti mes as many as Javed.
If Javed has more than 40 books, what i s the l east number of books t hat
Asl am coul d have?

(A) 200 (B) 205 (C) 210 (D)220 (E) 24
Student Guide 52
5 Asl am i s now 3 ti mes as ol d as Javed, but 5 years ago, he was 5 ti mes as
Javed was. How ol d i s Asl am now?

(A) 10 (B) 12 (C) 24 (D)30 (E) 36
6 If
% x
of
y
i s 10, what i s y?
(A)
x
10

(B)
x
100

(C)
x
1000

(D)
100
x

(E)
10
x


Answer Key
1 D
2 A
3 A
4 D
5 D
6 C

Student Guide 53
3 Analytical Ability

3.1.1 Question format

Each anal yti cal reasoni ng questi on i s a l ogi cal puzzl e, based on a gi ven set of
condi ti ons. Li ke mathemati cal questi o ns, these questi ons have exactl y one
correct answer, whi ch i s what you need to sel ect.

Anal yti cal reasoni ng questi ons are presented i n groups of four or fi ve
questi ons. Each group i s based on a short passage fol l owed by a set of
condi ti ons. Occasi onal l y, there are graphs and tabl es i nstead of a passage.
To understand the general f ormat of t he questi on, consi der the fol l owi ng
exampl e.

Question 1-4:
As part of thei r sports physi cal , seven col l ege athl etes F, G, H, I, J, K and L
are bei ng wei ghed. In announci ng the resul ts of the physi cal exams, the
coach has gi ven the f ol l owi ng i nf ormati on.
i. None of the athl etes i s exactl y the same wei ght as another athl ete.
i i . K i s heavi er than L, but l i ghter than H.
i i i . I i s heavi er than J
iv. Both F and G are heavi er than H.
1 Each of the f ol l owi ng coul d be true EXCEPT
A. F i s the heavi est.
B. G i s the heavi est.
C. I i s the heavi est.
D. More than three athl etes are heavi er than K.
E. More than three athl etes are l i ghter than K.
2 Whi ch of the fol l owi ng, i f true, woul d be suffi ci ent to determi ne whi ch
athl ete i s the l i ghtest?
A. I i s the heavi est
B. I i s l i ghter than K
C. K i s heavi er than J
D. J i s heavi er than K
E. Exactl y f i ve students are l i ghter than F.

3 If J i s heavi er than F, how many di fferent ranki ngs by wei ght, of the athl etes
are possi bl e?
A. 1 B. 2 C. 3 D. 4 E. 5
4 If H i s heavi er than I, whi ch of the f ol l owi ng CANNOT be true?
A. I s wei ght i s equal to the average of F s wei ght and G s wei ght.
B. I s wei ght i s equal to the average of K s wei ght and L s wei ght
C. J s wei ght i s equal to the average of K s wei ght and L s wei ght
D. J i s the second l i ghtest.
Answers:

1. E 2. D 3. C 4. A
3.1.2How to attempt
Si mpl i fy the i nformati on by usi ng abbrevi ati ons and symbol s. The fi rst
step i s to stri p away al l of the excess verbi age from the gi ven passage
and then to abbrevi ate the remai ni ng key words wi th si ngl e l etters. For
exampl e, i n the questi on statement f i ve musi ci ans a bassi st, a
drummer, a gui tari st, a pi ani st, and a trumpeter are performi ng i n a
tal ent show, you shoul d i mmedi atel y abbrevi ate them B, D, G, P and T.
Student Guide 54
You can use abbrevi ated l etters to represent a whol e sentence al so. You
shoul d use symbol s to represent condi ti ons. You may devel op your own
symbol i c conventi ons for thi s. The objecti ve i s to convert the probl em
i nto notati ons, so that, i t i s easi l y understandabl e. The fol l owi ng i s a
basi c set of symbol s, whi ch are most commonl y used.
A represents the statement Akbar i s goi ng.
B represents the statement Babur i s goi ng.

Symbo
l
Meaning Examples
~ Not ~A Akbar i s not goi ng. Or you can say,
it i s not the case that Akbar i s
goi ng.


And A

B Akbar and Babur are goi ng.


A

~B Akbar i s goi ng and Babur i s not


goi ng.


Or A

B Akbar or Babur i s goi ng.


A

~B Akbar i s goi ng or Babur i s not goi ng.


If , then
A

B
If Akbar i s goi ng then Babur i s goi ng.
(A

B)

S
If Akbar and Babur are goi ng, then
Sal eem i s goi ng.


If and onl y i f
A

B
Babur i s goi ng, i f and onl y i f Akbar i s
goi ng.
Before l earni ng the tacti cs to attempt an anal yti cal reasoni ng questi on,
you must be fami l i ar wi th some ba si c l ogi c facts, whi ch are expl ai ned i n
the f ol l owi ng text. Consi der A and B are two statements.
o A i s true means ~A i s f al se.
o ~A i s true means A i s f al se.
o (A

B) i s true means both A and B are true.


o (A

B) is true means ei ther A or B or both are f al se.


o (A

B) i s true means ei ther A or B or both are true.
o (A

B) i s f al se means both A and B are f al se.
o ~(A

B) i s equi val ent to (~A

~B).
o ~(A

B) i s equi val ent to (~A

~B).
o If (A

B) i s true then
If A i s true B i s al so true.
If A i s f al se B may be true or f al se.
o If (A

B) i s f al se then A i s true and B i s f al se.
o (A

B) i s equi val ent to (~B

~A)
o (A

B) i s true means:
If A i s true B i s true.
If A i s fal se B i s fal se.
o (A

B) i s f al se means:
If A i s true B i s f al se.
If A is f al se B i s true.
o (A

B) i s equi val ent to [(A

B)

( B

A)].

You must be fami l i ar wi th the most common types of anal yti cal
reasoni ng questi ons. The f ol l owi ng f our types occur more frequentl y
than the others, and when you see them, you shoul d i mmedi atel y know
what you need to do to answer them.
o Which of the following could be true? If onl y one of the
answer choi ces coul d be true, then each of the other four choi ces
must be fal se; that i s, each one must vi ol ate at l east one of the
gi ven condi ti ons.

Student Guide 55
o Which of the following must be true? Si nce onl y one of the
answer choi ces must be true, then for each of the choi ces, ei ther
i t i s fal se or i t i s possi bl y (but not defi ni tel y) true. You have to
choose onl y that choi ce whi ch i s def i ni tel y true.

o Which of the following cannot be true? Si nce onl y one of the
answer choi ces cannot be true, then each of the other choi ces
coul d be true. The correct answer i s the onl y choi ce, whi ch
vi olates at l east one of the gi ven condi ti ons or i s otherwi se
i nconsi stent wi th what you know must be true.
o How many possibilities are there? Thi s questi on asks, How
many di fferent ways are there to sati sfy al l of the gi ven
condi ti ons? Here, you must systemati cal l y count or l i st al l of the
possi bi l i ti es that do not vi ol ate any of the condi ti ons.

Identi fy the key words that serve to l i mi t the si tuati on. Certai n words
are cri ti cal to your understandi ng of the si tuati on. Be sure to
i ncorporate your symbol s. Some frequentl y used key words are l i sted
bel ow:

Af ter All Al ways At l east At most
Before But Can be Cannot be Consecuti ve
Di fferent Di rectl y Each
No fewer
t han
No more than
Onl y Possi bl e Enti re Every Exactl y
Except Fewer Fi rst If If and onl y i f
Imme di atel y Impossi bl e Last Least Most
Must be Same Some The Least The Most
Unl ess Smal l est Greatest None

Note that certai n key words have onl y one f uncti on, to rul e out a
potenti al ambi gui ty.

El i mi nati ng the choi ces i s al ways a good strategy. Whi l e el i mi nati ng the
choi ces, fi rst of al l , el i mi nate those whi ch are rul ed out by i ndi vi dual
condi ti ons: Then work through the remai ni ng choi ces.

Study condi ti ons, not merel y for what they state but al so for what they
i mpl y. Certai n anal yti cal reasoni ng questions resembl e the i nference
questi ons you fi nd i n the readi ng comprehensi on secti on. To answer
them correctl y, you must understand not onl y what the condi ti ons state
expl i ci tl y, but al so what they i mpl y.

Often the key to answeri ng anal yti cal reasoni ng que sti ons i s to organi ze
the gi ven i nf ormati on i n a l i st or tabl e.
On some anal yti cal reasoni ng questi ons, an excel l ent way to deal wi th
the i nformati on i s to draw a si mpl e di agram, pi cture, or map. Thi s i s
parti cul arl y hel pf ul when you are deal i ng wi th the physi cal or temporal
order of thi ngs. It i s much easi er to tel l whether person A can be
seated opposi te person B i f you have sketched a di agram of the tabl e; i t
i s easi er to know whether person C i s ol der or younger than person D i f
you have entered al l of the gi ven i nformati on on a ti me l i ne; and i t i s
easi er to determi ne whether town E i s east of town W i f you have drawn
a si mpl e map.
Student Guide 56

3.1.3 Example questions with Answers and
Explanations
Questions 1-5:
Si x actors ---- Bob, Carol , Dave Ed, Frank, and Gra ce audi ti on for a part i n an
off-Broadway pl ay. The audi ti ons wi l l take pl ace over four consecuti ve days,
starti ng on a Thursday. Each actor wi l l have one audi ti on; the days on whi ch
the di f f erent actors wi l l audi ti on must conf orm to the f ol l owi ng condi t i ons.

i. At l east one audi ti on wi l l take pl ace each day.
i i . No more than two audi ti ons wi l l take pl ace on any day.
i i i . No more than three audi ti ons wi l l take pl ace on any two
consecuti ve days.
iv. Bob s audi ti on must take pl ace on Saturday.
v. Carol s audi ti on must take pl ace on the same day as another
audi ti on.
vi. Frank s audi t i ons must take pl ace on the day before Grace s
audi ti on.
vi i . Dave s audi ti on must take pl ace on a day af ter Ed s audi ti on.

1 If onl y one audi ti on takes pl ace on Thursday whi ch actor coul d have that
audi ti on?
(A) Bob (B) Car ol (C) Dave (D)Frank (E) Grace

2 If Bob s and Frank s audi ti ons are on the same day, whi ch of the fol l owi ng
must be true
(A) Dave s audi ti on wi l l take pl ace on Thursday
(B) Dave s audi ti on wi l l take pl ace on Fri day
(C) Grace s audi ti on wi l l take pl ace on Thursday
(D)Carol s audi ti on wi l l take pl ace on Sunday
(E) Ed s audi ti on wi l l take pl ace on Sunday

3 If the di rector deci des to hol d two audi ti ons on Thursday and two on
Sunday, how many actors woul d be el i gi bl e to audi ti on on Fri day?
(A) 1 (B) 2 (C) 3 (D)4 (E) 5

4 If Ed and Grace have thei r audi t i ons on the same day whi ch of the fol l owi ng
must be true?
(A) Ed s audi ti on wi l l take pl ace on Thursday.
(B) Frank s audi ti on wi l l take pl ace on Fri day.
(C) Carol s audi ti on wi l l take pl ace on Saturday.
(D)Grace s audi ti on wi l l take pl ace on Saturday.
(E) Carol s audi ti on wil l take pl ace on Sunday.

5 If Ed s audi ti on i s on Saturday, whi ch of the fol l owi ng actors cannot
audi ti on on the same day as any other actor?
(A) Bob
(B) Car ol
(C) Ed
(D)Frank
(E) Gr
ace
Questions 6-10:
Duri ng the fi rst hal f of the year, from January through June, the chai rperson of
the mathemati cs department wi l l be on sabbati cal . The dean of the col l ege has
asked each of the si x professors i n the department --- Arkes, Borofsky, Chang,
Denture, Hobbes, and Lee --- to serve as acti ng chai rperson duri ng one of
Student Guide 57
those months. The mathemati ci ans can deci de the order i n whi ch they wi l l
serve, subj ect onl y to the f ol l owi ng cri teri a establ i shed by the dean.
i. Chang wi l l serve as chai rperson i n February.
i i . Arkes wi l l serve as chai rperson bef ore Hobbes does.
i i i . Borofsky and Dexter wi l l serve as chai rpersons i n consecuti ve
months.

6 Whi ch of the f ol l owi ng prof essors coul d serve as chai rperson i n January?

(A) Borodfsky (B) Chang (C) Dexter (D)Hobbes (E) Lee

7 In how many ways can the schedul e be made up i f Lee has to serve as
chai rperson i n May?
(A) 1
(B) 2
(C) 3
(D)4
(E) 6

8 If Lee serves i n Apri l , al l of the f ol l owi ng coul d be true EXCEPT
(A) Arkes serves i n January
(B) Hobbes serves i n march
(C) Borof sky serves i n may
(D)Borof sky serves i n June
(E) Hobbes serves i n June
Student Guide 58
9 If Borof sky serves i n May, what i s the l atest month i n whi ch Arkes coul d
serve?
(A) January
(B) February
(C) March
(D)April
(E) June

10 Whi ch of the f ol l owi ng CANNOT be true?
(A) Arkes and Lee serve i n consecuti ve months.
(B) Lee and Hobbes serve i n consecuti ve months.
(C) Hobbes and Dexter serve i n consecuti ve months.
(D)Arkes and Chang serve i n consecuti ve months.
(E) Borof sky and Chang serve i n consecuti ve months.

Solutions 1-5:
Fi rst express each of the condi ti ons symbol i cal l y:
B, C, D, E, F, and G: 1 audi ti on each
Days: Thu, Fri , Sat, Sun
Each day: 1 or 2 audi ti ons
2 consecuti ve days: 2 or 3 audi ti ons
B=Sat Cx F<G E<D
1 A vi ol ates the condi ti on that Bob s audi ti on wi l l take pl ace on Saturday
(B=Sat). B vi ol ates the condi ti on that Carol s audi ti on cannot be the onl y
audi ti on on a parti cul ar day (Cx). Choi ces C and E are i mpossi bl e. Si nce
Dave s audi ti on must take place on a day af ter Ed s audi ti on (E<D) and Grace s
audi ti on must take pl ace on a day after Frank s audi ti on (F<G) nei ther can
take pl ace on Thursday. Onl y choi ce D does not vi ol ate any of the gi ven
condi ti ons, so thi s i s the correct answer.

2 The condi ti on that Bob s and Frank s audi ti ons are on the same day compl etel y
determi nes the schedul e. They must take pl ace on Saturday (B=Sat). To
avoi d havi ng more than three audi ti ons on two consecuti ve days, there can be
onl y one audi ti on on Fri day and one on Sunday, whi ch means there wi l l be two
on Thursday. Si nce Frank must have to precede Grace (F<G), Grace s audi ti on
wi l l take pl ace on Sunday. Si nce Ed must precede Dave, Ed s audi ti on wi l l take
pl ace on Thursday and Dave s audi ti on on Fri day. Fi nal l y, Carol s audi ti on wi l l
be the second audi ti on on Thursday. The fi nal schedul e i s C and E on
Thursday, D on Fri day, B and F on Saturday and G on Sunday. Onl y choi ce B
i s consi stent wi th thi s schedul e, so B i s the correct choi ce.
3 Si nce onl y one audi ti on can take pl ace on Fri day, i t cannot be Carol s (Cx);
and, of course, i t cannot be Bob s (B = Sat). Any of the other four actors
coul d audi ti on on Fri day as i ndi cated i n the f ol l owi ng schedul es:
E/F on Thu, D on Fri , B on Sat, C/G on Sun
C/F on Thu, E on Fri , B on Sat, D/G on Sun
C/E on Thu, E on Fri , B on Sat, D/G on Sun
E/F on Thu, G on Fri , B on Sat, C/D on Sun
So the correct choi ce i s D.

4 The onl y schedul e that ful fi l s the condi ti ons i s F on Thu, E/G on Fri , B on Sat,
and C/D on Sun. Onl y choi ce E i s consi stent wi th thi s schedul e.

5 Si nce Ed and Bob s audi ti ons are both taki ng pl ace on Saturday, el i mi nate
choi ces A and C. Si nce Carol e must audi ti on on the same day as another
actor, el i mi nate B. Fi nal l y, si nce Dave s audi ti on must take pl ace on Sunday
(E < D), Frank s audi ti on must take pl ace on Thursday and Grace s audi ti on on
Fri day (F < G). El i mi nate choi ce D. The compl ete schedul e i s: C/F on Thu, G
on Fri , B/F on Sat, and D on Sun.
Solutions 6-10:
Let A, B, C, D, H, L represents prof essor names.
C=February, A<H, B<<D and D<<B

Student Guide 59
6 Onl y choi ce E i s there, whi ch does not vi ol ate any of the condi ti ons, so i s the
correct choi ce.

7 Wi th C servi ng i n February and L i n May, the onl y co nsecuti ve months
avai l abl e f or B and D are March and Apri l . Then si nce A must serve is before H
i n June. There are two possi bl e schedul es, dependi ng on the order of B and D,
so the correct choi ce i s B.

8 If L serves i n Apri l , the consecuti ve months avai l abl e for B and D are May and
June; so choi ces C and D coul d be true. Si nce A must serve before H, choi ces
A and B must be true, onl y choi ce E cannot be true.

9 Si nce A must serve before H does, A cannot serve i n June. Can A serve i n
Apri l ? No, because then, D woul d serve i n June (B<<D or D<<B), and agai n A
woul d not precede H. The l atest that A coul d serve i n March, whi ch coul d
occur i n the f i nal order: L, C, A, D, B and H.

10 The onl y professors that can serve i n January are A and L, so, one of them
must serve i n January, and nei ther serves i n February. So choi ce A cannot be
true.

3.2 Logical Reasoning

Each l ogi cal reasoni ng questi on requi res you to anal yze an argument presented i n
a short passage. Often you are asked ei ther to fi nd a concl usi on that i s a l ogi cal
consequence of the passage, or to choose a statement that, i f true, strengthen or
weakens the argument.

3.2.1Question format
Logi cal reasoni ng questi ons are based upon a passage cal l ed argument. You have
to anal yze the argument presented i n the passage. The passage i s f ol l owed by a
questi on. Occasi onal l y, there i s more than one questi on rel ated be the same
passage. No matter what the number i s, the questi ons al ways ai m at your abi l i ty
to understand the concl usi on reached by the author of the passage, and to gi ve
argument and contra arguments. Logi cal reasoni ng questi ons are a l ot l i ke readi ng
comprehensi on questi ons i n a verbal secti on.

For each l ogi cal reasoni ng questi on, the argument i s fol l owed by a mul ti choi ce
questi on. The choi ces are si mpl e statements. Mostl y the questi on statement
begi ns wi th the phrase which of the fol l owi ng statements. Here are a few
exampl es:
Whi ch of the fol l owi ng statements i s an assumpti on on whi ch the concl usi on
of thi s argument i s based?

Whi ch of the f ol l owi ng statements i denti f i es a f l aw i n the reasoni ng of thi s
argument?

Whi ch of the fol l owi ng statements can be most reasonabl y i nferred, from
the statements i n the gi ven passage?

Whi ch of the fol l owi ng statements, i f true, woul d most seri ousl y, weaken
the argument of f ered?

Whi ch of the f ol l owi ng statements, i f true, woul d strengthen the concl usi on
i n the precedi ng argument?

Whi ch of the f ol l owi ng statements woul d be the most i mportant to know to
eval uate the argument gi ven i n the precedi ng paragraph? Every l ogi cal
reasoni ng questi on does not fi t thi s mol d, but you shoul d try.


Student Guide 60
3.2.2 How to attempt

Whi l e attempti ng l ogi cal reasoni ng questi ons, you shoul d read the questi on
statement before readi ng the argument. Then you shoul d anal yze the
argument presented i n the passage. You must know what aspect of the
argument you are to concentrate on, and focus on i t. By thi s, you wi l l not
be unnecessari l y wasti ng your ti me .
You must be abl e to spot the questi on type by readi ng the questi on
statement. If you do thi s, you wi l l be better abl e to approach the
argument i n hand. The fol l owi ng si x categori es are those whi ch most
commonly occur:

1 Assumption: Questi ons that test your abi l i ty to recogni ze the premi ses on
whi ch an argument i s based, of ten take the f ol l owi ng f orms:
o The concl usi on above depends on whi ch of the fol l owi ng
assumpti ons?
o The author of the passage above makes whi ch of the fol l owi ng
assumpti ons?
o In the passage above, the author assumes whi ch of the f ol l owi ng
statement to be true?

2 Inference: Questi ons, whi ch test your abi l i ty to go beyond the author s
expl i ci t statements and see what these statements i mpl y, may be worded l i ke
t hese.
o It can be i nf erred f rom the passage above that the author
bel i eves that
o Whi ch of the f ol l owi ng i s i mpl i ed by the passage above?
o From the i nformati on above, whi ch of the fol l owi ng i s the mos t
reasonabl e i nference?

3 Conclusion: Questi ons that test your abi l i ty to determi ne what cl ai m can
l ogi cal l y be made on the basi s of evi dence i n the passage above?
o If the statements above are true, whi ch of the fol l owi ng i n a
concl usi on that can be properl y drawn?
o The statements i n the passage, i f true, best supports whi ch of the
f ol l owi ng concl usi ons?

4 Central Point: Questi ons that test your abi l i ty to understand the
thrust of an argument.
o The statement si ted above conveys whi ch of the fol l owi ng
proposi ti ons?
o The author of the passage above argues that
o Whi ch of the fol l owi ng expresses the poi nt the author of the
passage above makes?

5 Support: Questi ons that test your abi l i ty to recogni ze whether an
asserti on supports or undermi nes an argument.
o Whi ch of the fol l owi ng, i f true, best supports the author s
concl usi on?
o Whi ch of the fol l owi ng, i f true, most weakens the author s
concl usi on?

6 Argument Evaluation: Questions that test your ability to judge
an argument.
o Whi ch of the f ol l owi ng i denti f i es a f l aw i n the speaker s
reasoni ng?
o Whi ch of the f ol l owi ng woul d be most i mportant to know when
eval uati ng the accuracy of the argument above?

Student Guide 61
Do not try to ski m the passage, read each argument careful l y. It i s not
enough to have a general i dea about the argument; you must be abl e to
anal yze i t very careful l y.

You must fi nd the concl usi on of the argument, whi ch the author cl ai ms to
have reached. That most common si tuati ons are as f ol l ows:
o The concl usi on i s the l ast sentence of the passage, often starti ng
by words s uch as so, therefore, thus, hence, consequentl y etc.
o The concl usi on i s the fi rst sentence of the passage fol l owed by
the supporti ng evi dence.
o Occasi onal l y, the concl usi on i s not present i n the passage; i n thi s
case, the questi on asks you to i denti f y the co ncl usi on.

Pay parti cul ar attenti on to si gnal words such as accordi ngl y, for thi s
reason, hence, al though, but, except, i n contrast, neverthel ess, unl i ke etc.

El i mi nati ng the choi ces i s al ways the best strategy i f you do not know what
the correct answer is. Thi s process wi l l el i mi nate some obvi ous wrong
choi ces. And you wi l l be abl e to make an educated guess from the
remai ni ng ones.

Every argument i s based upon certai n assumpti ons made by the author. If
an argument s basi c premi ses are sound, the argume nt i s strengthened. If
an argument s basi c premi ses are fl awed, the argument i s weakened. In
support questi ons, where you have to deci de about weakeni ng or
strengtheni ng the questi on, pi npoi nt what the argument assumes. Then
compare that assumpti on wi th the answer choi ces. If the questi on asks you
to fi nd the choi ce, whi ch most strengthens the argument, l ook for the
choi ce that i s most i n keepi ng wi th the argument s basi c assumpti on. If
the questi on asks you to choose the choi ce that most weakens the
argument, l ook for the answer that casts the most doubt on that
assumpti on.

Some l ogi cal reasoni ng questi ons are essenti al l y mi ni anal yti cal reasoni ng
questi ons, so, be fami l i ar wi th al l of the i mportant l ogi cal f acts and appl y
whenever needed. Exampl e questi ons wi th Answers and Expl anati ons

3.2.3Example Questions with Answers and Explanations

Questions 1-2:
The mi crowave oven has become a standard appl i ance i n many ki tchens, mai nl y
because i t of f ers a f ast way of cooki ng f ood. Yet, some homeowners beli eve that
the ovens are sti l l not compl etel y safe. Mi crowaves, therefore, shoul d not be a
standard appl i ance unti l they have been careful l y researched and tested.

1 Whi ch of the fol l owi ng, i f true, woul d most weaken the concl usi on of the
passage above?

(A) Homeowners, of ten purchase i tems despi te knowi ng they may be unsaf e.
(B) Those homeowners i n doubt about mi crowave safety ought not to purchase
mi crowaves.
(C) Research and testi ng of home appl i ances sel dom reveal s saf ety hazards.
(D)Mi crowaves are not as dangerous as steam i rons, whi ch are used i n al most
every home.
(E) Homeowners of ten purchase i tems that they do not need.

2 Whi ch one of the f ol l owi ng, i f true, woul d most strengthen the concl usi on of
the passage above?

(A) Homeowners of ten doubt the adverti sed saf ety of al l new appl i ances.
Student Guide 62
(B) Speed of f ood preparati on i s not the onl y concern of today s homeowner.
(C) Modern homeowners have more f ree ti me than ever bef ore.
(D)Food preparati on has become al most a sci ence, wi th more compl i cated and
i nvol ved reci pes.
(E) Many mi crowave ovens have been f ound to l eak radi oacti ve el ements.

3 Years ago, a pol l concl uded that there are more tel evi si ons than there are
bathtubs i n Ameri can homes. No doubt that fact remai ns today, especi al l y i n
l i ght of the growi ng popul ari ty of home computers. Now, in addi ti on to owni ng
tel evi si ons for entertai nment, more and more fami l i es are purchasi ng TV
moni tors for use wi th a personal computer. We can safel y guess that there are
sti l l many more peopl e stari ng at a pi cture tube than si ngi ng i n the shower.
Whi ch of the f ol l owi ng statements can be i nf erred f rom thi s passage?
(A) Personal computers probabl y cost l ess than i nstal l i ng a shower or bathtub.
(B) Peopl e can wash themsel ves wi thout a tub or shower, but they cannot
watch tel evi si on unl ess they own a tel evi si on set.
(C) TV moni tors wi l l work wi th personal computers i n pl ace of regul ar
computer moni tors.
(D)As many computers are sol d today as tel evi si on sets a f ew years ago.
(E) More tel evi si on moni tors are now used wi th personal computers than are
used to watch commerci al tel evi sion broadcast s.

4 Some sci enti sts have proposed that, over two hundred mi l l i on years ago, one
gi ant l and mass, rather than vari ous conti nents and i sl ands, covers one thi rd
of the earth. Long bef ore there was any human l i f e, and over vast peri ods of
time, is l ands and conti nents dri f ted apart. Austral i a was the fi rst to separate,
whi l e South Ameri ca and Afri ca were l ate i n spl i tti ng apart. Some i sl ands, of
course, were formed by vol canoes and were never part of the great l and mass.

Al l the f ol l owi ng woul d s upport the author s cl ai m EXCEPT

(A) Many of the pl ants of the South Ameri can rai n forests are markedl y si mi l ar
to those of Af ri can rai n f orests.
(B) Austral i a has more ani mal s that are not f ound in any other conti nent than
have several of the much l arger conti nent s.
(C) Vol cani c i sl ands l i ke Hawai i have ecosystems very di fferent from those of
conti nental l ands wi th the same average temperature.
(D)The pl ants of si mi l ar condi ti ons i n South Ameri ca have l ess i n common wi th
those of Austral i a than wi th those of Asi a, Af ri ca or Europe.
(E) The pri mi ti ve l anguages of Austral i a are unl i ke those of Afri ca, whi ch
resembl es those of South Ameri ca.
5 Every Saturday, Ami r has pi zza for l unch and then goes to the movi es.
If the statement above i s true, whi ch of the fol l owi ng statements must al so be
true?
1 If i t i s not Saturday, than Ami r i s not havi ng pi zza for l unch and i s not
goi ng t o the movi es.
2 If Ami r has pi zza f or l unch and then goes to the movi es, i t i s Saturday.
3 If Ami r has pi zza f or l unch, but does not go to the movi es, i t i s not a
Saturday.

(A) 1 onl y
(B) 2 onl y
(C) 3 onl y
(D)1 and 2 onl y
(E) 2 and 3 onl y

6 Anti freeze l owers the mel ti ng poi nt of any l i qui d to whi ch i t i s added so that
the l i qui d wi l l not freeze i n col d weather. It i s commonl y used to mai ntai n the
cool i ng system i n automobi l e radi ators. Of course, the weather may become
so col d that even anti f reeze i s not ef f ecti ve, but such a severe cl i mati c
condi ti on rarel y occurs i n wel l -travel ed pl aces.
Student Guide 63

Whi ch of the f ol l owi ng can be deduced f rom the passage?
(A) Wel l -travel ed pl aces have means of transportati on other than automobi l es.
(B) Anti freeze does not l ower the mel ti ng poi nt of certai n l i qui ds i n extreme
condi ti ons.
(C) Severe cl i mati c condi ti ons rarel y occur.
(D)It i s not often that many travel ers who use anti freeze have thei r cool i ng
system f reeze.
(E) Anti f reeze rai ses the mel ti ng poi nt of some l i qui ds.
Solutions:
1 The concl usi on of the passage i s that, because of safety concerns, more
research and testi ng ought to be done before mi crowaves become standard
househol d appl i ances. If, however, research and testi ng are i neffecti ve means
of di scerni ng safety probl ems (as choi ce C says), then research and testi ng
woul d be i rrel evant. Thi s cri ti ci sm seri ousl y weakens the concl usi on. So
choi ce C i s the correct answer.

2 If many mi crowave ovens have been f ound to l eak radi oacti ve el ements (as
choi ce E says), then the concl usi on that mi crowaves shoul d not be standard
appl i ances unti l they are more careful l y researched and tested i s further
strengthened because more safety concerns need to be addressed. So, choi ce
E i s the correct answer.

3 Though Choi ces A and B may wel l be true, they cannot be i nf erred f rom the
i nformati on i n the passage. But choi ce C can be i nferred si nce, more and
more fami l i es are purchasi ng TV moni tors for use wi th a personal computer.
TV moni tors must work wi th these computers, otherwi se, peopl e woul d not buy
them for that purpose. Choi ces D and E may or may not true, but they are not
i nferences from the passage, si mpl y addi ti onal i nf ormati on. So, the correct
choi ce i s C.

4 If Austral i a was the f i rst conti nent to separate, i t woul d f ol l ow that i ts f l ora
and f auna woul d devel op i n i sol ati on over a l onger peri od of ti me. Si mi l arl y,
we may expect the pl ants and ani mal of South Ameri ca and Afri ca that
separated l ater, to be more al i ke. Choi ces A, B, and D support these i deas.
The separatel y devel oped i sl ands are di f f erent at i s al so i n accord wi th the
passage. However the l anguages of al l the conti nents woul d have devel oped i n
i sol ati on, si nce man di d not evol ve unti l af ter the break-up of the l andmass,
and i t i s surpri si ng that Af ri can and South Ameri can l anguages are si mi l ar.
Human l i keness or di fferences are i rrel evant to the cl ai ms of the passage. So
choi ce E i s the correct answer.

5 Thi s l ogi cal reasoni ng questi on i s very easy as soon as you express the gi ven
statement symbol i cal l y. If i t i s Saturday, then Ami r has Pi zza and goes to
Movi es transl ates as
) ( M P S
. Thi s i s equi val ent to
S M P ~ ) ( ~
,
whi ch i s equi val ent to
S M P ~ ) ~ (~
. So i f ei ther P or M i s fal se, then S i s
f al se. Therefore, 3 i s true, nei ther 1 nor 2 are true. So, the correct choi ce i s
C.

6 Choi ce D i s the correct answer. Si nce severe cl i mati c condi ti ons rarel y occur
i n wel l -travel ed pl aces, i t i s not necessari l y true that It i s not often that
many travel ers who use anti freeze have thei r cool i ng systems freeze. Choi ce
A menti ons other means of transportati on, whi ch i s not addressed i n the
passage. Choi ce B ref ers to certai n l i qui ds.
Student Guide 64

4 Verbal Ability
The purpose of the Verbal Test i s to eval uate and anal yze your English
comprehensi on and understandi ng towards the l anguage. The questi ons wi l l be
basi cal l y asked to judge the sentence compl eti on, anal ogy and cri ti cal readi ng
ski l l s. The questi ons of di fferent types i .e. about sentence compl eti on and
anal ogy testi ng wi l l be asked randoml y. The questi ons about the cri ti cal readi ng
however wi l l be asked separatel y.

4.1 About the Verbal Questions
As already di scussed, thi s secti on wi l l consi st of the fol l owi ng types of questi ons
i.e. sentence compl eti on, anal ogy and the cri ti cal readi ng. The detai l about each
secti on i s as bel ow;

4.1.1 Sentence Completion
The questi ons that come under thi s category are provi ded wi th vari ous choi ces.
You are asked to compl ete the sentences by fi l l i ng i n the bl anks wi th the most
sui tabl e choi ce.
The questi ons for sentence compl eti on can be rel ated to any of the other areas of
study i .e. sci ence, l i terature, hi story, geography etc but the subject matter woul d
not hi nder your l anguage abi l i ti es. You are asked to compl ete the sentence wi th
the use of correct grammar or vocabul ary.

These questi ons try to determi ne your abi l i ty to recogni ze the correct sentence
structure, ri ght grammar and how you make the correct choi ce of vocabul ary.

Techniques for Sentence Completion

For the sentence compl eti on a few choi ces are gi ven that coul d be sel ected for
compl eti ng the sentences. Onl y one choi ce i s correct out of the several choi ces.
You have to compl ete the sentence by sel ecti ng the correct choi ce accordi ng to
the grammar or vocabul ary. For maki ng the ri ght choi ce you can benefi t from the
f ol l owi ng techni ques;

After you read the i ncompl ete sentence do not l ook at the choi ces. Try to
thi nk about the correct answer yoursel f. If you thi nk that you have
compl eted the sentence and found the correct choi ce you can consul t your
l i st of choi ces. If the answer you thought matches one of the choi ces
menti oned i n the l i st that i s most probabl y the ri ght choi ce to be marked.
If i t does not match wi th the choi ce you can l ook for a synonym
repl acement. Thi s tacti c i s very hel pful i n fi ndi ng the ri ght answer, i t
prevents you f rom conf usi ng yoursel f wi th the wrong choi ces.

Do not sel ect the choi ce hasti l y. Even i f you are sati sfi ed wi th your choi ce
try to substi tute i t wi th the other choi ces so that you are more sati sfi ed
wi th your deci si on. Someti mes the other choi ce fi ts more appropri atel y to
the sentence.

When you are asked to compl ete a sentence, whi ch has two spaces to be
fi l l ed i n, try to put the fi rst word of every choi ce i n the fi rst bl ank. Note
down the choi ce that you fi nd best. Now for the second bl ank try every
second choi ce of al l choi ces. Note the choi ce that you thi nk i s most
appropri ate. Check i f the two sel ected choi ces are matchi ng one of the
gi ven pai r of choi ces. If i t does then sel ect i t as your correct choi ce, i f not
then consi der thi s pai r as a wrong choi ce and try wi th the other choi ces.

If you fi nd di ffi cul ty i n maki ng sense out of certai n words and you are not
very fami l i ar wi th them you can try to make a guess wi th reference to the
Student Guide 65
context of the sentence. Try to break the word i nto vari ous parts and
anal yze i ts meani ng e. g. i f you do not know the meani ng of the word
ci vi l i zati on break i t i nto two i .e. ci vi l i ze and ati on now you may know
the meani ng of ci vi l i ze and through the term ati on you can make out that
the word i s a noun of ci vi l i ze. If you f i nd the word unf ami l i ar wi th pref i xes
and suf f i xes di vi de the word i nto i ts parts e.g. prerecordi ng. Thi s word
consi sts of both prefi x and suffi x. You can break the word l i ke pre -record-
i ng. Here you know that pre means before, record means to store and -ing
i s a term of conti nuous tense. So you can fi nd thi s break up of words qui te
hel pful i n maki ng out the ri ght sense. If none of the techni que works try
maki ng a guess wi th ref erence to the context.

When l ong and compl ex sentences confuse you then try to break that
sentence i nto smal l er more sentences by rephrasi ng i t. After you di vi de i t
compare wi th the ori gi nal sentence to avoi d any mi si nterpretati on. If you
are sati sfi ed read the smal l er sentences to get the i dea more cl earl y.

Example Questions

1. Mul tan ___________ a very hot cl i mate.
A. Has
B. Have
C. Has been
D. Wi th
2. One of the l east ef f ecti ve ways of sorti ng i nf ormati on i s l e arni ng
_________ i t.
A. Repeat
B. Repeati ng
C. To repeat
D. how repeat

3. Sal man fi ni shed__________ two of hi s publ i shed composi ti ons before
hi s twel fth bi rthday.
A. Wri tten
B. Wri ti ng
C. To wri te
D. Wrote

4. Sof i a __________ col l ect stamps, but now she has other i nterests.
A. Used t o
B. Was used t o
C. Used t o be
D. Usi ng to

5. After passi ng through a great trauma of her husband s death, she
__________ hard to achi eve mental rel axati on.
A. Struggl ed
B. Struggl i ng
C. Struggl e
D. To struggl e

6. In partnershi p wi th Paki stan, South Korea ____________on Motor way.
A. Hel ped worked
B. Hel pi ng work
C. Hel ped worki ng
D. To hel p worki ng

7. We wi l l wai t i f you __________ go.
A. Want ed t o
B. Want
C. Want t o
Student Guide 66
D. Wanti ng t o

8. If I had more ti me I ____________ checked my paper.
A. Woul d have
B. Woul d
C. Woul d had
D. Wi l l have

9. I thought that he___________ comi ng today.
A. Has been
B. Is
C. Was
D. Has

10. That prof essor enj oys teachi ng and ___________.
A. Wri ti ng
B. Wri tten
C. To wri te
D. Wri te

11. Just __________the f i l es on my tabl e.
A. Let
B. Leaves
C. St ay
D. Leave

12. Thank you f or __________ me your book.
A. Borrowi ng
B. Lendi ng
C. Borrowed
D. Had l ent

13. ____________ di scovery of i nsul i n, i t was not possi bl e to treat
di abetes.
A. Pri or
B. Bef ore to the
C. Pri or to the
D. To pri or the

14. Di stri bute the handouts ___________ the candi dates.
A. Between
B. Among
C. To
D. In

15. Onl y _________ were present at the semi nar.
A. a f ew peopl e
B. a l i ttl e peopl e
C. a f ew peopl es
D. the l i ttl e peopl e

Answer Key

1. A
2. C
3. B
4. A
5. A
6. C
7. C
8. A
9. C
10. A
11. D
12. B
13. C
14. B
15. A
Student Guide 67

4.1.2Analogy Questions

Anal ogy means si mi l ari ty i n exampl es or descri bi ng l i keness between two or more
words. These questi ons ask the reader to anal yze the rel ati onshi p between two
words i n a pai r and l ook f or another si mi l ar or equi val ent pai r of words. You are
provi ded wi th fi ve other pai rs of words. You are expected to match the ori gi nal
pai r, whi ch i s gi ven i n the questi on wi th one of the pai rs i n the gi ven choi ces on
the bases of si mi l a r rel ati onshi ps between them. Thi s exerci se or such questi ons
try to determi ne your basi c understandi ng towards vocabul ary and your abi l i ty to
recogni ze the rel ati onshi p between words. Some questi ons may al so ask you to
sel ect a sui tabl e antonym f or a given word.

Techni ques for Anal ogy Questi ons
For the anal ogy questi ons you can f ol l ow the gui del i nes menti oned bel ow;

Do not read the choi ces before you have anal yzed the rel ati onshi p between
the pai r of words, yoursel f. Try to understand the words more
appropri atel y and thi nk on whi ch basi s the rel ati onshi p between the words
i s f ormed. Af ter you reach a concl usi on read the gi ven choi ces af terwards
to get a proper match wi th another pai r havi ng the same rel ati onshi p.

When you f i nd yoursel f stuck wi th a word of di f f i cul t vocabul ary, do not f eel
confused. Try to understand i ts meani ng reference to the context or i f i t i s
somewhat fami l i ar try to remember where and when you heard the word
bef ore. It can be a great hel p.

Someti mes you f i nd that there i s more than one pai r that fi ts wel l to the
questi on and i s appropri ate for the choi ce, gi ve the ori gi nal pai r a l i ttl e
more thought so that you can further study the rel ati onshi p between the
words and narrow i t down to a more di sti nct one. Af ter you have be en
successful i n fi ndi ng a cl oser rel ati onshi p you can now scruti ni ze the two
other pai rs that confused you earl i er. Repeati ng the same procedure wi th
these words woul d prove usef ul .
Do not get caught up by the tri cks of the test makers. Someti mes the
questi ons are provi ded wi th very tri cky and dodgi ng choi ces that mi sgui de
greatl y. Try to thi nk of every choi ce more speci fi cal l y and narrowl y.

If you are f ami l i ar wi th the parts of speech and thei r nature, i t can be
benefi ci al i n maki ng a more sensi bl e choi ce. Remember i f the words i n the
ori gi nal pai r are a noun and an adj ecti ve, the correct choi ce you make
shoul d al so contai n the words i n the same grammati cal order. Otherwi se,
your choi ce i s wrong. So, i f you are confused wi th two pai rs and cannot
choose the correct choi ce you can easi l y l ook at thei r grammati cal order
and gi ve pref erence to the one, whi ch matches the ori gi nal one.

Excl ude the choi ce f rom your consi derati on that you thi nk i s
i ncorrect, e.g. the choi ces that do not have the same grammat i cal
uni t as of the ori gi nal pai r cannot match the ori gi nal pai r i n anyway.
Spend more ti me on consi deri ng the more possi bl e choi ces.

You shoul d know about the vari ous ki nds of anal ogi es that are more
frequentl y asked. Some of the common anal ogy types are as
f ol l ows;

i. Synonyms
Some words are l i nked together i n a pai r whi ch means the same or
has a si mi l ar di cti onary defi ni ti on.e. g Pret t y- Beauti ful

Student Guide 68
ii. Describing Qualities
Some pai rs have some words i n whi ch one word descri bes the other
word. Heavy- Rai n

iii. Cl ass and Member
Some pai rs have words whi ch are based on cl ass and member basi s
e. g. El ectroni cs- Radi o

iv. Antonyms
Some pai rs consi st of the words that are opposi te to each other
e.g.
Love- Hate

v. Descri bi ng Intensi ty
Some pai rs consi st of the words i n whi c h one descri bes the
i ntensi ty of the other e.g.
Smile- Laught er

vi. Function
In some pai rs a word descri bes the functi on of the other word e.g.
Pen- Wri te

vii. Manners
Some words i n a speech descri be the manners and behavi or e.g.
Pol i te- Speech

viii. Worker- Workplace
Some pai rs i n a word descri be the professi on and i ts workpl ace e.g.
Doctor- Clinic

Example Questions

1. HEIGHT: MOUNTAIN

(A) Depth : Trench
(B) Shade: Tree
(C) Wei ght : Age
(D) Speed: Hi ghway
(E) Mi neral : Mi ne

2. OBLIVIOUS : AWARENESS

(A) Comatose : Consci ousness
(B) Serene : Composure
(C) Erudi te : Knowl edge
(D) Adroi t : Ski l l
(F) Invi gorate : Energy

3. BELLWETHER : BAROMETER

(A) Prosel yte : Spark pl ug
(B) Panhandl er : Ki l l
(C) Embezzl er : Abduct
(D) Cynosure : Magnet
(F) Morass : Catal yst
Student Guide 69

4. ACT : ACTION

(A) Therapy : Thermometer
(B) Obl i vi on : Obvi ous
(C) Li turgy : Li terature
(D) Image : Imagi ne
(E) Bowl : Bowdl eri ze

5. BIBULOUS : DRINK

(A) Rapaci ous : Cl othi ng
( B) Gl uttonous : Food
(C) Al trui sti c : Money
(D) Vegetari an : Meat
(E) Controversy : Reconci l e

6. SONG : RECITAL

(A) Author : Bi bl i ography
(B) Epi sode : Seri es
(C) Coach : Team
(D) Dancer : Agi l e
(E) Poetry : Prose

7. HOUSE : BIG

(A) Home : Li ve
(B) School : Dai l y
(C) Water : Col d
(D) Cl othes : Socks



8. ANIMAL : MONKEY

(A) Zebra : Gi raffe
(B) Stati onery: Penci l
(C) Book : Cap
(D) Tree : Wood

9. HEAVEY : LIGHT

(A) Fat : Thin
(B) Stupi d : Idiot
(C) Rough : Surf ace
(D) Beauti ful : Diary

Choose the lettered word or phrase that is most nearly opposite in
meani ng to the word i n capi tal l etters.

10. DETER

(A) Twi st
(B) Inti mi date
Student Guide 70
(C) Encourage
(D) Strai ghten
(E) Suppl y

11. INDIGENOUS

(A) Modern
(B) Al i en
(C) Ornamental
(D) Natural
(E) Conceal

12. THERAPEUTIC

(A) Curati ve
(B) Injuri ous
(C) Medi ci nal
(D) Practi cal
(E) Sl ander

13. QUIZZICAL

(A) Amused
(B) Unequi vocal
(C) Contorted
(D) Di ssi mi l ar
(E) Lax

14. ANCILLARY

(A) Dependent
(B) Concomi tant
(C) Appendi x
(D) Pri mary
(E) Buffet

15. VIRTUOSO

(A) Wi cked
(B) Dazzl i ng
(C) Medi ocre
(D) Honorabl e
(E) Bawdy


Answer Key

1. A 4. D 7. C 10. C 13. B
2. A 5. B 8. B 11. B 14. D
3. A 6. B 9. A 12. B 15. E

4.1.3 Critical Reading Questions

Student Guide 71
Questi ons rel ated to cri ti cal readi ng try to j udge your readi ng ski l l s and how you
understand and i nterpret what you read. The paper i ncl udes a few passages that
ask answeri ng questi ons rel ated to the passage.


Techniques for Critical Reading Exercises

There are a few techni ques rel ated to the Cri ti cal Readi ng Questi ons that
prove to be a good gui del i ne for sol vi ng such questi ons.

Do not read the questi ons before readi ng the whol e passage. Try to
ski m through the whol e passage and then read the questi ons to l ook
for a more speci fi c answer. Read the passage qui ckl y wi th
understandi ng but do not pani c. Try to anal yze what the whol e
passage i s about and what the author real l y i ntends to convey.
Whi l e readi ng mark the l i nes where you thi nk the passage carri es the
most i mportant poi nts. These strategi es woul d defi ni tel y hel p you
fi nd the answers.

When you fi nd yoursel f stuck wi th a questi on, do not waste your ti me
on i t and go ahead for the next questi ons. Someti mes, answeri ng
other questi ons gui de you about the earl i er questi on. But, i f you sti l l
do not fi nd the answer mark i t for doi ng i n the end more cal ml y,
havi ng enough ti me to thi nk.

Try to f ami l i ari ze yourself wi th the types of cri ti cal readi ng
questi ons. Once you know the nature of such questi ons, you wi l l be
abl e to f i nd the answers more qui ckl y even when you are readi ng the
passage. The exampl es of some commonl y asked questi ons are as
f ol l ows:

o Central Idea
Mostl y, questi ons are asked to expl ai n the central i dea or mai n
theme of the whol e passage, whi ch anal yzes how you ski m
through i t. Someti mes, the openi ng and cl osi ng l i nes can gi ve you
a better cl ue about answeri ng such questi ons properl y.

o Specific Details
Someti mes to anal yze your scanni ng abi l i ti es you are asked to
answer some speci fi c detai l s about the passage. Such questi ons
are about when , where , whi ch and who . You can get the
answers of thi s ki nd of questi ons from the area of the passage
whi ch you marked i n the f i rst readi ng, where you thi nk the most
i mportant and i nformati onal remarks of the author l i es.

o Maki ng Inferences
Most of the questi ons ask you to i nfer from the passages, maki ng
your opi ni on about what i s sai d i n the paragraph, i mpl yi ng
meani ng and maki ng your own poi nt of vi ew. These questi ons try
to assess your judgment; you must be cl ear i n your mi nd about
what the author i s referri ng to and then make your own opi ni on
accordi ng to your understandi ng and comprehensi on. Read and
thi nk about al l the choi ces and anal yze each of i t l ogi cal l y
accordi ng to your comprehensi on rather than the author s poi nt of
vi ew.
Student Guide 72

o Meani ng i n Context
Some sel ected words from the passage are poi nted out to expl ai n
them wi th ref erence to the context to check your readi ng
comprehensi on. Someti mes the word that descri bes somethi ng i n
a di cti onary portrays i t the other way when i t appears i n the
context. The test tri es to judge your abi l i ty to make sense of the
word i n the context.

o Author s Approach
Some questi ons ask you to expl ai n the mood i n whi ch the author
i s wri ti ng whether i t i s sarcasti c, humorous, wi tty, sad etc. When
you are asked questi ons l i ke these you can l ook for certai n
expressi ons, words, phrases or excl amati ons, whi ch descri be the
tone, mood or styl e of the author. The feel i ngs of the wri ter are
mostl y exhi bi ted through choi ce of words. Whi l e answeri ng these
questi ons read the message caref ul l y observi ng parti cul arl y the
use of words.

o Ti tl e Sel ecti on
Some passages ask for sel ec ti ng a ti tl e that best sui ts the
passage. Remember that the chosen ti tl e shoul d not be narrowl y
or broadl y sel ected. Try to avoi d choosi ng those ti tl es that
descri bes onl y one or two paragraphs but the one, whi ch i s
appl i cabl e to the whol e passage and portrays i t best.
Example Questions
Passage I:
We are profoundl y i gnorant about the ori gi ns of l anguage and have to
content oursel ves wi th more or l ess pl ausi bl e specul ati ons. We do not
even know for certai n when l anguage arose, but i t seems l i kel y that i t goes
back to the earl i est hi story of man, perhaps hal f a mi l l i on years. We have
no di rect evi dence, but i t seems probabl e that speech arose at the same
ti me as tool maki ng and the earl i est forms of speci fi cal l y human
cooperati on. In the great Ice Ages of the Pl ei stocene peri od, our earl i est
human ancestors establ i shed the Ol d Stone Age cul ture; they made fl i nt
tool s and l ater tool s of bone, i vory, and antl er; they made fi re and cooked
thei r food; they hunted bi g game, often by methods that cal l ed for
c onsi derabl e cooperati on and coordi nati on. As thei r materi al cul ture
gradual l y i mproved, they became arti sts and made carvi ngs and engravi ngs
on bones and pebbl es, and wonderf ul pai nti ngs of ani mal s on the wal l s of
caves. It i s di ffi cul t to bel i eve that the makers of these Pal eol i thi c cul tures
l acked the power of speech. It i s a l ong step Admi ttedl y, f rom the earl i est
f l i nt weapons to the spl endi d art of the l ate Ol d Stone Age: the f i rst crude
fl i nts date back perhaps to 500,000 B.C., whi l e the fi nest ac hi evements of
Ol d Stone Age man are l ater than 100,000 B.C.; and, i n thi s peri od, we can
envi sage a correspondi ng devel opment of l anguage, from the most
pri mi ti ve and l i mi ted l anguage of the earl i est human groups to a f ul l y
devel oped l anguage i n the fl oweri ng ti me of Ol d Stone Age cul ture.

How di d l anguage ari se i n the fi rst pl ace? There are many theori es about
thi s, based on vari ous types of i ndi rect evi dence, such as the l anguage of
chi l dren, the l anguage of pri mi ti ve soci eti es, the ki nds of changes that
have taken pl ace i n l anguages i n the course of recorded hi story, the
Student Guide 73
behavi or of hi gher ani mal s l i ke chi mpanzees, and the behavi or of peopl e
sufferi ng from speech defects. These types of evi dence may provi de us
wi th useful poi nters, but they al l suffer f rom l i mi tati ons, and must be
treated wi th cauti on. When we consi der the l anguage of chi l dren, we have
to remember that thei r si tuati ons are qui te di fferent from that of our
earl i est human ancestors, because the chi l d i s growi ng up i n an
envi ronment where there i s al ready a f ul l y devel oped l anguage, and i s
surrounded by adul ts who use that l anguage and are teachi ng i t to hi m.
For exampl e, i t has been shown that the earl i est words used by chi l dren
are mai nl y the names of thi ngs and peopl e (Dol l , Spoon, Mummy):
but, thi s does not prove that the earl i est words of pri mi ti ve man were al so
the names of thi ngs and peopl e. When the chi l d l earns the name of an
object, he may then use i t to express hi s wi shes or demands: Dol l !: often
means Gi ve me my dol l ! Or I ve dropped my dol l : pi ck i t up for me!;
the chi l d i s usi ng l anguage to get thi ngs done, and i t i s al most an acci dent
of adul t teachi ng that the words used to f ormul ate the chi l d s demands are
mai nl y nouns, i nstead of words l i ke Bri ng! Pi ck up!; and so on.

1 The mai n i dea of thi s excerpt i s
A. to provi de evi dence of the ori gi n of l anguage.
B. to present the need for l anguage.
C. to di scuss how earl y man communi cated.
D. to present the cul ture of earl y man.
E. to narrate the story of Engl i sh.
F.
2 Theori es of the ori gi n of l anguage i nc l ude al l of the fol l owi ng
EXCEPT
A. Changes occurri ng through the years.
B. The need to communi cate.
C. Language of chi l dren.
D. The fi rst man s extensi ve vocabul ary.
E. Communi cati on among pri mi ti ve men.

3 The purpose of the di scussi on of the word, Dol l , i s i ntended to
A. Trace the evol uti on of a noun.
B. Support the fact that nami ng thi ngs i s most i mportant.
C. Indi cate how adul ts teach l anguage to chi l dren.
D. Show the evol uti on of many meani ngs for one word.
E. Evi nce man s mul ti pl e uses of si ngl e words

4 The implication of the author regarding the early elements of
l anguage i s that
A. There were speci fi c real steps fol l owed to devel op our l anguage.
B. Care must be exerci sed when exhumi ng what we consi der the roots
of l anguage.
C. We owe a debt of grati tude to the chi mpanzee contri buti on.
D. Adul ts created l anguage i n order to i nstruct thei r chi l dren.
E. Language was ful l y devel oped by pri mi ti ve man.

5 If we accept that primitive man existed for a very long period of
ti me wi thout l anguage, then we may assume that
A. Language i s not necessary to man s exi stence.
B. Language devel oped wi th the devel opi ng cul ture of pri mi ti ves.
C. Pri mi ti ves exi sted i n total i sol ati on from one another.
Student Guide 74
D. Chi l dren brought about a need for l anguage.
E. Manki nd was not i ntended to communi cate.

6 After a readi ng of thi s arti cl e, one mi ght i nfer that
A. Soci ety creates probl ems wi th l anguage.
B. Language i s f or adul ts to i nstruct chi l dren.
C. Soci ety uses l anguage to i mprove i tsel f.
D. Wi th the evol uti on of l anguage came wi sdom.
E. Language bri ngs power.

Answer Key

1. A 2. D 3.C 4. B 5. B 6. E

Student Guide 75








COMMON QUERIES
Student Guide 76

5 About the General Test
Q What i s the General Test generally about?

The NTS General Test i s general l y di vi ded i nto three secti ons i .e. The
Verbal Secti on, Quanti tati ve Secti on and the secti on of Anal yti cal
Reasoni ng. The durati on of these tests i s 120 mi nutes whi ch you can
yoursel f manage f or each secti on, as there i s no speci f i c ti me di stri buti on
for any.


The fol l owi ng ti me tabl e i nforms of the number of questi ons i n each secti on
as wel l as the marks gi ven for that secti on:

Test Sections
No of
Questions
Marks
Verbal 25 25
Analytical Reasoning 35 35
Quantitative 40 40

Q What Ki nd of Questi ons are asked?

The nature of the test i s objecti ve type, each secti on i s based on Mul ti ple
Choi ce Questi ons (MCQs), and every questi on wi l l have at l east two choi ces
or fi ve choi ces at the most.

Q What are the Contents of the Test?

The General Test i s general l y di vi ded i nto three secti ons i .e. The Verbal,
Quanti tati ve and Anal yti cal Reasoni ng. Al l the questi ons are based on
Mul ti pl e- Choi ce Questi on format.

The Verbal secti on of the test i s based on compl eti ng sentences wi th the
appropri ate words, comprehensi on exerci ses to assess the basi c knowl edge
about the vocabul ary and grammar of the l a nguage. Questi ons basi cal l y
about sentence compl eti on, anal ogy, f i ndi ng antonyms, and cri ti cal readi ng
are asked.

In the Quantitative Section the NTS tri es to measure your basi c
mathemati cal ski l l s and check your understandi ng of the el ementary l evel
mathemati cs. The MCQs i n thi s secti on are based on areas l i ke Ari thmeti c,
Al gebra and Geometry, data anal ysi s etc.

In the Analytical Reasoning Section vari ous obj ecti ve type questi ons are
asked to anal yze the capabi l i ty of every student wi th regard to t hei r
probl em sol vi ng apti tude i n thei r dai l y l i fe acti vi ti es and al so the academi c
i nterests. Questi ons asked are basi cal l y on Anal yti cal Reasoni ng and
Logi cal Reasoni ng.

Q From Where Can I Get Hel p to Prepare for the Test?

Student Guide 77
NTS does not refer to one spec i fi c book for consul tati on duri ng the test
preparati on. However, the candi dates can go through this Student Gui de,
publ i shed by NTS, whi ch i s easi l y avai l abl e i n the market. It contai ns
sampl e questi on papers as wel l as exampl es of the types of questi ons you
may fi nd i n each segment . You can al so practi ce by sol vi ng the sampl e
tests and the other questi ons f ound i n this gui de and the NTS websi te. It
woul d hel p you i n becomi ng f ami l i ar wi th the nature of the test. Try
practi ci ng the tests by keepi ng the al l otted ti me i n mi nd. Gi ve yoursel f an
average of one mi nute to compl ete each questi on. It woul d hel p you pace
yoursel f through out the test. Do not l eave any questi on unsol ved. For
f urther measures you can consul t materi al s easi l y avai l abl e at any good
bookstore for TOEFL, SAT, GRE et c.

Q What are the General Gui del i nes to Attempt the Test?

This Student Gui de by the NTS i s the ri ght source to be consul ted. The
sampl e questi on papers f or any of the three areas of study are a great hel p
for you. The t est is composed on the same test pattern and the questi ons
are sel ected on the same l evel . You may al so use addi ti onal books for
your preparati on. If you answer 75% of the questi ons correctl y, you wi l l
recei ve an excel l ent score.

Q Can I Get My Registrat i on Cancel l ed?

If you cannot take the test and want to cancel your regi strati on, you
shoul d i nform the NTS Offi ce i n wri ti ng by sendi ng an appl i cati on or emai l
at l east one week before the test date.
Q How Can I Get My Test Schedul e or Test Centre Changed?

Once you have been al l otted a Test Center and the ti me of the test, no
changes can be made possi bl e to that schedul e. However, you can ask NTS
by emai l and NTS may try to hel p you, i f i t fi nds i t admi ni strati vel y
feasi bl e.

Q Can I Appear for the Test More Than One Ti me?

Yes, i f you want to i mprove your score and ranki ng, you can reappear i n
the test whenever the new schedul e i s announced. However you have to
regi ster agai n wi th ful l fee payment. You al so have to wai t when the next
schedul e of the test i s announced.


Q From Where Can I Get the Registration Forms for the General
Test (NAT)?

For the paper based tests, you can get a free of charge NTS Regi strati on
Form, encl osed i n i ts brochure, whi ch i s made avai l abl e i n the NTS al l i ed
bank branches, the Front Of f i ce or Recepti on of the NTS or the al l i ed
i nsti tutes of NTS. However, pl ease ensure that you get these before the
l ast date of regi strati on.

Q How Can I Regi ster for NTS General Test ?

After recei vi ng the form, fi l l i t appropri atel y. Submi t i t at the NTS Al l i ed
Insti tute whi l e submi tti ng the NTS Regi strati on Forms NTS Test Fee i s
Student Guide 78
charged by the NTS Bank Branches / Al l i ed Insti tutes accordi ng to the
announced speci fi cati ons.

Fi l l the form accordi ng to the i nstructi ons gi ven at the back of it . Submi t i t
by payi ng the Test Fee i n any branch of NTS al l i ed
bank/ insti tutes/organi zati ons/NTS by emai l or post . You get a Test Entry
Coupon from the bank/i nsti tute, whi ch shows you your Rol l Number on i t.

Q How Shal l I Fi l l the Forms?

A few thi ngs must be clear to you before you fi l l i n the Form. Any
i ncompl ete or ambi guous i nformati on i n the form may cause del ay i n the
regi strati on process and you may not be abl e to take the test. For f i l l i ng i n
the forms correctl y i t i s necessary to;

1. Use bl ack i nk and BLOCK LETTERS to fi l l i n the form.
2. Wri te the appropri ate test ti tl e, for whi ch you want to regi ster i .e.
General Test

3. Be careful whi l e spel l i ng your name.
4. Wri te the mai l i ng address correctl y where you can be contacted i n
future.
5. Ensure that the Test Fee is submi tted i n the bank/Insti tute through
Chal l an Form attached wi th the Regi strati on Form. Af ter
submi tti ng the fee, one copy of the Chal l an Form must be attached
t o thi s Regi strati on Form and you wi l l need to keep the other for
yoursel f . Do not forget to fi l l i n the bank Chal l an detai l s i n the
appropri ate col umns. If the Chal l an Form i s not attached wi th your
f orm, the Regi strati on Form wi l l not be accepted and no f urther
processi ng wi l l be made on your request for regi strati on.
6. Al so i ndi cate your choi ce of ci ty, where you want your test center to
be. Your group and Test Center i nformati on wi l l be i nti mated to you
wel l before the test day.
7. Speci fy the i nsti tute that you want your test score to be f orwarded
t o. You can get the l i st of Al l i ed Insti tutes from the NTS websi te.
8. If handed over to the same bank where you have submi tted the fee.
The bank wi l l gi ve you one part of thi s f orm as a Test Entry Coupon.
You have to show thi s coupon at the ti me of test. Wi thout thi s , you
may not be al l owed to enter the Test Center.


NOTE: These instructions are also given on the back of the Registration Form. The
General Test for some allied institutes/organizations may not need NTS Registration
Form.

Q How Can I Pay the Registration Fee for the Paper-based
Testing?

You can submi t the Regi strati on Fee through a Bank Voucher i n an NTS
Authori zed Bank al ong wi th your Regi strati on Form. However the f ee and
fee deposi ti on procedure i s subject to change from ti me to ti me. The exact
parti cul ars are communi cated on the announcement of the test schedul e.
On occasi ons the Test Fee can al so be submi tted at the i nsti tutes where
the test i s bei ng conducted.

Student Guide 79
Q How to Submi t the Form for the Paper-based Testi ng?

After you have compl eted the f orm accordi ng to the i nstructi ons, get i t
submi tted at the same branch of the bank/i nsti tute/organi zati on f rom
where you col l ected your regi strati on f orm. Do not forget to col l ect a
stamped and photographed Test Entry Coupon from the bank, whi ch
ensures your i denti f i cati on on the Test Day. Af ter recei vi ng the f orms, the
NTS Head offi ce checks the essenti al requi rements for NTS regi strati on i .e.

i Three copi es of the recent photographs
ii Candi date' s name cl earl y wri tten i n the bl ock l etters
iii Candi date' s f athers cl earl y wri tten name i n bl ock l etters.
iv Candi dates postal address cl earl y wri tten i n bl ock l etters
v Phone number cl earl y wri tten et c.

If any essenti al requi rement i s mi ssi ng f r om the Regi strati on Form, i t i s
cl earl y marked as rejected.

Note: The Regi strati on Forms are at ti mes di fferentl y pri nted for di fferent
i nsti tutes/organi zati ons. The i nstructi ons f or those f orms are avai l abl e at
the back of the Regi strati on Form.

Q How does NTS Confirm My Registration for the Paper-based
Testing?

Af ter submi tti ng the Regi strati on Form at the bank/ i nsti tute/organi zati on,
you get your Test Entry Coupon, whi ch i ndi cates your Rol l Number and al so
serves as your Identi fi cati on Sl i p. Thi s sl i p confi rms your regi strati on.

In case of any confusi on or probl ems wi th the regi strati on process you may
contact NTS.

Q May I Register on the Day of Examination f or the General
Test?

The regi strati on of candi dates on the day of t he NTS Exami nati on cannot
be entertai ned. The candi date shoul d have got oneset regi stered before
the deadl i ne, announced by NTS for the cl ose of the regi strati on process.

Q How Can I Get Help in Case of Any Problem During the
Regi strati on Process?

If you have any confusi on about the regi strati on process you can cl ari fy
thi s by cal l i ng the NTS Offi ce or by sendi ng an emai l .

Q How Can I Get the Test Schedule for the Test ?

If you have any conf usi on about the Test Schedul e you can obt ai n detai l s
from the news medi a as wel l as NTS websi te.

Q How Can I Get My Test Entry Coupon for the Paper-based
Testing?

Student Guide 80
You are gi ven your Test Entry Coupon when you submi t your form. It
serves as your Identi fi cati on Sl i p and al so i nforms you about the l ocati on
of the center, Rol l Number and other detai l s.

Q What do I have to do wi th the Test Entry Coupon?

Take the Test Entry Coupon wi th you on the Test Day. At the Test Center
you are asked to show the Test Entry Coupon that was gi ven to you af ter
you submi tted your form there. The staff at the Test Center i denti fi es you
wi th the photograph attached on the Coupon. Remember to attach a
photograph, whi ch i s not more than si x months ol d. If you f ai l to show
your Test Entry Coupon to the admi ni strati on on the test day or i f your
photograph does not match your appearance, you may not be al l owed to
enter the Test Center.

Q What Should I Take With Me to the Test Center for the Paper-
based Testing?

You are requi red to bri ng your own sharpened penci l s and eraser. Besi des
this, the candi date i s al so asked to show hi s/her Test Entry Coupon.

Q What Things Should I not bri ng wi th me i n the Test Center for
Paper-based Testi ng?

Cal cul ators, cel l phones, books, bookl ets, any ki nd of paper or cheati ng
materi al are stri ctl y prohi bi ted i nsi de of the Test Center. If any of these
i tems are found to be i n your possessi on duri ng the test you wi l l be
di squal i fi ed f rom the exami nati on.

Q What are the Rules and Regulations that are appl i ed to me
inside the Test Center for the Paper-based Testing?

The test i s t aken on the day and at the ti me schedul ed. You are asked to
observe punctual i ty.
1. You are assi gned a seat by the i nvi gi l ati on staff.
2. Testi ng ai ds are not permi tted to be taken al ong to the Test Center.
Items l i ke cal cul ators, pagers, cel l phones, headphones, and any
paper or wri ti ng materi al , di cti onari es etc are prohi bi ted i n the test
center.
3. You are not al l owed to eat, dri nk, or use tobacco duri ng the test.
4. If you do not show the Test Entry Coupon and your photograph does
not match, you wi l l not be permi tted to take the test.
5. No di scussi on or communi cati on wi th your fel l ow candi dates i s
al l owed duri ng the test sessi on.
6. You are not al l owed to l eave the test center wi thout the permi ssi on
of the supervi sor.
7. Under no ci rcumstances can the test questi ons or any part of a test
be removed, reproduced or di scl osed verbal l y, el ectroni cal l y, or
physi cal l y or by any other means to any person or enti ty.
8. Test centers do not have l arge wai ti ng areas. Fri ends or rel ati ves
who accompany you to the test center are not permi tted to wai t i n
the near vi ci ni ty of the test center or contact you whi l e you are
taki ng the test.
Student Guide 81
9. If you need to l eave your seat at any ti me duri ng the test, rai se
your hand and ask the i nvi gi l ator. Ti mi ng of the secti on does not
stop.
10. Repeated unschedul ed breaks wi l l be documented and reported to
NTS.
11. NTS reserves the ri ght to vi deotape any or al l of the testi ng
sessi ons.
12. Dress i n such a way that you can adapt to any room temperature.
13. You cannot communi cate wi th anyone (other than the test center
staff) whi l e the test sessi on i s i n progress or even duri ng the
breaks.
14. You are not permi tted to l eave the test center vi ci ni ty duri ng the
test sessi on or breaks.
15. If at any ti me duri ng the test you bel i eve that you have a probl em
wi th your test, or need the Invi gi l ati on Staff for any reason, rai se
your hand to noti fy the Invi gi l ati on Staff.

Q On What Basi s Can I be di squa l i fi ed from the Test?

If you are f ound i ndul ged i n any of the f ol l owi ng acti vi ti es, you wi l l be
di squal i f i ed f rom the test:

1. Arri vi ng l ate t o the test center.
2. Taki ng the test for someone el se.
3. Fai l i ng to provi de the Test Entry Coupon.
4. Usi ng any objecti onabl e materi al duri ng the test.
5. Tryi ng to create a di sturbance i nsi de the test center.
6. Showi ng i l l behavi or towards the i nvi gi l ati on staff.
7. Tryi ng to communi cate wi th f el l ow candi dates whi l e the test i s i n
progress.
8. Tryi ng to tear pages from the questi on paper.
9. Showi ng i ndi sci pl i ne i .e. eati ng, smoki ng etc.
10. Leavi ng the test center wi thout permi ssi on.

Q How Shal l I Mark the Answer Sheet?

1. Fi l l i n the parti cul ars careful l y l i ke your Name, Father s Name,
Enrol ment Number and the Test Name i .e. General Test and other
parti cul ars as menti oned on the answer sheet.
2. Read the di recti ons careful l y before you begi n your test.
3. The i nvi gi l ator announces the procedure f or marki ng the answer
sheets. Li sten careful l y to the i nstructi ons. When you mark the
answer sheets, make sure that the space i s compl etel y fi l l ed i n. You
can change the answer, by erasi ng the earl i er choi ce neatl y.
4. If a questi on i s found wi th no answer or i t has mo re than one
marked choi ce, i t wi l l not be consi dered for marki ng.
5. Compl etel y fi l l i n the corr espondi ng ci rcl es on the answer sheet
usi ng a l ead penci l . Do not use i nk, bal l- pen or any other marki ng
materi al to mark the oval .
6. Fi l l i ng more than one oval wi l l render your answer i ncorrect.
7. Pl ease fi l l i n the oval compl etel y and make no other marks on i t.

Incorrect

Student Guide 82
Correct

Q What are the Other Test Taking Strategies for the General
Test?

1. The test i s di vi ded i nto three secti ons i .e. Verbal , Anal yti cal
Reasoni ng and Quanti tati ve. The di recti ons gi ven at the begi nni ng
of every secti on wi l l tel l you the total number of questi ons and the
recommended ti me.
2. You are provi ded wi th a few bl ank pages i n the test paper. You can
use them for your rough work.
3. Some f ormul as and hel pi ng tabl es are gi ven at the end of the
questi on paper. You can take hel p f rom those f or sol vi ng the
questi ons.
4. Keep a very caref ul track of ti me.
5. Do not pani c whi l e sol vi ng your paper. Gi ve each secti on a speci fi c
ti me peri od and try to compl ete every secti on on ti me.
6. If you fi nd di ffi cul ty wi th some questi ons, do not waste your ti me by
thi nki ng about them for a l ong ti me. Try fi ni shi ng the easi er
questi ons fi rst and you can go back and f i ni sh the di f f i cul t questi ons
l at er.
7. Do not l eave any questi on unanswered, if you are conf used or have
no i dea about a certai n questi on, try maki ng a guess for the ri ght
answer.
8. Do not make any extra marks on the answer sheet. Computer mi ght
regi ster these and consi der them as another marked choi ce.
Q If I am di squal i fi ed from the Test, Can I apply for it at a Later
Ti me?

If you are di smi ssed f rom the test sessi on, you wi l l be gi ven a second
chance and wi l l be al l owed to reappear i n the next testi ng sessi on.

Q How can I Ask for Resul t Reporti ng?

You can ask for your resul t by requesti ng i t from NTS, through a phone
cal l , fax or by sendi ng an e- mail. You can al so vi si t the web si te of NTS to
check your resul t s. The resul t s wi l l be sent to you by post on request. A
proper resul t woul d al so be prepared for the i nsti tutes and the candi dates
can check up thei r perf ormances by contacti ng thei r respecti ve i nsti tute.

Q How i s My NTS Test Scored?

The papers are marked accordi ng to the i nternati onal marki ng standards.
Each questi on carri es one mark. Every questi on whi ch i s correctl y
answered wi l l be gi ven one mark unl ess the computer fi nds an error wi th
your marki ng.

Q Is There Any Negati ve Marki ng Whi l e Scori ng?

If the answer i s wrong you wi l l l ose one mark but there i s no negati ve
marki ng for an i ncorrect answer.

Q If I have al ready appeared in the Test, How Will the First
Score Affect My New Score?
Student Guide 83

If the candi date has appeared more than two ti mes i n an NTS test, hi s/her
hi ghest score woul d be consi dered to be val i d.

Q Is There Any Chance to Improve My Score?

If you are not sati sfi ed wi th your test score and you want to i mprove your
grade you wi l l be al l owed to appl y and appear i n the test agai n.

Q What Should I do, i f I Regi ster and Then Can Not Take the
Test?

If you do not appear to take the test you wi l l be gi ven another chance to
take the test on the next schedul ed test , but no refunds can be made.

Q Who Shal l I Contact i n Case of any Confusi on?

If you have any confusi ons about the testi ng servi ce, regi strati on, test
executi on process and the announcement of resul ts etc you can cal l , mai l
or fax the NTS Head offi ce for the detai l s.

Queries for Online General Test

Q. In What Ways is NTS General Test Different from Other NTS Tests?

The NTS General Tests are different from its other tests in many ways. Unlike the NTS
Subject tests, the General Tests aims to assess the verbal, quantitative and analytical
abilities that have been attained over a period of time and that are not necessarily
related to any specific field of study.


Q. How Can I Register for NTS-General Tests?

For the NTS, General Test Registration, you have to fill the NTS Online Registration
Form on the NTS website. Before you get registered online you have to pay the
registration fee as per the specified procedure for that test. You can fill the form online
at our present website and submit it. You will get e-mail from NTS after the submission
of the form, which indicates that your form has been received by NTS. NTS will confirm
your registration by sending you an email once the bank confirms the receipt of your
fee. This form is available when the NTS has announced its online test schedule.

Q. What happens when I submit my Online Registration Form?

After you submit your Online Registration Form you receive an email from NTS which
confirms the receipt of your registration request. NTS takes 3-5 days time to check up
the valid Bank Challan No from the bank. When NTS confirms the deposition of your fee
from the bank, it sends you the Admission Ticket which gives you your User ID, location
of the Test Center, Test Time etc.

Q. When do I receive confirmation of my final Registration Status?

After submitting your Online Registration Form, you receive the confirmation of your
Test Registration and some other information about the test in the form of an Admission
Ticket within 3-5 days, through an email by NTS. If your registration is not confirmed
Student Guide 84
during the given time you can Contact Us for inquiring about the reasons.

Q. How Can I Pay the Registration Fee?

For the NTS General Test, the Test Fee is to be deposited in the prescribed Bank
Branches or a draft to be sent to NTS or the allied institute as prescribed. The reference
of your Bank Deposit Slip Number (that you received from the bank after depositing the
fee) is necessary to be endorsed on your Online Registration Form for getting your
registration confirmed.

Q. Will NTS Confirm my Registration?

Yes, after receiving a completely and properly filled NTS Online Registration form, NTS
confirms your registration by sending you an email as soon as the bank confirms the
receipt of your registration.

Q. May I Register on the Day of Online Test?

No, for the NTS Online General Test, you have to get registered by the due date,
intimated on the NTS website or advertised through various newspapers prior to the
test.

Q. Where Shall I Contact if I Do Not Get my Registration Confirmed?

If NTS does not confirm your registration within a weeks time after you submit your
form, you may contact support@nts.org.pk by giving the reference of your Bank Deposit
Receipt Number and your other personal particulars.

Q. Can I Get My Registration Cancelled?

If you cannot take the test and want to cancel your registration, you should inform the
NTS Office in writing by sending an application or email at least one week before the
test date.

Q. How Can I Get the Test Schedule?

The Test Schedule that includes the Test Date, Test Time and the Venue of the test, is
sent to you by NTS through an email.

Q. How Can I Get My Test Schedule or Test Centre Changed?

Once you have been allotted a Test Center and the time of the test, no changes might
be possible to that schedule. However, you can ask NTS by email and NTS may try to
help you, if it finds feasible administratively.

Q. Can I Appear for the Test More Than One Time?

Yes, if you want to improve your score and ranking, you can reappear in the test
whenever the new schedule is announced. However you have to register again with full
fee payment. You also have to wait when the next schedule of the test is announced.


Q. Is There Any Chance to Improve My Score?

If you want to improve your score and ranking, you are allowed to apply and appear in
Student Guide 85
the test again. However, you have to register again, once the schedule is announced.

Q. What Computer Skills are Necessary for NTS Online General Tests?

For the NTS computer-based online General Tests, minimal computer skills are required.
The skills that are required include, using a mouse, entering and confirming a response,
changing a response, accessing the help function, and moving to the next question
although a help function is available throughout the test. These Test Instructions will
also be given to you before the test is given.

Q. How Often is the NTS Online General Test Conducted?

A series of computer-based General Tests is planned. NTS announces the
Schedule from time to time on its website www.nts.org.pk and through newspaper
advertisements.

Q. When Should I Arrive at the Test Center?

Try to arrive at the testing center at least 45 minutes before your scheduled Test Time
so that no delays are made during the check-in procedures and your identification.

Q. What Should I Take With Me to the Test Center?

Bring your ID card with you for your identification. No other items e.g. stationary,
books, calculators are required nor allowed to be taken inside the Test Centre.

Q. How Long Will I be at the Test Center?

Although the actual test takes 3 hours, you should be prepared to spend up to 4 hours
at the center. You will spend extra time on taking Test Instructions, getting seated, and
other administrative arrangements, etc.

Q. What are the important Things to be careful about for Online General Tests?

You should be particularly careful about a few things while taking the Online General
Test:
Online testing does not allow you to come back and reattempt the question that you
have already marked.
If you face any difficulty in logging on to your system or face any other related
problem you can ask for help from any lab assistant or invigilator.




Q. How to take NTS Online Test?

You will go through the following screen shots to learn how to take NTS Online test.

Step: 1


This is the first screen named as Candidate Login Screen. You will enter your Candidate ID and
Password provided to you by NTS
Student Guide 86



Student Guide 87
Step: 2
You will see Test Instructions Screen after you successfully login. Please read the instructions
carefully to avoid any confusion during the test. After reading the instructions, press Start Test
button on center bottom of the page.


Student Guide 88
Step: 3
After you click on the Start Test button your test starts and a page appears which shows your first
question of the test. Each question has various choices, if you know the answer you can select the
correct choice for your answer and press Next Question button. You can also add this question
in the Pass Box to answer it at some another stage. You will also find some additional information
about your test on this page. This information includes:

Total Number of Questions in the Test
Total Number of Questions Answered by you
Total Number of Questions in the Pass box to be attempted later



Student Guide 89
Step: 4
If you place a question in Pass Box you will notice that the Questions in Pass Box field will
increase by one.




Student Guide 90
Step: 5
If you want to answer the questions in Pass Box, simply click on Questions in Pass Box link at
the top. It will take you on the following screen. Now click Answer this Question button for the
question you want to answer.




Student Guide 91
Step: 6
You have selected this question from Pass Box now select its answer and proceed for next
question.


Student Guide 92
Step: 7
You have attempted all questions and now this last screen will show your detailed result.


Wish You Good Luck with your Test.
National Testing Service



Q. How is NTS Online Test different from paper-based Test?

For the Paper-based Test,

A pencil, eraser and a sharpener is required to attempt the paper-based test.
You can make the changes in the answer that you have already marked.
The announcement of results is delayed than the online tests.
You have to fill in a separate answer sheet to mark your answers.

Where as for the Online Test,

No stationery items are required to attempt online test.
If you want to attempt a question at the end, you can place it in the Pass Box for
attempting later.
You can not reattempt a question that you have already answered.
As soon as you finish the test, the result is displayed on the screen.
In case of a system failure during the test, you will have to log-in again and the
test will start from the same question where you had left. No information will be
lost while the system was unavailable.

Student Guide 93
Q. How Can I Ask for Result Reporting?

Your result is reported to you right after you finish your test. You are given the
certificate fifteen days after the conduct of the test of the last batch. If you still do not
get the result you can Contact Us. You can also visit the website of NTS to check your
result. The final result is sent to you by email.

Q. Is There Any Negative Marking While Scoring?

There is no negative marking for wrong answers. However the negative marking may be
activated if it is required by the allied institute or organization.


Q. What are the Rules and Regulations that apply to me in the Test Center
when taking the Online General Test?

Rules and Regulations:
If you do not appear with the Identity Card (NIC) on the Test Center, you will not be
allowed to take the test.
The test will be given on the day and at the scheduled time. You are asked to observe
punctuality. Arriving late at the center may disqualify you from taking the test.
You are not allowed to bring any testing aids inside the test center. Nothing, except
the original Identity Card is required to be taken along.
You are not allowed to smoke, eat or drink inside the test center.
No discussion or any form of communication with the fellow candidates is allowed
during the testing session.

You will also not be allowed to leave the test center without the permission of the
supervisor.
Test centers do not have large waiting areas. Friends or relatives who accompany you
to the test center will not be permitted to wait in the test center or contact you while
you are taking the test.
You will be required to sign the attendance sheet before and after the test session and
any time you leave or enter the premises where the test is being conducted.
If you need to leave your seat at any time during the test (which shall only be allowed
in case of serious illness), raise your hand and ask the invigilator.
Repeated unscheduled breaks will be documented and reported to NTS. NTS reserves
the right to videotape all or any of Testing Sessions and use it to determine any
misconduct, etc.
If at any time during the test you believe that you have a problem with your test, or
need the Invigilation Staff for any reason, raise your hand to notify the Invigilation Staff.
Student Guide 94
NOTE: The rest of the queries regarding the test format, contents and other
procedures have almost the same answers as of the paper based tests, given
above.
Student Guide 95









DRILL TESTS
General
















Note: The sample papers do not include quantitatively the same number
of questions as there would be in the actual papers. They are merely
meant to provide conceptual guidance to the users or prospective
candidates.
Student Guide 96









Drill Test I
Student Guide 97


1. The number of degrees through which the hour hand of a clock moves in 2 hours and
12 minutes is

A. 66
B. 72
C. 732
D. 723
E. None of these



2. A cylindrical container has a diameter of 14 inches and a height of 6 inches. Since
one gallon equals 231 cubic inches, the capacity of the tank is approximately

A. 2-2/7 gallons
B. 4 gallons
C. 1-1/7 gallons
D. 2-2/7 gallons
E. None of these



3. A train running between two towns arrives at its destination 10 minutes late when it
goes 40 miles per hour and 16 minutes late when it goes 30 miles per hour. The
distance between the two towns is

A. 720 miles
B. 12 miles
C. 8-6/7 miles
D. 12-7/7 miles
E. None of these


4. If the base of a rectangle is increased by 30% and the altitude is decreased by 20%
the area is increased by

A. 25%
B. 10%
C. 5%
D. 1%
E. 4%


5. If the sum of the edges of a cube is 48 inches, the volume of the cube is

A. 512 inches
B. 96 cubic inches
C. 64 cubic inches

I
Quantitative
Section
No. Of
Questions
10
Choose the correct answer for each
question and shade the
corresponding CIRCLE on the
answer sheet
A
B C D E
A B C D E
A B C D E

A B C D E

Student Guide 98
D. 698 cubic inches
E. None of these




6. A certain triangle has sides, which are, respectively, 6 inches, 8 inches, and 10
inches long. A rectangle equal in area to that of the triangle has a width of 3 inches. The
Perimeter of the rectangle, expressed in inches, is

A. 11
B. 16
C. 22
D. 23
E. 24



7. The cube of 1/3 is

A. 3/9
B. 3/27
C. 1/81
D. 1/27
E. 1/9




8. In general, the sum of the squares of two numbers is greater than twice the product
of the numbers. The pair of numbers for which this generalization is not valid is

A. 8,9
B. 9,9
C. 9,10
D. 9,8
E. 8,10



9. A piece of wire 132 inches long is bent successively in the shape of an equilateral
triangle, a square, a regular hexagon, a circle. The plane surface of the largest area is
included when the wire is bent into the shape of a

A. Circle
B. Square
C. Hexagon
D. Triangle
E. Line



10. If pencils are bought at 35 cents per dozen and sold at 3 for 10 cents the total
profit on 5 1/2 dozen is

A B C D E

A B C D E

A B C D E

A B C D E
A B C D E

Student Guide 99
A. 25 cents
B. 35 cents
C. 27 1/2 cents
D. 28 1/2 cents
E. 31 1/2 cents

0


A B C D E

Student Guide 100



For question 1 to 4

Three adultsR, S, and Vwill be traveling in a van with five childrenF, H, J, L, and M.
The van has a driver's seat and one passenger seat in the front, and two benches behind
the front seats, one bench behind the first. Each bench has room for exactly three
people. Everyone must sit in a seat or on a bench, and seating is subject to the
following restrictions: An adult must sit on each bench. Either R or S must sit in the
driver's seat. J must sit immediately beside M.

1. Which of the following can sit in the front passenger seat?

A: J
B: L
C: R
D: S
E: V




2. Which of the following groups of three can sit together on a bench?

A: F, J, and M
B: F, J, and V
C: F, S, and V
D: H, L, and S
E: L, M, and R



3. If F sits immediately beside V, which of the following CANNOT be true?

A: J sits immediately beside S.
B: L sits immediately beside V.
C: H sits in the front passenger seat.
D: F sits on the same bench as H.
E: H sits on the same bench as R.



4. If S sits on a bench that is behind where J is sitting, which of the following must be
true?

A: H sits in a seat or on a bench that is in front of where M is sitting.
B: L sits in a seat or on a bench that is in front of where F is sitting.
C: F sits on the same bench as H.
D: L sits on the same bench as S
II
ANALYTICAL
Section
No. Of
Questions
20
Choose the correct answer for
each question and shade the
corresponding CIRCLE in the
answer sheet
A B C D E

A B C D E

A B C D E

Student Guide 101
E: M sits on the same bench as V.



For question 5 to 9
The principal of a school is forming a committee. There are to be five members: three
teachers, chosen from Mr. J, Ms. K, Ms. L, Mr. M, and Mr. N; and two students, chosen
from O, P, Q, and R. The composition of the committee must conform to the following
conditions: Ms. J will serve only if R is also on the committee. Ms. L will not serve
unless Ms. K and O also serve. Neither Mr. M nor Mr. N will serve without the other. If
P serves, neither Q nor R can serve.

5. Which of the following is an acceptable committee?

A: J, L, M, N, O
B: K, L, N, O, P
C: K, M, N, O, R
D: L, M, N, O, R
E: M, N, O, P, Q



6. How many different committees could include Mr. J and Q?

A: 1
B: 2
C: 3
D: 4
E: 5



7. If Q and R are both on the committee, who else must be on the committee?

A: J
B: K
C: L
D: M
E: O



8. If M is not on the committee, each of the following must be on the committee EXCEPT

A: J
B: L
C: O
D: Q
E: R


9. In how many different ways can the principal select an acceptable committee?

A: Fewer than 3
B: 3
A B C D E

A B C D E
A B C D E

A B C D E

A
B C D E
Student Guide 102
C: 5
D: 7
E: More than 7



For question 10 to 13

A contractor will build five houses in a certain town on a street that currently has no
houses on it. The contractor will select from seven different models of housesT, U, V,
W, X, Y, and Z. The town's planning board has placed the following restrictions on the
contractor: No model can, be selected for more than one house. Either model W must
be selected or model Z must be selected, but both cannot be selected. If model Y is
selected, then model V must also be selected. If model U is selected, then model W
cannot be selected.


10. If model U is one of the models selected for the street, then which of the
following models must also be selected?

A: T
B: W
C: X
D: Y
E: Z




11. If T, U, and X are three of the models selected for the street, then which of the
following must be the other two models selected?

A: V and W
B: V and Y
C: V and Z
D: W and Y
E: Y and Z



A B C D E

A B C D E

A B C D E

Student Guide 103

12. Which of the following is an acceptable combination of models that can be
selected for the street?

A: T, U, V, X, Y
B: T, U, X, Y, Z
C: T, V, X, Y, Z
D: U, V, W. X, Y
E: V, W, X, Y, Z



13. If model Z is one model not selected for the street, then the other model NOT
selected must be which of the following?

A: T
B: U
C: V
D: W
E: X




For question 14 to 16

Seven childrenF, J, K, M, R, S, and Tare eligible to enter a spelling contest. From
these seven, two teams must be formed, a red team and a green team, each team
consisting of exactly three of the children. No child can be selected for more than one
team. Team selection is subject to the following restrictions: If M is on the red team, K
must be selected for the green team. If F is on the red team, R, if selected, must be on
the green team. R cannot be on the same team as S. J cannot be on the same team as
K.

14. Which of the following can be the three members of the Red team?

A: F, J, and K
B: F, R, and T
C: J, K, and T
D: K, M, and R
E: M, R, and T



15. If M and F are both on the red team, the green team can consist of which of the
following?

A: J, K, and R
B: J, S, and T
C: K, R, and S
D: K, R, and T
E: R, S, and T



A
B C D E
A B C D E

A B C D E

A B C D E

Student Guide 104
16. If M is on the red team, which of the following, if selected, must also be on the
red team?

A: F
B: J
C: R
D: S
E: T



For question 17 to 20

A mail carrier must deliver mail by making a stop at each of six buildings: K, L, M, O, P,
and S. Mail to be delivered are of two types, ordinary mail and priority mail. The
delivery of both types of mail is subject to the following conditions: Regardless of the
type of mail to be delivered, mail to P and mail to S must be delivered before mail to M
is delivered. Regardless of the type of mail to be delivered, mail to L and mail to K must
be delivered before mail to S is delivered. Mail to buildings receiving some priority mail
must be delivered, as far as the above conditions permit, before mail to buildings
receiving only ordinary mail.

17. If K is the only building receiving priority mail, which of the following lists the
buildings in an order, from first through sixth, in which they can receive their mail?

A: L, K, P, S, O, M
B: L, K, S, P, M, O
C: K, L, P, M, S, O
D: K, P, L, S, O, M
E: O, K, L, P, S, M



18. If L, M, and S are each receiving priority mail, which of the following lists the
buildings in an order, from first to sixth, in which they must receive their mail?

A: K, L, P, S, O, M
B: L, K, O, P, S, M
C: L, K, S, P, M, O
D: M, L, S, P, K, O
E: S, L, M, P, K, O



A B C D E
A B C D E

A B C D E

Student Guide 105

19. If the sequence of buildings to which mail is delivered is O, P, L, K, S, M and if S
is receiving priority mail, which of the following is a complete and accurate list of
buildings that must also be receiving priority mail?

A: O, L
B: O, P
C: P, L
D: P, M
E: O, P, L, K



20. If only one building is to receive priority mail, and, as a result, O can be no
earlier than fourth in the order of buildings, which of the following must be the building
receiving priority mail that day?

A: K
B: L
C: M
D: P
E: S


A B C D E
A
B C D E
Student Guide 106



Each sentence below has one or two blanks, each blank indicating that
something has been omitted. Beneath the sentence are five lettered words or
sets of words. Choose the word or set of words that, when inserted in the
sentence, best fits the meaning of the sentence as a whole.

1. Florence Nightingale was ___ in the development of modern medicine, ___ such
practices as sanitization of hospital wards and isolation of actively infected
patients.

A. a collaboratorrejecting
B. a maverickprotesting
C. an innovatorinitiating
D. a pioneercriticizing
E. an individualiststandardizing




2. As a journalist who works to overturn erroneous convictions, Griffin Nicholson
was opposed to the court ruling ___ appeals for inmates who might be ___ .

A. barringculpable
B. curbingexonerated
C. encouraginginnocent
D. scrutinizingeligible
E. shieldingesteemed



3. Linda Greenhouse's articles for the New York Times are an outstanding example
of ___, capsulizing prose into a necessarily limited space.

A. Callousness
B. Brevity
C. Intuition
D. Propriety
E. Fortitude



III
VERBAL
Section
No. Of
Questions
20
Choose the correct answer for each
question and shade the
corresponding CIRCLE in the
answer sheet
A B C D E
A B C D E

A B C D E

Student Guide 107

4. Roberto Clement was seen as ___ during his life because of both his selflessness
on the baseball field and his humanitarian work in his native Nicaragua.

A. An individualist
B. a grandstander
C. a sybarite
D. an altruist
E. an opportunist




5. His habit of spending more than he earned left him in a state of perpetual------
but he------------hoping to see a more affluent day.

A. indigence: persevered in
B. confusion: compromised by
C. enervation: retaliated by
D. motion: responded



6. Known for his commitment to numerous worthy causes, the philanthropist
deserved------ for his-----------.
A. recognition: folly
B. blame: hypocrisy
C. reward: modesty
D. credit: altruism



7. You should --------this paragraph in order to make your essay more-----.

A. Delete, succinct
B. Enlarge, redundant
C. Remove, discursive
D. Revise, abstruse



8. A------glance pays-----attention to details.

A. Furtive: meticulous
B. Cursory: little
C. Cryptic: close
D. Keen: scanty




Each question below consists of a related pair of words or phrases, followed by
five lettered pairs of words or phrases. Select the lettered pair that best
expresses a relationship similar to that expressed in the original pair.

A B C D E

A B C D E

A B C D E

A B C D E

A B C D E

Student Guide 108
9. TIRADE: ABUSIVE

A. Diatribe: familial
B. Satire: pungent
C. Panegyric: laudatory
D. Eulogy: regretful



10. SOLDIER: REGIMENT

A. Colonel: martinet
B. Dancer: balletomane
C. Singer: chorus
D. Trooper: rifle



11. ASYLUM: SHELTER

A. Harbor: concealment
B. Palisade: display
C. Stronghold: defense
D. Cloister: storage



12. STATIC: MOVEMENT

A. Humdrum: excitement
B. Chronic: timeliness
C. Ecstatic: decay
D. Diligent: industry




13. INTEREST: FASCINATE

A. Vex: enrage
B. Vindicate: condemn
C. Regret: rue
D. Appall: bother


A B C D E

A B C D E

A B C D E
A B C D E

A B C D E
Student Guide 109

14. LAUREL: VICTOR

A. Chevrons: army
B. Oscar: movie star
C. Power: glory
D. blue ribbon: cooking



15. PRECEDENT: JUSTIFICATION

A. Kindness: obedience
B. Authority: sanction
C. Usage: submission
D. Tradition: novelty


16. IMPLICATE: COMPLICATE

A. Vitality: inevitable
B. Empathy: sympathy
C. Importune: construct
D. Imply: simplify



17. PERMEATE: RUEFUL

A. Truculent: merciful
B. Sadden: pitiful
C. Evaporate: mournful
D. Penetrate: sorrowful


18. ENERVATE: STRENGTHEN

A. Aver: attribute
B. Divert: turn
C. Apprise: appraise
D. Stultify: enliven




Read the passages and answer the questions given at the end:

Recent technological advances in manned undersea vehicles have overcome some of the
limitations of divers and diving equipment. Without vehicles, divers often become
sluggish and their mental concentration was limited. Because of undersea pressure that
affected their speech organs, communication among divers was difficult or impossible.
But today, most oceanographers make observations by the means of instruments that
are lowered into the ocean or from samples taken from the water direct observations of
the ocean floor are made not only by divers of more than seven miles and cruise at the
depth of fifteen thousand feet. Radio equipment buoys can be operated by remote
A B C D E
A B C D E

A B C D E

A B C D E
A B C D E
Student Guide 110
control in order to transmit information back to land based laboratories, including data
about water temperature, current and weather.
Some of mankinds most serous problems, especially those concerning energy and food,
may be solved with the help of observations made by these undersea vehicles.

19. With what topic is the passage primarily concerned?

A. Recent technological advances.
B. Communication among divers.
C. Direct observation of the ocean floor
D. Undersea vehicles




20. Divers have problems in communicating underwater because?

A. The pressure affected their speech organs
B. The vehicles they used have not been perfected.
C. They did not pronounce clearly
D. The water destroyed their speech organs.



A B C D E

A B C D E

Student Guide 111









Drill Test II
Student Guide 112


1. A piece of wood 35 feet, 6 inches long was used to make 4 shelves of equal length. The
length of each shelf was
A. 9 feet, 1 1/2 inches
B. 8 feet, 10 1/2 inches
C. 7 feet, 10 1/2 inches
D. 7 feet, 1 1/2 inches
E. 6 feet, 8 1/2 inches



2. The tiles in the floor of a bathroom are 15/16 inch squares. The cement between the tiles
is 1/16 inch. There are 3240 individual tiles in this floor. The area of the floor is

A. 225 sq. yds.
B. 2.5 sq. yds.
C. 250 sq. ft.
D. 22.5 sq. yds
E. 225 sq. ft.



3. A man bought a TV set that was listed at $160. He was given successive discounts of
20% and 10%. The price he paid was

A. $129.60
B. $119.60
C. $118.20
D. $115.20
E. $112.00



4. Mr. Jones' income for a year is $15,000. He pays 15% of this in federal taxes and 10%
of the remainder in state taxes. How much is left?

A. $12,750
B. $9,750
C. $14,125
D. $13,500
E. $11,475



5. The radius of a circle which has a circumference equal to the perimeter of a hexagon
whose sides are each 22 inches long is closest in length to which one of the following?
A. 7
B. 21
C. 14
D. 28
E. 24



I
Quantitative
Section
No of
Questions
10
Choose the correct answer for
each question and shade the
corresponding CIRC LE in the
answer sheet
A B C D E
A B C D E

A B C D E

A B C D E

A B C D E

Student Guide 113
6. If a, is a multiple of 5 and b = 5a, which of the following could be the value of a + b?

I. 60 II. 100 III. 150
A. I only
B. III only
C. I and III only
D. II and III only
E. None of these


7. Which of the following expressions has the greatest value?

A. 4 4 4 + 4
B. 4 4 4 + 4
C. 4 4 4 4
D. 4 4 + 4 4
E. 2 2 + 2 2



8. If (a + 3) / 5 is an integer, what is remainder when a is divided by 5?
A. 1
B. 2
C. 3
D. 4
E. 5


9. The integral part of logarithm is called
A. Characteristic
B. Mantissa
C. Solution
D. Root
E. None of these



10. On the y-axis, the x-coordinate is
A. 1
B.
C. zero
D. -
E. 1

A B C D E

A B C D E

A B C D E

A B C D E

A B C D E

Student Guide 114



For question 1 to 3

A volunteer uses a truck to pick up donations of unsold food and clothing from stores
and to deliver them to locations where they can be distributed. He drives only along
a certain network of roads. In the network there are two-way roads connecting each
of the following pairs of points: 1 with 2, 1 with 3, 1 with 5, 2 with 6, 3 with 7, 5
with 6, and 6 with 7. There are also one-way roads going from 2 to 4, from 3 to 2,
and from 4 to 3. There are no other roads in the network, and the roads in the
network do not intersect. To make a trip involving pickups and deliveries, the
volunteer always takes a route that for the whole trip passes through the fewest of
the points 1 through 7, counting a point twice if the volunteer passes through it
twice. The volunteer's home is at point 3. Donations can be picked up at a
supermarket at point 1, a clothing store at point 5, and a bakery at point 4.
Deliveries can be made as needed to a tutoring center at point 2, a distribution
center at point'6, and a shelter at point 7.

1. If the volunteer starts at the supermarket and next goes to the shelter, the first
intermediate point his route passes through must be

A: 2
B: 3
C: 5
D: 6
E: 7



2. If, starting from home, the volunteer is then to make pickups for the shelter at
the supermarket and the bakery (in either order), the first two intermediate
points on his route, beginning with the first, must be

A: 1 and 2
B: 1 and 3
C: 2 and 1
D: 2 and 4
E: 4 and 2



II
ANALYTICAL
Section
No. Of
Questions
20
Choose the correct answer for
each question and shade the
corresponding CIRCLE in the
answer sheet
A B C D E

A B C D E

Student Guide 115

3. If, starting from the clothing store, the volunteer next is to pick up bread at
either the supermarket or the bakery (whichever stop makes his route go
through the fewest of the points) and then is to go to the shelter, the first two
points he reaches after the clothing store, beginning with the first, must be

A: 1 and 2
B: 1 and 3
C: 4 and 2
D: 6 and 2
E: 6 and 4



For question 4 to 5

There are seven cages next to each other in a zoo. The following is known about the
cages. Each cage has only one animal, which is either a monkey or a bear. There is
a monkey in each of the first and last cages. The cage in the middle has a bear. No
two adjacent cages have bears in them. The bears cage in the middle has two
monkey cages on either side. Each of the two other bear cages are between and
next to two monkey cages

4. How many cages have monkeys in them?

A: 2
B: 3
C: 4
D: 5
E: 6



5. The bear cage in the middle must have

A: No other bear cage to its left
B: No monkey cage on its right.
C: A bear cage to its left and to its right
D: Other bear cages next to it.
E: No monkey cage to its left.



For question 6 to 8

A nursery class in a school has a circular table with eleven seats around it. Five girls
(Kiran, Lado, Maryam, Omera and Parveen) and five boys (Farhan, Ghaus, Haris,
Imdad and Jahangir) are seated around the table. None of the girls are seated in a
seat adjacent to another girl. Kiran sits between Farhan and Ghaus, and next to
each of them. Jahangir does not sit next to Imdad.

6. Which of the following is a possible seating order around the table?

A: Empty seat, Farhan, Kiran, Ghaus, Lado, Omera, Haris, Imdad, Parveen, Jahangir,
and Maryam.
A B C D E

A B C D E

A B C D E

Student Guide 116
B: Empty seat, Farhan, Kiran, Ghaus, Lado, Jahangir, Parveen, Omera, Imdad,
Maryam, Haris.
C: Empty seat, Farhan, kiran, Ghaus, Omera, Jahangir, Parveen, Imdad, Maryam,
Haris, Lado.
D: Empty seat, Omera, Farhan, Kiran, Ghaus, Lado, Jahangir, Imdad, Parveen,
Haris, Maryam.
E: Empty seat, Maryam, Farhan, Kiran, Ghaus, Lado, Jahangir, Perveen, Imdad,
Omera, Haris.



7. If Lado, Haris, Maryam, Jahangir, and Ghaus are seated in that order, which of
the following is a correct completion of the seating order after Ghaus?

A : Kiran, Farhan, Omera, Imdad, Parveen, empty seat.
B : Kiran, Farhan, Imdad, Omera, empty seat, Parveen
C : Farhan, Parveen, Kiran, Imdad, Omera, empty seat.
D : Kiran, Farhan, Parveen, Imdad, empty seat, Omera.
E : Kiran, Farhan, Omera, empty seats, Parveen, Imdad.



8. If Jahangir leaves his seat and occupies the empty seat, his new seating position
would be between:

A : Farhan and Kiran
B : Maryam and Ghaus
C : Kiran and Ghaus
D : Imdad and Lado
E : Parveen and Lado



For question 9 to 11
Four telephone operators (Abid, Baqir, Chauhan, and Daud) each have to perform
duties at the telephone exchange on four different days, Thursday through Sunday.
The following is known about their duty schedule: Chauhan has his duty day before
Abid. Daud has his duty day later than Baqir.

9. Which of the following is a possible order of duty days for the four operators?

A: Chauhan, Daud, Abid and Baqir.
B: Daud, Chauhan, Abid, and Baqir.
C: Baqir, Chauhan, Daud and Abid.
D: Abid, Chauhan, Daud and Baqir.
E: Abid, Baqir, Daud and Chauhan.





10. If Chauhan has his duty day on Saturday, who must have his duty day on
Thursday?

A: Either Abid or Daud.
A B C D E

A B C D E
A B C D E

A

B C D E

Student Guide 117
B: Daud
C: Abid
D: Either Baqir or Daud.
E: Baqir.



11. Each of the following is possible EXCEPT:

A: Chauhan has his duty on Thursday.
B: Baqir has his duty on Thursday.
C: Daud has his duty on Saturday.
D: Baqir has his duty on Sunday
E: Abid has his duty on Sunday.



For question 12 to 13
There are 12 seats facing the blackboard in a classroom, four seats
(A1, A2, A3 & A4) in that order are in row A, the first row from the blackboard.
Immediately behind row A is row B with four seats (B1, B2, B3 & B4) in that order.
Immediately behind row B, is the last row C with four seats (C1, C2, C3 & C4) in that
order. Six students attend the class the following is known about there seating
arrangement: Ejaz sits exactly in front of Comil, Seat A2 is always unoccupied Daud
does not sit next to Farhat, Gharuy sits in seat A4 Hamid does not sit in seat B4 All
the seats in row C always remain empty

12. If Daud sits in seat B3, then Farhat must sit in seat:

A: A3
B: A1
C: B4
D: B2
E: C2



13. Suppose that Hamid and Ejaz are sitting in seats A1 and A3 respectively, then it
CANNOT be true that seat:

A: B1 is occupied by Daud.
B: B2 is empty
C: B1 is empty
D: B3 is OCCUPIED BY Comil
E: B4 is empty




For question 14 to 17

The principal of a school is forming a committee. There are to be five members:
three teachers, chosen from Mr. J, Ms. K, Ms. L, Mr. M, and Mr. N; and two students,
chosen from O, P, Q, and R. The composition of the committee must conform to the
following conditions: Ms. J will serve only if R is also on the committee. Ms. L will
A B C D E

A B C D E

A B C D E

A B C D E

Student Guide 118
not serve unless Ms. K and O also serve. Neither Mr. M nor Mr. N will serve without
the other. If P serves, neither Q nor R can serve.

14. Which of the following is an acceptable committee?

A: J, L, M, N, O
B: K, L, N, O, P
C: K, M, N, O, R
D: L, M, N, O, R
E: M, N, O, P, Q



15. How many different committees could include Mr. J and Q?
A: 1
B: 2
C: 3
D: 4
E: 5




16. If Q and R are both on the committee, who else must be on the committee?
A: J
B: K
C: L
D: M
E: O




17. In how many different ways can the principal select an acceptable committee?
A: Fewer than 3
B: 3
C: 5
D: 7
E: More than 7


A B C D E

A B C D E

A B C D E

A B C D E

Student Guide 119

For question 18 to 20

Three adultsR, S, and Vwill be traveling in a van with five childrenF, H, J, L,
and M. The van has a driver's seat and one passenger seat in the front, and two
benches behind the front seats, one bench behind the other. Each bench has room
for exactly three people. Everyone must at in a seat or on a bench, and seating is
subject to the following restrictions: An adult must sit on each bench. Either R or S
must sit in the driver's seat. J must sit immediately beside M.

18. Which of the following can sit in the front passenger seat?
A: J
B: L
C: R
D: S
E: V



19. Which of the following groups of three can sit together on a bench?
A: F, J, and M
B: F, J, and V
C: F, S, and V
D: H, L, and S
E: L, M, and R



20. If F sits immediately beside V, which of the following CANNOT be true?
A: J sits immediately beside S.
B: L sits immediately beside V.
C: H sits in the front passenger seat.
D: F sits on the same bench as H.
E: H sits on the same bench as R.


A B C D E

A B C D E
A B C D E

Student Guide 120


Each sentence below has one or two blanks; each blank indicates that
something has been omitted. Beneath the sentence are five lettered words or
sets of words. Choose the word or set of words that, when inserted in the
sentence, best fits the meaning of the sentence as a whole.

1. Surprisingly enough, it is more difficult to write about the--------than about the--
-and strange.

A. specific, foreign
B. abstract, prosaic
C. commonplace, exotic
D. simple, routine



2. A-----response is one that is made with----------.

A. stupid, fear
B. speedy, alacrity
C. sure, slowness
D. harmful, grimaces



3. A----is a-------.


A. norm, standard
B. criterion, mistake
C. discipline, school
D. doctrine, follower



4. It is widely believed that a nuclear war could ____ enough smoke and dust to
block out the sun and freeze the earth.

A. billow
B. extinguish
C. generate
D. duplicate
E. decimate



III
VERBAL
Section
No. Of
Questions
20
Choose the correct answer for each
question and shade the corresponding
CIRCLE in the answer sheet
A B C D E
A B C D E
A B C D E

A B C D E
Student Guide 121

5. Consumption of red meat has ____ because its fat content has become a
worrisome and ____ matter.

A. abated ... dubious
B. skyrocketed ... stressful
C. abounded ... divisive
D. stabilized ... newsworthy
E. declined ... controversial



6. It takes ____ character to ____ the extremities of the arctic region.

A. an unflappable ... sustain
B. a dictatorial ... brook
C. a Spartan ... negotiate
D. an inimitable ... resist
E. a nomadic ... espouse



7. Consumers refused to buy meat products from the company because of rumors
that the water supply at the meat processing plant was ______; the rumors,
however, were quite ______, with no hard evidence to back them up.

A. uninspected .. reckless
B. contaminated .. unsubstantiated
C. impure .. damaging
D. misdirected .. scandalous
E. unscrupulous .. vicious



8. Many kinds of harmful viruses are unhindered when passing through different
parts of the host organism; indeed, there are few organic substances which such
viruses cannot______.

A. undermine
B. disseminate
C. aerate
D. exterminate
E. perforate


9. Their conversation was unsettling, for the gravity of their topic contrasted so
oddly with the ______ of their tone

A. uniqueness
B. rapidity
C. lightness
D. precision
E. reverence


A B C D E

A B C D E

A B C D E

A B C D E

A B C D E

Student Guide 122

10. Throughout the animal kingdom, ____ bigger than the elephant.

A. whale is only the
B. only the whale is
C. is the whale only
D. only whale is the
E. whale is only



Each question below consists of a related pair of words or phrases, followed
by five lettered pairs of words or phrases. Select the lettered pair that best
expresses a relationship similar to that expressed in the original pair.

11. YAWN: BOREDOM ::

A. dream : sleep
B. anger : madness
C. smile : amusement
D. face : expression
E. impatience : rebellion



12. OBSTRUCTION : BUOY ::

A. construction : building
B. boy : girl
C. danger : red light
D. iceberg : titanic
E. arise : lay down



13. CONCERT : MUSIC ::

A. performance : artist
B. exhibition : art
C. play : actor
D. operetta : singer
E. rock : role



14. TEAMMATE : ADVERSARY ::

A. felon : criminal
B. enemy : associate
C. pacifier : agitator
D. winner : loser
E. friend : foe



A B C D E

A B C D E

A B C D E

A B C D E

A B C D E
Student Guide 123
For Question 15-20 read the following passage:

A popular theory explaining the evolution of the universe is known as the Big Bang
Model. According to the model at some time between twenty billion years ago, all
present matter and energy were compressed into a small ball only a few kilometers
in diameter. It was, in effect, an atom that contained in the form of pure energy all
of the components of the entire universe. Then, at a moment in time that
astronomers refer to as T = 0, the ball exploded, hurling the energy into space.
Expansion occurred. As the energy cooled most of it became matter in the form of
protons, neutrons and electrons. These original particles combined to form
hydrogen and helium and continued to expand. Matter formed into galaxies with
stars and planets.


15. Which sentence best summarizes this passage?

A. The big band theory does not account for the evolution of the universe
B. According to the Big Bang Model, an explosion caused the formation of the
universe
C. The universe is made of hydrogen and helium
D. The universe is more than ten billion years old


16. According to this passage when were the galaxies formed?

A. Ten Billion Years ago
B. Fifteen billion Years ago
C. At T = 0
D. Twenty billion years ago


17. The word compressed in the passage could best be replaced by

A. Excited
B. Balanced
C. Reduced
D. Controlled


18. It may be inferred that

A. Energy and matter are the same
B. Protons, neutrons, and electrons are not matter
C. Energy may be converted into matter
D. The galaxies stopped expanding as energy cooled



19. The word it in the passage refers to

A. Energy
B. Space
C. Expansion
D. Matter

A B C D E
A B C D E

A B C D E

A B C D E

A B C D E

Student Guide 124


20. The environment before the Big Bang is described as all the following EXCEPT

A. Compressed matter
B. Energy
C. All the components of the universe
D. Protons, electrons and neutrons



A B C D E

Student Guide 125









Drill Test III
Student Guide 126


I
Quantitative
Section
No. Of
Questions
20
Choose the correct answer for each
question and shade the
corresponding OVAL in the answer
sheet




1. If the pattern of dots shown above is continued so that each row after Row One
contains 1 dot more than the row immediately above it, which row will contain 12
dots?

A. Seven
B. Eight
C. Nine
D. Ten
E. Twelve



2. Each of Steve's buckets has a capacity of 11 gallons, while each of Mark's
buckets can hold 8 gallons. How much more water in gallons can 7 of Steve's
bucket's hold compared to 7 of Mark's buckets?

A. 3
B. 7
C. 21
D. 24
E. 56



3. Two integers have a sum of 42 and a difference of 22. The greater of the two
integers is

A. 22
B. 25
C. 28
D. 31
E. 32




4. The average of five numbers is 34. If three of the numbers are 28, 30 and 32,
what is the average of the other two?

A. 40
B. 50
C. 60
A
B C D E
A
B C D E
A B C D E
Student Guide 127
D. 70
E. 80









5. In the figure above, what is the value of x?

A. 30
B. 40
C. 50
D. 80
E. 100



6. In a certain cake, two straight cuts (made along two different radii) succeed in
removing 4/15 of the total cake. What is the central angle in degrees of the
piece cut?

A. 26
B. 60
C. 85
D. 92
E. 96



7. If an equilateral triangle and a square have the same perimeter, what is the ratio
of the length of the sides of the equilateral triangle to the lengths of the sides of
the square?

A. 3:4
B. 4:3
C. 1:4
D. 1:3
E. 3:1



A B C D E
A B C D E
A B C D E
A B C D E

Student Guide 128
8. If 2 and 4 each divide q without remainder, which of the following must q divide
without remainder.

A. 1
B. 2
C. 4
D. 8
E. It cannot be determined from the information given.




9. The ratio of boys to girls in a certain classroom was 2 : 3. If boys represented
five more than one third of the class, how many people were there in the
classroom?

A. 15
B. 25
C. 30
D. 45
E. 75




10. Let xy = z, where x,y,z and nonzero numbers. If x is multiplied by 3 and z is
divided by 3, this is equivalent to multiplying y by

A. 1/9
B. 1/3
C. 1
D. 3
E. 9



11. If 5x = 3, then (5x + 3) =

A. 0
B. 9
C. 18
D. 36
E. 81


A B C D E
A B C D E
A B C D E
A B C D E

Student Guide 129

12. If the postal charges for a package are 62 cents for the first five ounces and 8
cents for each additional ounce, what is the weight of a package for which the
charges are $1.66? (Assume there are 16 ounces in one pound)

A. 1.05 pounds
B. 1.1 pounds
C. 1.125 pounds
D. 1.25 pounds
E. 1.5 pounds



13. If m/n = .75, then what is 3m + 2n?

A. 0
B. 8
C. 14
D. 17
E. 24



14. Which is greater?





A. if the quantity in Column A is greater
B. if the quantity in Column B is greater
C. if the two quantities are equal
D. if there is no relationship between these two quantities
E. if the relationship cannot be determined from the information given



15. Which of the following has a graph that is symmetric to the x-axis

A. y = x
B. y = x
2
+ 3
C. y
2
= x
D. y = x
3
x
E. y != x


Column A Column B
(10/4) / (3/2) * (3/7) (3/4) * (10/7) / (3/2)
A B C D E

A B C D E
A B C D E
A B C D E

Student Guide 130

16. The prime factors of 96 are:

A. 2 and 3
B. 6 and 8
C. 2, 3 and 4
D. 8 and 12
E. 3 and 9



17. 8 are what percent of the 6?

A. 1.25
B. 75
C. 125
D. 133.333
E. 150



18. 2- 3 =

A. -8
B. 8
C. -6
D. -1/8
E. 1/8




19. If x + 1 < 3x + 5, then

A. X > -2
B. x < -2
C. x = 0v v v v
D. x < 2
E. x > 2



20. Which of the numbers cannot be represented by a repeating decimal?

A. 11/9
B. 23/7
C. v 3
D. 4 1/3
E. 2



A B C D E

A B C D E

A B C D E

A B C D E

A B C D E

Student Guide 131

II
Analytical
Reasoning
No. of
Questions
15
Choose the correct answer for each
question and shade the
corresponding CIRCLE in the answer
sheet

The office staff of the XYZ Corporation presently consists of three bookkeepers (L, M and
N ) and five secretaries (O, P, Q, R and S). Management is planning to open a new office
in another city sending three secretaries and two bookkeepers from the present staff. To
do so they plan to separate certain individuals who do not function well together. The
following guidelines were established to set up the new office:

(a) Bookkeepers L and N are constantly finding faults with one another therefore should
not be sent together to the new office.
(b) N and P function well alone but not as a team. They should be separated.
(c) O and R have not been on speaking terms for many months. They should not go
together.
(d) Since O and Q have been competing for a promotion, they should not be in one team.
Based on the information given above find the correct answers to the following Questions:
1. If M insists on staying back then how many combinations are possible?

A. 1
B. 2
C. 3
D. None



2. If L is to be moved as one of the bookkeepers, which of the following CANNOT be a
possible working unit?

A. LMOPS
B. LMPQS
C. LMORS
D. LMPRS



3. If N is sent to the new Office which member of the staff CANNOT be sent?

A. O
B. M
C. Q
D. R



4. If O is sent to the new office then which of the following is a possible team?

A. LMOPR
B. MNOQS
C. MNOPS
D. LMOPS



5. If both N and Q are moved to the new office, how many combinations are possible?

A. 2
B. 3
A
B C D E
A B C D E
A B C D E

A B C D E

Student Guide 132
C. 4
D. 1




6. A map representing countries R, S, W, X, Y and Z is to be drawn. Adjacent countries
cannot have the same color in the map.

The countries adjacent to each other are as follows:

Each of R, S, X and Y is adjacent to W.
X is adjacent to Y.
Each of R and S is adjacent to Z.

If X is the same color as Z then it must be true that

A. W is a different color from any other country.
B. S is a different color from any other country.
C. X is the same color as Y.
D. S is the same color as X.



Two statements, labeled I. & II, follow each of the following questions. The
statements contain certain information. In the questions you do not actually
have to compute an answer, rather you have to decide whether the information
given in the statements I. and II. is sufficient to find a correct answer by using
basic mathematics and every day facts?

7. A long distance runner has just completed running 28 miles. How long did it take him to
finish the journey?

I. His record speed is 8.25 miles per hour.
II. His average speed through the journey was 8 miles per hour.

A. Statement I. ALONE is sufficient but II. ALONE is not sufficient to answer this
question.
B. Statement II. ALONE is sufficient but I. ALONE is not sufficient to answer this
question.
C. Statements I. and II. TOGETHER are sufficient to answer the question but
NEITHER of them is sufficient ALONE.
D. Statements I. and II. COMBINED are NOT sufficient to answer the question and
additional information is needed to find the correct answer.



8. Captain of the national cricket team has to be the most popular member of the team.
Who is the captain of Pakistans national cricket team?
I. Waqar is the best player on the team.
II. Waseem is the senior-most member.

A. Statement I. ALONE is sufficient but II. ALONE is not sufficient to answer this
question.
B. Statement II. ALONE is sufficient but I. ALONE is not sufficient to answer this
question.
C. Statements I. and II. TOGETHER are sufficient to answer the question but
NEITHER of them is sufficient ALONE.
D. Statements I. and II. COMBINED are NOT sufficient to answer the question and
additional information is needed to find the correct
answer.
A B C D E

A B C D E

A B C D E

Student Guide 133




9. In a BCE class at CIIT, 30 boys and 10 girls registered Calculus II. How many boys
passed the course?
I. 5 students could not pass.
II. There were 2 girls who obtained A grade.

A. Statement I. ALONE is sufficient but II. ALONE is not sufficient to answer this
question.
B. Statement II. ALONE is sufficient but I. ALONE is not sufficient to answer this
question.
C. Statements I. and II. TOGETHER are sufficient to answer the question but
NEITHER of them is sufficient ALONE.
D. Statements I. and II. COMBINED are NOT sufficient to answer the question and
additional information is needed to find the correct answer.




10. A horse ran 100 miles without stopping. What was its average speed in miles per hour?
I. The journey started at 8 PM and ended at 4 AM the following day.
II. The horse ran 20 miles per hour for the first 50 miles.

A. Statement I. ALONE is sufficient but II. ALONE is not sufficient to answer this
question.
B. Statement II. ALONE is sufficient but I. ALONE is not sufficient to answer this
question.
C. Statements I. and II. TOGETHER are sufficient to answer the question but
NEITHER of them is sufficient ALONE.
D. Statements I. and II. COMBINED are NOT sufficient to answer the question and
additional information is needed to find the correct answer.


A B C D E

A B C D E

A B C D E

Student Guide 134

11. How much time will computer a need to solve 150 problems?
I. The computer needs 50 seconds to solve one problem.
II. Computer never takes more than 60 seconds to solve a problem.

A. Statement I. ALONE is sufficient but II. ALONE is not sufficient to answer this
question.
B. Statement II. ALONE is sufficient but I. ALONE is not sufficient to answer this
question.
C. Statements I. and II. TOGETHER are sufficient to answer the question but
NEITHER of them is sufficient ALONE.
D. Statements I. and II. COMBINED are NOT sufficient to answer the question and
additional information is needed to find the correct answer.



12. How many pencils does Raheel have?

I. He bought two boxes each containing 10 pencils.
II. He lent two pencils to Khaleel

A. Statement I. ALONE is sufficient but II. ALONE is not sufficient to answer this
question.
B. Statement II. ALONE is sufficient but I. ALONE is not sufficient to answer this
question.
C. Statements I. and II. TOGETHER are sufficient to answer the question but
NEITHER of them is sufficient ALONE.
D. Statements I. and II. COMBINED are NOT sufficient to answer the question and
additional information is needed to find the correct answer.



13. In a certain farm there are 47 goats. How many large brown goats are there?

I. 16 of the goats are large.
II. There are 18 brown goats in the farm.

A. Statement I. ALONE is sufficient but II. ALONE is not sufficient to answer this
question.
B. Statement II. ALONE is sufficient but I. ALONE is not sufficient to answer this
question.
C. Statements I. and II. TOGETHER are sufficient to answer the question but
NEITHER of them is sufficient ALONE.
D. Statements I. and II. COMBINED are NOT sufficient to answer the question and
additional information is needed to find the correct answer.


14. Can there be more than 200 pictures in a 60-page book?

I. There is at least one picture in each page.
II. There are no more than 3 pictures in any page.

A. Statement I. ALONE is sufficient but II. ALONE is not sufficient to answer this
question.
B. Statement II. ALONE is sufficient but I. ALONE is not sufficient to answer this
question.
C. Statements I. and II. TOGETHER are sufficient to answer the question but
NEITHER of them is sufficient ALONE.
D. Statements I. and II. COMBINED are NOT sufficient to answer the question and
additional information is needed to find the correct answer.


A
B C D E
A B C D E

A B C D E

A B C D E

Student Guide 135
15. If P > Q and R > S, then, P + R > Q + S. Is X > Y?
I. X + A > Y + B
II. A > B

A. Statement I. ALONE is sufficient but II. ALONE is not sufficient to answer this
question.
B. Statement II. ALONE is sufficient but I. ALONE is not sufficient to answer this
question.
C. Statements I. and II. TOGETHER are sufficient to answer the question but
NEITHER of them is sufficient ALONE.
D. Statements I. and II. COMBINED are NOT sufficient to answer the question and
additional information is needed to find the correct answer.

A D E

B C

Student Guide 136



Each sentence bel ow has one or t wo bl anks, each bl ank i ndi cates
that somethi ng has been omi tted. Beneath the sentence are fi ve
l ettered words or sets of words. Choose the word or set of words
that, when i nserted i n the sentence, best fi ts the meani ng of the
sentence as a whol e.


1. Although its publicity has been ___, the film itself is intelligent, well-acted,
handsomely produced and altogether ___

A. tasteless respectable
B. extensive moderate
C. sophisticated moderate
D. risqu crude
E. perfect spectacular



2. The Inuit natives of Alaska's North Slope worry that ___ oil exploration might
___their sensitive natural environment.

A. additionalassist
B. currentbolster
C. curtailedshatter
D. unregulateddamage
E. controlledreassess



3. Ants live in colonies based on ___; each member contributes to the good of all by
actively working with others in performing necessary tasks.

A. Heredity
B. Individualism
C. Cooperation
D. Reasoning
E. Instinct



Each question below consists of a related pair of words or phrases, followed
by five lettered pairs of words or phrases. Select the lettered pair that best
expresses a relationship similar to that expressed in the original pair.

4. STUDYING: LEARNING::
A. running : jumping
B. investigating : discovering
C. reading : writing


Choose the correct answer for each
question and shade the corresponding
CIRCLE in the answer sheet
III
VERBAL
Section No of
Questions
10
A B C D E
A B C D E
A B C D E

Student Guide 137
D. dancing : swimming
E. talking : listening



5. AFTERNOON : DUSK ::

A. breakfast : dinner
B. yesterday : tomorrow
C. Sunday : Saturday
D. night : dawn
E. arise : lay down



6. VIBRATION: SOUND :

A. gravity : pull
B. watercolor : paint
C. accident : death
D. worm : reptile
E. arrive : home



7. RUN : RACE ::

A. walk : pogo stick
B. swim : boat
C. fly : kite
D. sink : bottle
E. repair : automobile



Read the passages and answer the questions asked at its end.

Almost a century ago Alfred Binet, a gifted psychologist, was asked by the French
Ministry of Education to help determine who would experience difficulty in school. Given
the influx of provincials to the capital, along with immigrants of uncertain stock, Parisian
officials believed they needed to know who might not advance smoothly through the
system. Proceeding in an empirical manner, Binet posed many questions to youngsters
of different ages. He ascertained which questions when answered correctly predicted
success in school, and which questions when answered incorrectly foretold school
difficulties. The items that discriminated most clearly between the two groups became,
in effect, the first test of intelligence.

Binet is a hero to many psychologists. He was a keen observer, a careful scholar, an
inventive technologist. Perhaps even more important for his followers, he devised the
instrument that is often considered psychology's greatest success story. Millions of
people who have never heard Binet's name have had aspects of their fate influenced by
instrumentation that the French psychologist inspired. And thousands of
psychometricians specialists in the measurement of psychological variables earn
their living courtesy of Binet's invention.
A B C D E

A B C D E

A B C D E

A B C D E

Student Guide 138

Although it has prevailed over the long run, the psychologist's version of intelligence is
now facing its biggest threat. Many scholars and observers and even some
iconoclastic psychologists feel that intelligence is too important to be left to the
psychometricians. Experts are extending the breadth of the concept proposing much
intelligence, including emotional intelligence and moral intelligence. They are
experimenting with new methods of ascertaining intelligence, including some that avoid
tests altogether in favor of direct measures of brain activity. They are forcing citizens
everywhere to confront a number of questions: What is intelligence? How ought it to be
assessed? And how do our notions of intelligence fit with what we value about human
beings? In short, experts are competing for the "ownership" of intelligence in the next
century.

8. According to the passage, which of the following is most similar to the "barometer"
developed by Binet?

A. The S.A.T. or other standardized college admission test.
B. The written portion of a driver's license test.
C. Open tryouts for a varsity athletic team
D. An electronic scan of brain-wave activity.
E. The trivia questions of a game show.



9. The author suggests which of the following about "citizens everywhere"?

A. They do not have a sufficiently accurate definition of intelligence to evaluate
recent scientific developments.
B. They stand to benefit from recent progress in the scientific assessment of
intelligence.
C. The experiments they are performing with new methods of intelligence
measurement are valuable and interesting.
D. They are at odds with the experts over who should have the right to define
"intelligence."
E. Traditionally they have not given careful consideration to some important
issues concerning intelligence.


10. As used in line # 8, "discriminated" most nearly means

A. equalized
B. predetermined
C. showed favoritism
D. displayed intolerance
E. distinguished


A B C D E

A B C D E

A B C D E

Student Guide 139





Drill Test IV
Student Guide 140


I
Quantitative
Section
No. Of
Questions
20
Choose the correct answer for each
question and shade the
corresponding CIRCLE in the
answer sheet


1. If the length of BC is twice the length of AC, what are the coordinates of B where
A=( x,y)?

A. (x,2y)
B. (-x,2y)
C. (2x,y)
D. (-2x,y)
E. (-2x,2y)


2. The average of five numbers is 34. If three of the numbers are 28, 30 and 32, what
is the average of the other two?
A. 40
B. 50
C. 60
D. 70
E. 80

A B C D E

A B C D E


1
Student Guide 141



3. In the figure above, rectangle AEJL has been divided into 8 congruent squares with
each of the 8 squares having an area of 16. What is the length of
AE + MF + LG+ AL + BK + CJ + DH + EG?

A. 32
B. 44
C. 88
D. 128
E. 176



4. For any positive integer x, #x = x/3 and &x = 9 /x. which of the following is an
expression for the product of #x and &x?

A. 3x
B. x
C. 1
D. x
3
/64
E. 27 x
3




5. In a certain town, p gallons of gasoline are needed per month for each car. How
long will q gallons last at this rate given that there are r cars in town?

A. pr/q
B. qr/p
C. r/pq
D. q/pr
E. pqr


G
A B C D E

A B C D E

A B C D E


Student Guide 142

6. Let xy = z, where x,y,z are nonzero numbers. If x is multiplied by 3 and z is divided
by 3, this is equivalent to multiplying y by

A. 1/9
B. 1/3
C. 1
D. 3
E. 9




7. If x, y, and z are different positive odd integers and x + y + z = 11, what is the
greatest possible value of z?

A. 10
B. 9
C. 8
D. 7
E. 6



8. If the postal charges for a package are 62 cents for the first five ounces and 8 cents
for each additional ounce, what is the weight of a package for which the charges are
$1.66? (Assume there are 16 ounces in one pound)

A. 1.05 pounds
B. 1.1 pounds
C. 1.125 pounds
D. 1.25 pounds
E. 1.5 pounds



9. What fraction of two weeks is 24 minutes?

A. 1/120
B. 1/336
C. 1/840
D. 1/2880
E. 1/20160


A B C D E

A B C D E

A B C D E

A B C D E

Student Guide 143

10. If the vertices of a triangle are at (0,0), (-3, 4) and (3, 4), what is the area of the
triangle?

A. 4
B. 6
C. 12
D. 14
E. 18




11. A water-tank has a base with dimensions 2 feet by 6 feet. If a cube with each side 1
foot is totally immersed in the water, how many inches will the water rise? (12inches
= 1 foot)

A. 1
B. 2
C. 4
D. 8
E. It cannot be determined from the information given





12. In the figure above, the quadrilateral ABCD is a trapezoid with x = 2. The diameter
of each semicircle is a side of the trapezoid. What is the sum of the lengths of the
four drawn semicircles? (Round to the nearest whole number.)

A. 13
B. 16
C. 19
D. 22
E. 31


A B C D E

A
B C D E
A B C D E

Student Guide 144

13. If n + 3 = n x 3, then n =

A. 0.5
B. 1.5
C. 2
D. 2.5
E. 3



14. If an equilateral triangle and a square have the same perimeter, what is the ratio of
the length of the sides of the equilateral triangle to the lengths of the sides of the
square?

A. 3:4
B. 4:3
C. 1:4
D. 1:3
E. 3:1




15. A restaurant has a special whereby both parents can eat for $20 and each child can
eat for $5. Assuming a family group consists of both parents and at least one child,
what is the maximum number of family groups that could have attended given that
the restaurant took $115?

A. 6
B. 5
C. 4
D. 3
E. 2




16. Which of the following points lays in the interior of the circle whose radius is 10 and
whose center is at the origin?

A. (-9, 4)
B. (5, -19)
C. (0, -10)
D. (10, -1)
E. (0,15)



A B C D E

A B C D E

A B C D E

A B C D E

Student Guide 145

17. If the perimeter of the rectangle ABCD is 14, what is the perimeter of BCD?




18. The roots of ax
2
+ bx + c = 0 are real only if

A. b
2
4ac 0
B. b
2
4ac = 0
C. b
2
+ 4ac = 0
D. b
2
4ac < 0
E. b
2
4bc < 0



19. The two numbers, whose sum is -13 and product -30, are

A. 2, 15
B. 2, -15
C. -3, 10
D. 3, 10
E. -3, -13



20. Let A = total area of five circles of radius r and let B = total area of three circles of
radius s. If A = B, then r / s =

A. 3/5
B. 3 / 5
C. 3 / 5
D. (3) / 5
E. 3





Two statements labeled I & II, follow each of the following questions. The statements contain
certain information. In the questions you do not actually have to compute an answer, rather you

II
Analytical
Reasoning
No. Of
Questions
15
Choose the correct answer for each
question and shade the
corresponding CIRCLE in the answer
sheet
A. 7
B. 12
C. 7 + 29
D. 86
E. It cannot be determined from
the information given.
C
B
D
A
A B C D E
A B C D E

A
B C D E
A
B C D E
Student Guide 146
have to decide whether the information given in the statements I. and II. is sufficient to find a
correct answer by using basic mathematics and every day facts?

1. What day of the week is today?

I. Today is March 25.
II. Akram left Pakistan on Wednesday.

A. Statement I. ALONE is sufficient but II. ALONE is not sufficient to answer this
question.
B. Statement II. ALONE is sufficient but I. ALONE is not sufficient to answer this
question.
C. Statements I. and II. TOGETHER are sufficient to answer the question but
NEITHER of them is sufficient ALONE.
D. Statements I. and II. COMBINED are NOT sufficient to answer the question and
additional information is needed to find the correct answer.



2. Can any of the four rivers be more than 300 meters wide?

I. The narrowest of the four rivers is 240 meters wide.
II. Average width of the four rivers is 300 meters.

A. Statement I. ALONE is sufficient but II. ALONE is not sufficient to answer this
question.
B. Statement II. ALONE is sufficient but I. ALONE is not sufficient to answer this
question.
C. Statements I. and II. TOGETHER are sufficient to answer the question but
NEITHER of them is sufficient ALONE.
D. Statements I. and II. COMBINED are NOT sufficient to answer the question and
additional information is needed to find the correct answer




3. If it is raining then there must be clouds. Are there clouds?

I. It is not raining.
II. It rained yesterday.

A. Statement I. ALONE is sufficient but II. ALONE is not sufficient to answer this
question.
B. Statement II. ALONE is sufficient but I. ALONE is not sufficient to answer this
question.
C. Statements I. and II. TOGETHER are sufficient to answer the question but
NEITHER of them is sufficient ALONE.
D. Statements I. and II. COMBINED are NOT sufficient to answer the question and
additional information is needed to find the correct answer.



Read the passage to answer the question 4-5
A map representing countries R, S, W, X, Y and Z is to be drawn. Adjacent countries
cannot have the same color in the map. The countries adjacent to each other are as
follows:
Each of R, S, X and Y is adjacent to W.
X is adjacent to Y.
A B C D E
A B C D E

A B C D E

Student Guide 147
Each of R and S is adjacent to Z.

4. Which of the following countries can be the same color as W?

A. S
B. X
C. Y
D. Z




5. Which of the following is a pair of countries that can be the same color?

A. R and S
B. S and W
C. W and X
D. X and Y


Questions 6 to 11 depends on the following passage

A college president wishes to select four members of a faculty-student committee as special
representatives to meet with the college's board of trustees.
The faculty-student committee consists of eight members four of which (F, G, H and I) are faculty
members whereas the other four (R, S, T and U) are students.
The president can select any four of the eight committee members as long as the following rules
are observed:
The four representatives must consist of exactly two faculty members and two students.
Either F or G must be one of the representatives but F and G both cannot be the representatives.
If R is a representative then H must also be a representative.
If T is a representative then G cannot be a representative.
6. If T is a representative but H is not a representative then the whole group can be
determined if it were also true that:

A. F is a representative.
B. I is a representative.
C. R is not a representative.
D. U is not a representative.


7. If R is a representative then which of the following CANNOT be a representative?

A. H
B. I
C. S
D. T


8. If G is a representative then which of the following can be the other three representatives?

A. F, S and U
B. H, I and R
C. H, R and S
D. I, R and U
A B C D E

A B C D E
A B C D E

A B C D E
Student Guide 148

9. If neither S nor U is a representative then which of the following is the pair of faculty-
member representatives?

A. F and G
B. F and H
C. F and I
D. G and H



10. If G, I and S are representatives then which of the following must also be a
representative?

A. H
B. R
C. T
D. U



11. If F and I are representatives then which of the following is not a representative?

A. I
B. S
C. U
D. R



Questions 12 to 14 depends on the following passage

At a congress of the Ruling Party, the seven top party leaders, who are all cabinet
ministers, are seated on a platform in order of rank the Prime Minister being in the
center. The closer a person is to the Prime Minister; the higher is his/her rank.
Moreover, a person sitting on the right of the PM outranks the one sitting equidistant on
the left of the PM. The seven leaders are T, U, V, W, X, Y, and Z.
Y is four places to the left of the Minister of Agriculture, who is two places to the right of
V.
Us neighbors are T and the Minister of Agriculture.
Z is two places to the left of W.
The Ministers of Education, Mining and Culture are seated together, in order, from left to
right.
The remaining Ministers are those of Social Welfare and Defense.

12. The fifth ranking person in the party hierarchy is:
A. Z, the Minister of Mining
B. Y, the Minister of Culture
C. W, the Prime Minister.
D. X, the minister of Defense.




13. How many of the seven party leaders outrank the Minister of Education?
A B C D E

A B C D E

A B C D E
A
B C D E
A B C D E

Student Guide 149
A. 3
B. 4
C. 5
D. 6




14. The lowest ranking Minister is
A. Minister of Social Welfare.
B. Minister of Defense.
C. Minister of Education.
D. Minister of Mining.




15. A meadow in springtime is beautiful, even if no one is there to appreciate it.
This statement would be a logical opposite to which of the following claims?
A. People will see only what they want to see.
B. Beauty exits only in the eyes of the beholder.
C. Beauty does not depend on seasons.
D. The greatest pleasure available to mankind is the contemplation of
beauty.


A B C D E
A
B C D E
A B C D E
Student Guide 150


Each sentence below has one or two blanks, each blank indicates that
something has been omitted. Beneath the sentence are five lettered words or
sets of words. Choose the word or set of words that, when inserted in the
sentence, best fits the meaning of the sentence as a whole.
1. Some illnesses such as smallpox, which have been almost eliminated in the United
States are still ____ in many places abroad.
A. discussed
B. prevalent
C. scarce
D. unknown
E. hospitalized



2. A recent study indicates that the crime rate in the United States remains ____ and
that one in three households ____ some form of major crime in any year
A. incredible ... witnesses
B. astronomical ... experiences
C. simultaneous ... perpetrates
D. unsuccessful ... initiates
E. defeated ... prosecutes



Each question below consists of a related pair of words or phrases, followed by
five lettered pairs of words or phrases. Select the lettered pair that best
expresses a relationship similar to that expressed in the original pair.

3. SALVAGE : TREASURE
A. settle : argument
B. incorporate : company
C. send : correspondence
D. rescue : victim
E. recycle : newspaper



4. CONTROVERSY : ARBITRATOR
A. peacemaker : conflict
B. artifact : anthropologist
C. game : referee
D. dispute : mediator
E. disease : pathologist




Read the passages and answer the questions given at its end:

III
VERBAL
Section
No. Of
Questions
10
Choose the correct answer for each
question and shade the
corresponding CIRCLE in the answer
sheet
A B C D E

A B C D E

A B C D E
A B C D E

Student Guide 151

We are profoundly ignorant about the origins of language and have to content ourselves
with more or less plausible speculations. We do not even know for certain when
language arose, but it seems likely that it goes back to the earliest history of man,
perhaps half a million years. We have no direct evidence, but it seems probable that
speech arose at the same time as tool making and the earliest forms of specifically
human cooperation. In the great Ice Ages of the Pleistocene period, our earliest human
ancestors established the Old Stone Age culture; they made flint tools and later tools of
bone, ivory, and antler; they made fire and cooked their food; they hunted big game,
often by methods that called for considerable cooperation and coordination. As their
material culture gradually improved, they became artists and made carvings and
engravings on bones and pebbles, and wonderful paintings of animals on the walls of
caves. It is difficult to believe that the makers of these Paleolithic cultures lacked the
power of speech. It is a long step Admittedly, from the earliest flint weapons to the
splendid art of the late Old Stone Age: the first crude flints date back perhaps to
500,000 B.C., while the finest achievements of Old Stone Age man are later than
100,000 B.C.; and, in this period, we can envisage a corresponding development of
language, from the most primitive and limited language of the earliest human groups to
a fully developed language in the flowering time of Old Stone Age culture.

How did language arise in the first place? There are many theories about this, based on
various types of indirect evidence, such as the language of children, the language of
primitive societies, the kinds of changes that have taken place in languages in the
course of recorded history, the behavior of higher animals like chimpanzees, and the
behavior of people suffering from speech defects. These types of evidence may provide
us with useful pointers, but they all suffer from limitations, and must be treated with
caution. When we consider the language of children, we have to remember that their
situations are quite different from that of our earliest human ancestors, because the
child is growing up in an environment where there is already a fully developed language,
and is surrounded by adults who use that language and are teaching it to him. For
example, it has been shown that the earliest words used by children are mainly the
names of things and people (Doll, Spoon, Mummy): but, this does not prove that
the earliest words of primitive man were also the names of things and people. When
the child learns the name of an object, he may then use it to express his wishes or
demands: Doll!: often means Give me my doll! Or Ive dropped my doll: pick it up
for me!; the child is using language to get things done, and it is almost an accident of
adult teaching that the words used to formulate the childs demands are mainly nouns,
instead of words like Bring! Pick up!; and so on.

5. The main idea of this excerpt is
(A) to provide evidence of the origin of language.
(B) to present the need for language.
(C) to discuss how early man communicated.
(D) to present the culture of early man.
(E) to narrate the story of English.



6. Theories of the origin of language include all of the following EXCEPT
(A) changes occurring through the years.
(B) the need to communicate.
(C) language of children.
(D) the first mans extensive vocabulary.
(E) communication among primitive men.

A B C D E
Student Guide 152


7. The purpose of the discussion of the word, Doll, is intended to
(A) Trace the evolution of a noun.
(B) Support the fact that naming things is most important.
(C) Indicate how adults teach language to children.
(D) Show the evolution of many meanings for one word.
(E) Evince mans multiple uses of single words



8. The implication of the author regarding the early elements of language is that
(A) There were specific real steps followed to develop our language.
(B) Care must be exercised when exhuming what we consider the roots of
language.
(C) We owe a debt of gratitude to the chimpanzee contribution.
(D) Adults created language in order to instruct their children.
(E) Language was fully developed by primitive man.



9. If we accept that primitive man existed for a very long period of time without
language, then we may assume that
(A) language is not necessary to mans existence.
(B) language developed with the developing culture of primitives.
(C) primitives existed in total isolation from one another.
(D) children brought about a need for language.
(E) mankind was not intended to communicate.



10. After a reading of this article, one might infer that
(A) society creates problems with language.
(B) language is for adults to instruct children.
(C) society uses language to improve itself.
(D) with the evolution of language came wisdom.
(E) language brings power.






A B C D E

A B C D E

A
B C D E
A B C D E
A B C D E

Student Guide 153






Answer Keys to
Drill Tests
Student Guide 154
DRILL TEST I - ANSWER KEY
Section-I Quantitative

1.
2.
3.
4.
5.
6.
7.
8.
9.
10.

Section-II Analytical Reasoning

1.
2.
3.
4.
5.
6.
7.
8.
9.
10.
11.
12.
13.
14.




15.
16.
17.
18.
19.
20.

Section-III Verbal
1.
2.
3.
4.
5.
6.
7.
8.
9.
10.
11.
12.
13.
14.
15.
16.
17.
18.
19.
20.

B C E

A
D E

B C

A D

C

D

B C

A D E

A D

B
A

C
A D

C

D E

B C

D E B

A E

C

A E

B

D

B
C A

B
A E

B
C
C D E

B
A D

C A E

B
C A

B
D

B
D B
A
D

B

A

C
C
C A D

E
E
E

E

A E

B C



B

B
B

A


A

A

A
A

E

E

E
E

D

C
C
D

D
E

D
D
A
D E C

A
E

B C

A
D E

B

A
D E

C
A
D

B C

A E

B

D
A
D E

B

A
D E

C

A
D E

C

D E

B C

A
E

B C

E

B C

D
A
D E

C

A
E

B C
A
D E B

A
D E

B

D E B C

E

B C

D
D E

C

D E

C

D E

C
A
E

B C


C A

E

B

C A

E

B
C D E

B

A
A
A














Student Guide 155
DRILL TEST II - ANSWER KEY

Section-I Quantitative

1.
2.
3.
4.
5.
6.
7.
8.
9.
10.

Section-II Analytical Reasoning

1.
2.
3.
4.
5.
6.
7.
8.
9.
10.
11.
12.
13.
14.




15.
16.
17.
18.
19.
20.

Section-III Verbal
1.
2.
3.
4.
5.
6.
7.
8.
9.
10.
11.
12.
13.
14.
15.
16.
17.
18.
19.
20.







C E

A
D E

C

A

C
D

B C

A D E

A D

B
A

C
A D

C
D E

B

E B

A E

C

D E

B

D E

A E

B
A E

B
C
C D

B
D

B
C A

B
A

B
D

B
C B
D B

A

B
C
C
A D

E
E
E

E
E

B C




B
B

A


A

A

A

A
A

E

E

E
E

D

C
C
D

D
E

D
D

C
D
D
D
A
D E B

A
E

B C

A
D E

C

A
E

B C

A
D

B C

A E

B

D
A
D E

D E

C

A
D E

D

B C

A
E

B

E C

D
A
E

C

A
E

B
A
D E

A
D E

B
D E B

E

C

D
D

B C

D E

C

D E

B
A
E

B

A

E

B

C

E

B
C E

B
A
A
A
A

C

A

C
B


C

C
C
B

B
B

A
A
A
A
D

D
D

D
D

A














Student Guide 156
DRILL TEST III - ANSWER KEY

Section-I Quantitative

1.
2.
3.
4.
5.
6.
7.
8.
9.
10.
11.
12.
13.
14.
15.
16.
17.
18.
19.
20.
Section-II Analytical Reasoning
1.
2.
3.
4.
5.
6.
7.
8.
9.
10.


11.
12.
13.
14.
15.

Section-III Verbal
1.
2.
3.
4.
5.
6.
7.
8.
9.
10.
A B

C

D E

A

B

C

D E

A

B

C

D E

A

B

C

D E

A

B

C

D E

A

B

C

D E

A

B

C

D E

A

B

C

D E

A

B

C

D E

A

B

C

D E

A B

C

D E

A

B

C

D E

A

B

C

D E

A

B

C

D E

A

B

C

D E

A

B

C

D E

A

B

C

D E

A

B

C

D E

A

B

C

D E

A

B

C

D E

A B

C

D E

A

B

C

D E

A

B

C

D E

A

B

C

D E

A

B

C

D E

A

B

C

D E

A

B

C

D E

A

B

C

D E

A

B

C

D E

A

B

C

D E

A B

C

D E

A

B

C

D E

A

B

C

D E

A

B

C

D E

A

B

C

D E

A

B

C

D E

A

B

C

D E

A

B

C

D E

A

B

C

D E

A

B

C

D E

A

B

C

D E

A

B

C

D E

A

B

C

D E

A

B

C

D E

A

B

C

D E

Student Guide 157

DRILL TEST IV - ANSWER KEY
Section-I Quantitative

1.
2.
3.
4.
5.
6.
7.
8.
9.
10.
11.
12.
13.
14.
15.
16.
17.
18.
19.
20.
Section-II Analytical Reasoning
1.
2.
3.
4.
5.
6.
7.
8.
9.
10.


11.
12.
13.
14.
15.

Section-III Verbal
1.
2.
3.
4.
5.
6.
7.
8.
9.
10.

A B

C

D E

A

B

C

D E

A

B

C

D E

A

B

C

D E

A

B

C

D E

A

B

C

D E

A

B

C

D E

A

B

C

D E

A

B

C

D E

A

B

C

D E

A B

C

D E

A

B

C

D E

A

B

C

D E

A

B

C

D E

A

B

C

D E

A

B

C

D E

A

B

C

D E

A

B

C

D E

A

B

C

D E

A

B

C

D E

A B

C

D E

A

B

C

D E

A

B

C

D E

A

B

C

D E

A

B

C

D E

A

B

C

D E

A

B

C

D E

A

B

C

D E

A

B

C

D E

A

B

C

D E

A B

C

D E

A

B

C

D E

A

B

C

D E

A

B

C

D E

A

B

C

D E

A

B

C

D E

A

B

C

D E

A

B

C

D E

A

B

C

D E

A

B

C

D E

A

B

C

D E

A

B

C

D E

A

B

C

D E

A

B

C

D E

A

B

C

D E

Student Guide 158









SAMPLE TEST
General
















Note: The Sample Test does not include quantitatively the same number
of questions as there would be in the actual papers. They are merely
meant to provide conceptual guidance to the users or prospective
candidates.
Student Guide 159



I
VERBAL
Section
No. Of
Questions
15
Choose the correct answer for each
question and shade the
corresponding CIRCLE in the answer
sheet

Each sentence bel ow has one or two bl anks, each bl ank i ndi cates
that somethi ng has been omi tted. Beneath the sentence are fi ve
l ettered words or sets of words. Choose the word or set of words
that, when i nserted i n the sentence, best fi ts the meani ng of the
sentence as a whol e.
1. Despite the millions of dollars spent on improvements, the telephone system in India
remains ________ and continues to ___________ the citizens who depend upon it.

A. Primitiveinconvenience
B. Bombastic...upset
C. Suspicious...connect
D. Outdated...elate
E. Impartial...vex

2. Unlike the images in symbolist poetry which are often vague and _______ , the
images of surrealist poetry are startlingly ________ and bold.

A. extraneous...furtive
B. trivial...inadvertent
C. obscure...concrete
D. spectacular...pallid
E. symmetricalvirulent

3. A good trial lawyer will argue only what is central to an issue, eliminating
___________ information or anything else that might __________ the client.

A. Seminal...amuse
B. Extraneous...jeopardize
C. Erratic...enhance
D. Prodigious...extol
E. Reprehensibleinitiate

4. Pollen grains and spores that are 200 millions old are now being extracted from
shale and are ____________ the theory that the breakup of the continents occurred
in stages; in fact, it seems that the breakups occurred almost __________ .

A. refining...blatantly
B. reshaping...simultaneously
C. countermanding...imperceptibly
D. forging...vicariously
E. supporting...haphazardly

Student Guide 160
Each question below consists of a related pair of words or phrases, followed by
five lettered pairs of words or phrases. Select the lettered pair that best
expresses a relationship similar to that expressed in the original pair.

5. DETENTION : RELEASE ::

A. viciousness : attack
B. calamity : repair
C. qualification : employ
D. induction : discharge
E. therapy : confuse

6. PONDEROUS : WEIGHT ::

A. eternal : temporality
B. convincing : decision
C. gargantuan : size
D. ancient : value
E. prototypical : affection

7. FEBRILE : ILLNESS ::

A. tenacious : astonishment
B. juvenile : maturity
C. classic : cultivation
D. eccentric : discrimination
E. delusional : insanity

8. EQUIVOCATION : MEANING ::

A. feint : intention
B. secrecy : stealth
C. geniality : amiability
D. travesty : insight
E. refinement : innovation

Choose the lettered word or phrase that is most nearly opposite in
meani ng to the word i n capi tal l etters.

9. WHIMSICAL :

A. chivalrous
B. perfect
C. predictable
D. hidden
E. backward


10. REVERE :

A. collide
B. succumb
C. threaten
D. divide
E. despise
Student Guide 161

11. INURED :

A. authoritative
B. dissolute
C. bereft
D. sensitive
E. taxing

12. ALACRITY :

A. skullduggery
B. reluctance
C. interment
D. bellicosity
E. specificity

Read the passages and answer the questions asked at its end.

Art, like words, is a form of communication. Words, spoken and written, render
accessible to humans of the latest generations all the knowledge discovered by the
experience and reflection, both of preceding generations and of the best and foremost
minds of their own times. Art renders accessible to people of the latest generations all
the feelings experienced by their predecessors, and those already felt by their best and
foremost contemporaries. Just as the evolution of knowledge proceeds by dislodging and
replacing that which is mistaken, so too the evolution of feeling proceeds through art.
Feelings less kind and less necessary for the well-being of humankind are replaced by
others kinder and more essential to that end. This is the purpose of art, and the more
art fulfills that purpose the better the art; the less it fulfills it, the worse the art.

13. The author develops the passage primarily by

A. theory and refutation
B. example and generalization
C. comparison and contrast
D. question and answer
E. inference and deduction

14. According to the author, knowledge is

A. evolutionary and emotional
B. cumulative and progressive
C. static and unmoving
D. dynamic and cyclical
E. practical and directionless

15. According to the passage, all of the following are true EXCEPT:

A. Art is a form of communication.
B. Art helps to refine sensibilities.
C. Art is a repository of experience.
D. Real art can never be bad.
E. Art is a progressive human endeavor.
Building Standards in Education and Professional Testing 162
II
Analytical
Reasoning
Section
No. Of
Questions
20
Choose the correct answer for
each question and shade the
corresponding CIRCLE in the
answer sheet

Questions 16-19 are based on the following.

The Western Derby is a race held annually at Bayshore Racetrack. There are eight
gates at the racetrack, but only seven horses are entered in this raceJulius
Caesar, King's Bounty, Longshot, Man Among Boys, Nocturnal, Odyssey, and
Phantom. One of the gates is left empty. The horses are at the gate, waiting for the
race to begin.

Gate 1, on the inside of the racetrack, is occupied.
Phantom is at a gate inside of Nocturnal.
The number of gates separating Julius Caesar and King's Bounty equals the number
of gates separating Longshot and Man among Boys.
Nocturnal and Odyssey are next to each other.


16. If Odyssey is at Gate 2, which of the following must be true?

A. Nocturnal is at the innermost gate.
B. King's Bounty is at the outermost gate.
C. A horse occupies the outermost gate.
D. Phantom is at the innermost gate.
E. The outermost gate is not empty.

17. Which of the following is a possible assignment for the horses, from the inside to
the outside?

A. Phantom, King's Bounty, Julius Caesar, Odyssey, Nocturnal, Man Among
Boys, Longshot, vacant
B. vacant, Phantom, Julius Caesar, Longshot, King's Bounty, Man Among Boys,
Nocturnal, Odyssey
C. Longshot, Man Among Boys, Nocturnal, vacant, Phantom, Odyssey, King's
Bounty, Julius Caesar
D. Julius Caesar, King's Bounty, Longshot, Phantom, vacant, Man Among Boys,
Nocturnal, Odyssey
E. Phantom, Julius Caesar, Nocturnal, vacant, Odyssey, King's Bounty,
Longshot, Man Among Boys


18. If Julius Caesar is at Gate 6, King's Bounty is at Gate 7, and Odyssey is at Gate
4, which of the following must be true?
I. Longshot is at Gate 1.
II. Nocturnal is at Gate 5.
III. Man Among Boys is at Gate 2.
IV. Gate 8 is vacant.
Building Standards in Education and Professional Testing 163


A. I and II only
B. II and III only
C. II and IV only
D. I, II, and III only
E. I, II, III, and IV

19. If Julius Caesar and King's Bounty are at the second and fourth gates,
respectively, all of the following can be true EXCEPT

A. Phantom is at Gate 1
B. Man Among Boys is at Gate 3
C. Longshot is at Gate 6
D. Odyssey is at Gate 7
E. Nocturnal is at Gate 7

20. Studies have shown that families who install smoke detectors and own fire
extinguishers have a reduced risk of losing a child in a house fire. Therefore, no
family who installs smoke detectors and owns a fire extinguisher will lose a child
in a house fire.

Of the following, the best criticism of the argument above is that the argument
does not

A. take into account the possibility of losing a child in a house fire despite all
precautionary measures
B. indicate that fire extinguishers are effective during early stages of a fire
C. cite the fact that smoke detectors have proven to be effective in waking
sleeping children during a house fire
D. differentiate between the two major causes of house fires: cooking and
heating
E. take into account that families who buy smoke detectors are also more likely
to purchase fire insurance

21. LSD is a drug known to cause synesthesia, a phenomenon in which sensory input
somehow becomes interchanged in the brain: a person with synesthesia might
smell a symphony, hear sun light, or taste a pinprick. While most cases are drug
induced, some people suffer from synesthesia in various forms since birth.
Which of the following can be most safely inferred from the information above?

A. Synesthesia is not always a drug-induced phenomenon.
B. Some great artists of this century have been known for their synesthetic
proclivities.
C. LSD is an addictive drug.
D. Synesthesia is rarely bothersome to those who experience it.
E. Synesthesia at birth is a result of mothers who have tried LSD.

22. Palindromes are easier to solve than acrostics, but acrostics are more difficult to
create than palindromes. Rebuses are more difficult to solve than acrostics, yet
rebuses are easier to create than palindromes.
Building Standards in Education and Professional Testing 164
If the above information is true, then it must also be true that

A. acrostics are more difficult to create than rebuses
B. palindromes are more difficult to solve than rebuses
C. rebuses are easier to solve than acrostics
D. acrostics are easier to create than rebuses
E. rebuses are easier to solve than palindromes
Questions 23-25 are based on the following.

A university has a procedure for registering and recording complaints. Due to
strict bureaucratic regulations, the following system of passing complaints must be
observed:

A is the first registrar to receive all incoming complaints.
F is the recorder and final administrator to handle a complaint.
Personnel B, C, D, and E may pass complaints only as follows:
A to B
B to either C or D
C to either B or E
D to C
E to either D or F

23. Which is an acceptable path for a complaint to follow, passing from A?

A. B to C to D to F
B. B to D to C to F
C. B to C to E to F
D. B to E to F
E. D to C to F

24. If a complaint is received and is handled by each personnel member only one
time, which of the following could be one of the passes?

A. A to C
B. C to B
C. C to F
D. D to C
E. E to D

25. Between which two personnel may a complaint pass by means of two different
paths without any duplication of passes?

A. B to E
B. C to D
C. C to E
D. D to B
E. E to B

Building Standards in Education and Professional Testing 165
Questions 26-31 are based on the following.

In a basebal l fi el d, one team can practi ce at a ti me. There are seven teams the
Aces, the Bears, the Cubs, the Ducks, the Eagl es, the Fal cons, and the Gi ants.
The basebal l fi el d i s open seven eve ni ngs a week f rom Monday to Sunday
(Sunday bei ng consi dered the l ast day of the week), and the al l ocati on of
practi ce ti mes i s governed by the fol l owi ng rul es:

On any eveni ng, onl y one team can pl ay.
The Aces must practi ce on Monday.
The Ducks practi ce exactl y one day before the Fal cons practi ce.
The Fal cons practi ce exactl y one day before the Gi ants practi ce.
The Cubs and the Bears must practi ce earl i er i n the week than the Eagl es.

26. The latest day in the week that the Bears can practice is

A. Tuesday
B. Wednesday
C. Thursday
D. Friday
E. Saturday

27. If a person went to the baseball field on three consecutive evenings, he or she
could see which of the following teams in the order listed?

A. the Falcons, the Giants, the Cubs
B. the Falcons, the Giants, the Ducks
C. the Aces, the Ducks, the Cubs
D. the Bears, the Cubs, the Falcons
E. the Ducks, the Eagles, the Falcons

28. One week, the Cubs practiced on Wednesday and the Ducks practiced the next
day. That week, the Bears must have practiced on

A. Monday
B. Tuesday
C. Friday
D. Saturday
E. Sunday

29. If the Giants practice on Thursday, the Eagles and the Ducks must practice on
which days, respectively?

A. Sunday and Tuesday
B. Saturday and Tuesday
C. Friday and Wednesday
D. Wednesday and Thursday
E. Tuesday and Monday

30. If the Falcons practice on Saturday, the Eagles must practice on what day?

A. Tuesday
Building Standards in Education and Professional Testing 166
B. Wednesday
C. Thursday
D. Friday
E. Sunday

31. The practice schedule has to adhere to which of the following?

A. The Ducks practice earlier in the week than the Eagles.
B. The Falcons practice on a later day than the Eagles.
C. The Falcons practice earlier in the week than the Giants.
D. The Cubs practice earlier in the week than the Ducks.
E. The Bears practice earlier in the week than the Cubs.

32. Wine, cheese, butter, and raisins are all examples of early techniques to preserve
food. In modern times, food scientists have developed other techniques such as
dehydration, hermetic sealing, and radiation. Of these, radiation is the most con-
troversial because preliminary studies have shown that radiation alters the
natural chemical bonds in fruits and vegetables. Instead of providing salutary
effects, eating radiated produce may well introduce irritating chemicals into the
body, creating a possible health hazard.
Whi ch of the fol l owi ng, i f true, supports the concl usi on that eati ng radi ated
produce poses a possi bl e heal th hazard?
A. Radiation affects only those chemical bonds associated with water, that is,
hydrogen and oxygen.
B. Radiation kills microorganisms that hasten food decay.
C. The radiation-induced bonds are unlike any of those found in non-radiated
produce.
D. Certain microorganisms, namely those found in yogurt cultures, are essential
for proper digestion.
E. Radiation has no effect on foods preserved by drying.

33. Blue Blood, Inc., is a private blood products company that buys blood only from
qualified donors. To qualify, a person must weigh at least 105 pounds, must not
have taken malaria medication in the last three years, must never have had
hepatitis, and must never have used intravenous drugs. Blue Blood nurses know
that traveling has an effect on the possibilities for blood donation: Everyone who
travels to Malaysia is required to take malaria medication; no one who enters
Singapore can have ever used intravenous drugs; everyone traveling to Gorisimi
gets hepatitis.
Whi ch of the fol l owi ng si tuati ons woul d not au tomati cal l y di squal i fy a person
from sel l i ng bl ood to Bl ue Bl ood?

A. traveling to Malaysia two years ago
B. having once weighed 110 pounds and now weighing 95 pounds
C. being denied admission to Singapore
D. traveling to Gorisimi five years ago
E. using intravenous drugs that were legal at the time

Building Standards in Education and Professional Testing 167
34. Before marriage, couples should be tested for AIDS and any other sexually
communicable diseases. Negative results will guarantee the health and safe-ness
of their marriage.
Whi ch of the fol l owi ng i s an assumpti on of the argument i n the passage
above?

A. Current state laws require couples who are planning to get married to be
tested for infectious disease in order to prevent possible health problems in
the future.
B. There are many infectious diseases that can be sexually transmitted from
one individual to another.
C. Fortunately even if a test proves positive for a communicable disease,
couples can still lead healthy marriages by taking the proper precautions.
D. Due to advances in medical research over the years, infectious diseases that
used to be fatal can now be effectively treated.
E. All the diseases detectable through testing have no incubation period and the
results of these tests can immediately indicate whether or not the individual
has the disease.

Question 35 is based on the following.

Nine athletes attend a sports banquet. Three of the athletes}, K, and Lare varsity
football players; two of the athletesM and Nare varsity basketball players. The
other four athletes O, P, Q, and Rbelong to the hockey club. All nine athletes will
be seated at three small tables, each seating three athletes. The athletes must be
seated according to the following rules: O and J do not sit at the same table.
P si ts together wi th at l east one of K or M.
There can be at most onl y one footbal l pl ayer at a tabl e.
There can be at most onl y one basketbal l pl ayer at a tabl e.

35. Suppose just one varsity athlete sits at a certain table, and that athlete happens
to be J. If so, who else sits with J?

A. P, Q
B. P, R
C. Q, R
D. O, Q
E. O, P






Building Standards in Education and Professional Testing 168




36. If vy = 9, then y
2
- vy =

A. v 3-9
B. 0
C. 9-v 3
D. 6552
E. 6561

37. If (x+3)/6 = 12/(x+4), what is the positive value of x?

A. 2
B. 3
C. 5
D. v60
E. 12

38. Cindy wants to paint her office. She can buy three cans of the same-priced paint
and three identical brushes for $21, or she can buy four cans of the same paint
and one brush for $22. How much does a can of paint cost?

A. $2
B. $3
C. $4
D. $5
E. $6

39. Which of the following must be true?

I . (25 - 81) = (5 - 9) (5 + 9)
I I . 7(9 + 6) = 7(9) + 7(6)
I I I . 6 (3 - 1) = (6 3) - ( 61)

A. I only
B. II only
C. III only
D. I and II only
E. I, II, and III


40. The sum of a and 9 - 2a is less than 8. Which of the following is (are) the
value(s) of a?

I. a<- 1
II. a< 1
III
Quantitative
Section
No of
Questions
15
Choose the correct answer for
each question and shade the
corresponding CIRCLE in the
answer sheet
Building Standards in Education and Professional Testing 169
III. a>1

A. I only
B. II only
C. III only
D. I and II only
E. I and III only

41. Susan is having a party. At 7:00 P.M., guests begin arriving at a uniform rate of
8 people every 15 minutes. If this pattern continues, how many guests will have
arrived by 9:30 P.M.?

F. 10
G. 20
H. 40
I. 64
J. 80

42. For positive integers p and q, if p
2
+ 2q
2
= 41, and 2p
2
+q
2
= 34, then p
2
=

A. 2.5
B. 7
C. 3
D. 9
E. 16

43. If a:b is 7:6 and 3b:2c is 2:3, what is c/a ?

A. 14/27
B. 7/9
C. 6/7
D. 9/7
E. 27/14



Building Standards in Education and Professional Testing 170
44. In the figure above, if the radius of the circle is 8, and triangle TRS is inscribed
in the circle, then the length of arc TRS is

A. 16p/3
B. (32 p)/3
C. 16 p
D. (128 p)/3
E. 64 p

45. For developing pictures, XYZ Photo Lab charges a service fee of $3 for every
order it receives in addition to a printing fee. If the order consists of 12 pictures
or less, the printing fee per picture is $0.36. If the order consists of more than
12 pictures, the printing fee per picture is $0.24. What is the total cost per
picture for an order consisting of 30 pictures?

A. $0.11
B. $0.24
C. $0.34
D. $0.46
E. $3.24

46. Lisa found an easy way to add up a sequence of positive even integers with an
even number of terms. She formed pairs of equal sums by adding the first
integer to the last, the second integer to the next-to-last, and so on. She then
computed the total by adding these equal sums. If the total Lisa obtained was
930, how many terms were there in the sequence of positive even integers if
the sequence started with the number 2?

A. 30
B. 39
C. 40
D. 60
E. 465

47. December is the busiest month at Lamont's Gift Shoppe, where sales in
December are 40 percent higher than average. If sales in February are typically
20 percent lower than average, what is the ratio of February sales to December
sales?

A. 1:2
B. 4:2
C. 4:5
D. 4:7
E. 6:7

48. How many 4-digit numbers are there that consist of only odd digits?

A. 20
B. 625
C. 1,024
D. 4,500
E. 5,000
Building Standards in Education and Professional Testing 171

49. For some integer m, let [m] be defined by the equation {m} = m (1- m). If n +
1 = {n + 1}, then n =

A. -2
B. -1
C. 0
D. 1
E. 2


50. Box A and box B have 6 cards each. Each card is marked with one integer, 1
through 6. Both boxes can have more than one card with the same integer, but
the sum of all the integers in each box must be 18. Two of the cards in box/1
are 6's and two of the cards in box B are 5's. If one card is drawn from box A
and one from box B, but neither a 6 nor a 5 is drawn, what is the largest pos-
sible sum of the integers on the cards drawn from the two boxes?

A. 3
B. 4
C. 7
D. 8
E. 12

You might also like